˘ˇˆ˙ - ain shams universitymed.asu.edu.eg/uploads/med/student affairs/iee_io__2.pdf• lt. lower...

103
Faculty Of Medicine - ASU Student Union Scientific Committee (SUSC) Examination Note - 4 th year Page | 1

Upload: vannguyet

Post on 20-Mar-2018

220 views

Category:

Documents


3 download

TRANSCRIPT

Page 1: ˘ˇˆ˙ - Ain Shams Universitymed.asu.edu.eg/uploads/med/Student Affairs/IEE_IO__2.pdf• Lt. Lower lobe bronchiectesis • Recurrent pul. Embolism • Chronic broncitis (May 96)

Faculty Of Medicine - ASU

Student Union Scientific Committee (SUSC)

Examination Note - 4th year P a g e | 1

������ ���� � ���

� �� �� ��� ����

����

������ ��� �� !�"�#$�� ��%&�

Page 2: ˘ˇˆ˙ - Ain Shams Universitymed.asu.edu.eg/uploads/med/Student Affairs/IEE_IO__2.pdf• Lt. Lower lobe bronchiectesis • Recurrent pul. Embolism • Chronic broncitis (May 96)

Faculty Of Medicine - ASU

Student Union Scientific Committee (SUSC)

Examination Note - 4th year P a g e | 2

Content :

- Internal Medicine …………… 3 • Cases ……………………………. 25 • Complete exams …………. 38

- Pediatrics …………………..…. 98 • Complete exams ……….. 110

Page 3: ˘ˇˆ˙ - Ain Shams Universitymed.asu.edu.eg/uploads/med/Student Affairs/IEE_IO__2.pdf• Lt. Lower lobe bronchiectesis • Recurrent pul. Embolism • Chronic broncitis (May 96)

Faculty Of Medicine - ASU

Student Union Scientific Committee (SUSC)

Examination Note - 4th year P a g e | 3

IIIIIIIInnnnnnnntttttttteeeeeeeerrrrrrrrnnnnnnnnaaaaaaaallllllll

MMMMMMMMeeeeeeeeddddddddiiiiiiiicccccccciiiiiiiinnnnnnnneeeeeeee

Page 4: ˘ˇˆ˙ - Ain Shams Universitymed.asu.edu.eg/uploads/med/Student Affairs/IEE_IO__2.pdf• Lt. Lower lobe bronchiectesis • Recurrent pul. Embolism • Chronic broncitis (May 96)

Faculty Of Medicine - ASU

Student Union Scientific Committee (SUSC)

Examination Note - 4th year P a g e | 4

CARDIOLOGY 1. Enumerate causes of hypertension. (Nov. 75) 2. What are the compl. Of mitral stenosis. (Nov. 75) 3. How would you ttt a case of coronary thrombosis. (Nov. 75) 4. Describe the cl. picture and diagnosis of rtheumatic fever. (Nov. 76) 5. Give the ttt of left vent. Failure. (Nov. 76) 6. Describe: (Nov. 77)

• Anatomy of arch of aorta. • Causes of chronic corpulmonale. • Signs and symptoms of cardiac asthma. • Treatment of rheumatic fever.

7. Give an account on: • The management of myocardial infarction. (Nov. 81, 87) • The complications of subacute bacterial endocarditis. (Nov. 87) • The causes and diagnostic features of A.F. (Nov. 76, 87)

8. Causes and diagnostic features of curable hypertension. (Nov. 78) 9. Describe the surface anatomy of the heart. (Nov. 79) 10. Give an account on Aetiology, C.P., Diagnosis, Investigations and treatment of subacute bacterial

endocarditis. (Nov. 79) 11. What are the causes & ttt of A.F. (Nov. 80, 81) 12. Give the causes of pericarditis. 13. Describe the C.P. of left vent.failure. 14. Enumerate the causes and C.P. of pulmonary hypertension. 15. Discuss the management of femoral thrombosis. (Nov. 81, 83) 16. Discuss the possible causes, C.P. & differential diagnostic of acute pulmonary embolism. 17. Discuss the management of: (Nov. 82)

• Acute myocardial infraction. • Pericarditis • Rapid A.F.

18. Give short note on oedema of R.Arm. (May 83) 19. What are the compartments of the mediastinum and what is the imp. Structure in each. Discuss the

causes, C.P. & investigations of mediastinal syndrome. (Nov. 83) 20. Discuss the C.P. and ttt of acute coronary thrombosis (Nov. 83) 21. Enumerate causes of systemic hypertension. Give the name, dose, side effects of three commonly

used antihypertensive drugs. 22. Describe the emergency ttt of a patient with (in details):

• Cardiac asthma • Paroxysmal tachycardia (May 84)

23. Give an account on causes, clinical features & comps of aortic insufficieny. (Nov. 84)

24. Give the details of emergency ttt of a patient with supravent. Tachycardia. (May, 85)

25. Give five comps. Of acute myocardial infraction that are likely to happen in the 1st few days. How would you diagnose each and how would you act to their development. (May 85)

26. Give the details of emergency ttt of: • Acute left vent failure. • Hypertensive encephalopathy (May, 86).

27. How can you advise a person about prophylaxis against rheumatic fever. 28. Enumerate 8 comps. of acute myocardial infraction.

Page 5: ˘ˇˆ˙ - Ain Shams Universitymed.asu.edu.eg/uploads/med/Student Affairs/IEE_IO__2.pdf• Lt. Lower lobe bronchiectesis • Recurrent pul. Embolism • Chronic broncitis (May 96)

Faculty Of Medicine - ASU

Student Union Scientific Committee (SUSC)

Examination Note - 4th year P a g e | 5

29. Describe the C.P. of a patient with conistrictive pericarditis. (Dec. 88) 30. Give the causes of secondary hypertension. (Dec. 88) 31. Give the causes of mitral incompetence. 32. What are the methods used to abort an attack of paroxysmal supravent. (atrial) tachycardia.

(May 89). 33. Discuss the C.P. of recent uncomplicated myocardial infarction. & Give the expected results of

confirmatory investigations. 34. Define malignant hypert. & Describe its manifestations. (May 93) 35. Enumerate the endocrinal disorder that may cause H.P.N.

• Give the cause of H.P.N in each. (Nov. 93) 36. Discuss the cause & management of regular tachycardia. (Nov. 93) 37. What diatery advice would you give to a pt., who has recovered from myocardial infraction, give

your reasons. (Nov. 93) 38. Enumerate causes of pericarditis. (May 94) 39. Discuss the causes, C/P & management of inf. Endocarditis. (Dec. 94) 40. Mention the essential symptoms & signs of acute myocardial infraction (Nov. 95)

• Select 3 serious signs & discuss the immediate management for each. 41. Mention 2 main causes of raised V.P. & how can you differentiate between the 2 causes from

examination of the neck vein alone. (May 96)

CHEST

1. Describe the clinical manifestation of bronchial asthma. 2. Enumerate causes of haemoptysis. 3. How would you treat a case of status asthmaticus. (Nov. 75) 4. Write full account on cyanosis. (Nov. 76) 5. Give the C.P. of pneumothorax. 6. Give the treatment of bronchial asthma. (Nov. 76) 7. Discuss:

• Indications of bronchoscopy. • Signs & symptoms of bronchiectases. • Management of case of lobar pneumonia. (Nov. 77)

8. Discuss • Causes of lung abscess. • The diagnostic features of diff. Types of penumothorax. • The ttt of status asthmaticus.

9. What are the causes of pulmonary collapse? (Nov. 79) 10. Describe the signs & symptoms of lung abscess. (Nov. 79) 11. How can you treat status asthmaticus? (Nov. 79) 12. What are the common presentation of pulmonary T.B.? (Nov. 80) 13. How can you treat bronchial asthma? (Nov. 80) 14. Describe the surface anatomy of the lung and pleura. (May 81) 15. How can you treat a case of pulmonary T.B.? (May 81) 16. Describe the C.P. of bronchopneumonias. Discuss their man agement. 17. Describe the C.P. & management of empyema. (Nov. 82) 18. Discuss causes of dullness on the lower half of the Rt. side of the chest.

• How can you arrive at the diagnosis of each. (Nov. 82) 19. Discuss the clinical types, manifestations & ttt of pulmonary tuberculosis. (Nov.

83) 20. How would you suspect then confirm the diagnosis of:

• Pulmonary embolism • Bronchogenic carcinoma

Page 6: ˘ˇˆ˙ - Ain Shams Universitymed.asu.edu.eg/uploads/med/Student Affairs/IEE_IO__2.pdf• Lt. Lower lobe bronchiectesis • Recurrent pul. Embolism • Chronic broncitis (May 96)

Faculty Of Medicine - ASU

Student Union Scientific Committee (SUSC)

Examination Note - 4th year P a g e | 6

21. Discuss: • Comps & side effects of antituberculous drugs. • Diagnosis of cystic lung disease. • C.P. of spontaneous pneumothorax (May 85)

22. Give 3 basic requirements for effective antituberculous therapy. (Dec 88)

23. Enumerate the characters of malignant pleural effusion. (Dec. 88) 24. Enumerate 5 criteria you depend upon for diag. Of T.B. pleural effusion. (May 85) 25. Give 5 points to suspect the diag. Of amoebic lung abscess. (May 85) 26. What symptoms make you suspect the diag. Of acute massive pulmonary embolism. Give the main

lines of management. (Nov. 89) 27. Give two possible clinical chest presentations for a bronchial carcinoma & give briefly the points that

may differentiate them from similar non-malignant conditions. (Nov. 89, II) 28. A 40 years old patient presented with pyrexia and dullness on the upper lobe of the Rt. lung:

• Give 3 possible diag. • Give the expected results of 3 investigations • Give the management of one of the possible diagnosis (Nov. 91)

29. Give 3 clinical differentiating criteria between lung abscess and pancoast tumour. (Nov. 91) 30. Give the management of acute sever B.A. (Nov. 93)

• What are the differentiating points between B.A and cardiac ath. 31. Give the diagnostic value of sputum (May 93) 32. Describe the management of:

• Pulmonary embolism • Acute pulmonary oedema

33. Write short essay on classification of B.A (Dec. 94) 34. Discuss causes of hemoptysis (Dec. 94) 35. In pt. Complaining of cough, mention how can you from his. alone suspects that he may have…

• Lt. Lower lobe bronchiectesis • Recurrent pul. Embolism • Chronic broncitis (May 96)

36. Classify the causes of dyspnea & Select 2 causes you mention & state.. • How can you recognize each of the 2 from the other causes of dyspnea • How would you manage each of the 2 causes? (Nov. 95)

NEUROLOGY

1. Describe the anatomy of the facial nerve. What are the signs & symptoms of paralysis? 2. Give the ttt of epilepsy (Nov. 76) 3. Discuss the causes of polyneuritis (Nov. 76) 4. Give an account on:

• Causes of polyneuropothy • C.P. of cerebellar tumours (Nov. 77)

5. Give an account on the causes, classification & ttt of amnesia (Nov. 77) 6. Give the anatomy of the spinal cord at dorsal 10 thoracic including its tracts, blood supply. Discuss

the causes, the diagnostic features of paraplagia at this level. (Nov. 78, I) 7. What are the causes of spinal cord compression (Nov. 79, 80) 8. Describe the signs of cerebellar lesions (Nov. 79) 9. How would you ttt a case of parkinsonism (Nov. 79) 10. Discuss the causes and differential diag. Of atrophy of small muscles of the hand. (Nov. 79) 11. Describe various gaits which occur in neurologic disorder. Give examples of causative diseases.

(Nov. 80) 12. What are the causes and C.P. of polyneuritis (Nov. 80)

Page 7: ˘ˇˆ˙ - Ain Shams Universitymed.asu.edu.eg/uploads/med/Student Affairs/IEE_IO__2.pdf• Lt. Lower lobe bronchiectesis • Recurrent pul. Embolism • Chronic broncitis (May 96)

Faculty Of Medicine - ASU

Student Union Scientific Committee (SUSC)

Examination Note - 4th year P a g e | 7

13. Give short note on occipital headache (May 83) 14. Discuss the ttt of rheumatic chorea. (May 83) 15. Describe the presenting symptoms of the main 3 types of epilepsy & the expected

electroencephalographic changes in each type. (Nov. 84) 16. Discuss the differential diag. And investigations needed in a case of hemiplagia developing at the

age of 30 years. (Nov. 85) 17. Describe the emergency ttt of a patient with epileptic fit (in details). (May 86) 18. Give the main points to differentiate between upper and lower motor neurone 7th cranial nerve

paralysis. (Dec. 88) 19. How can you proceed to diagnose the site of acute Rt. Sided hemiplegia. What are the steps you

take to reach its aetiology. (May 89) 20. What are the important points that determine drug therapy in epilepsy. (Nov. 92) 21. Enumerate 4 benign intracranial tumors, giving their usual sites.

(Nov. 92) 22. Enumerate the sites of lesions causing vertigo & ch.ch. features of each. (May 93) 23. Enumerate the causes of bilateral facial nerve palsy

• Describe the clinical features suggestive of each cause. • Give one investigation specific for each cause. (Nov. 93)

24. Discuss the ttt of parkinsonism giving the role of each drug used. (May 94)

25. How can you differentiate between organic & hystericl hemiplagia (Dec. 94) 26. Classify the causes of paraplegia developed in less than 3 weeks duration. Mention one inv.

Essential for each cause. (95) 27. In a pt. With Rt. sided hemiplegia, mention the clinical data that make you define its levels as:

cortical, lower pontine level, spinal.

LIVER & G.I.T 1. What are the complications of peptic ulcer (Nov. 75) 2. Enumerate causes of vomiting (Nov. 75) 3. Describe the management of oesophageal varices 4. Describe the C.P. of pyloric obstruction 5. Enumerate causes of diarrhea (Nov. 76) 6. A patient aged 45 ys developed jaundice, how would you arrive the diagnosis. (Nov. 79) 7. Give an account on:

• Causes of upper GIT bleeding • The management of hepatic coma • The causes of prolonged diarrhea (Nov. 79)

8. What are the clinical points which suggest malignancy in gastric ulcer. (Nov. 79) 9. Mention the causes of hepatomegaly. (Nov. 79) 10. Describe the surface anatomy of the liver. How can you arrive at the diagnosis of jaundice at the

bed side of the patient. (Nov. 80) 11. Describe the causes, C.P. and management of upper abdominal pain. (Nov. 81) 12. Describe the anatomy of the spleen. What are the diseases that are accompanied enlarged spleen.

Give short notes on diagnostic features of each. 13. How can you manage a patient with upper GIT bleeding presenting as emergency (Nov. 82) 14. Describe the anatomy of pancreas. Describe the aetiology of lesions in this organ Describe the C.P.,

differential diag. And management of chronic pancreatitis. (May83) 15. Discuss the ttt of chronic duodenal ulcer. (May 93) 16. Give an account on:

Page 8: ˘ˇˆ˙ - Ain Shams Universitymed.asu.edu.eg/uploads/med/Student Affairs/IEE_IO__2.pdf• Lt. Lower lobe bronchiectesis • Recurrent pul. Embolism • Chronic broncitis (May 96)

Faculty Of Medicine - ASU

Student Union Scientific Committee (SUSC)

Examination Note - 4th year P a g e | 8

• Management of ulcerative colitis • Investigations of suspected pancreatic disease • Types and ttt of T.B. peritonitis (Nov. 83)

17. Give 5 causes for big enlargement of the liver and discuss differential diagnosis (Nov. 84) 18. Give a detailed dietary to a patient with duodenal ulcer. (May 85) 19. How would you proceed to investigate a suspected case of ascitis to arrive at definitive diagnosis.

(May 86) 20. Give 6 points to suspect the diagnosis of amoebic hepatitis. 21. Give the most important symptoms and signs of acute pancreatitis. Discuss the results of 3

investigations that confirm diagnosis (Nov. 89) 22. Give the description of abdominal pain caused by uncomplicated duodenal ulcer. Give the dose,

duration of therapy and side effects of a drug theraputically effective in this disease. (Nov. 89) 23. What should you refer a patient with proven stomach ulcer for surgery? (Nov. 90) 24. What are the possible consequences of sever upper GIT hemorrhage in a patient with liver cirrhosis.

(Nov. 90) 25. A patient with liver cirrhosis developed rapid swelling of abdomen:

• Give 3 other signs important to look for giving your reasons • Give 3 investigations you consider important at this stage and state the importance of each.

(Nov. 91) 26. A peptic ulcer patient on treatment with bismuth subcitrate and antacids passed black stools:

• What are the possibilities for this discalouration • How can you differentiate between them

27. Enumerate the factors that ppt hepatic encephalopathy in a pt with chronic liver disease. Explain the mechanisms by which GIT bleeding causes it. (Nov. 92)

28. How can you D.D organic pyloric obstruction & pylorospasm. & Enumerate the factors that may cause acute pancreatitis. (Nov. 92)

29. Describe the different non-surgical therapies for gall stones (Nov. 92) 30. Describe the neuropsychiatric manifestation of hepatienc encephalo-pathy. (May 93) 31. Give the causes of bloody diarrhea. Describe how they can differentiated.

(Nov. 93) 32. What is the risk factors favoring formation of gallstones. How do they act? (Nov. 93) 33. How would you suspect & confirm the diagnosis of cancer head of pancreas.

(Nov. 93) 34. Describe the diff. Mechanisms by which drugs can cause jaundice, giving 2 drugs as a example for

each mechanism. (Nov. 93) 35. How can you DD clinically between gall bladder pain & duodenal ulcer pain?

(May 94) 36. Enumerate the possible clinical manifestations of functional disorders in different parts of the GIT.

(May 94) 37. Discuss the diagnostic value of ascitic fluid analysis. (May 94) 38. Discuss the causes of upper GIT bleeding. Discuss the ttt of amoebic liver abscess. (Dec. 94) 39. State the questions you ask that can help you to reach the possible cause in a pt presenting with

dysphagia, mentioning the reason in each question. (95) 40. What are the causes of enlarged tender liver? What are the clinical examination data only that can

differentiate them from each other. (May 96)

ENDOCRINE 1. What are the complications of D.M. (Nov. 75) 2. Describe the aetiology and C.P. of acromegaly. (Nov. 75) 3. Give the ttt of hyperthyroidism. (Nov. 76) 4. Describe the C.P. of hyperthroidism

Page 9: ˘ˇˆ˙ - Ain Shams Universitymed.asu.edu.eg/uploads/med/Student Affairs/IEE_IO__2.pdf• Lt. Lower lobe bronchiectesis • Recurrent pul. Embolism • Chronic broncitis (May 96)

Faculty Of Medicine - ASU

Student Union Scientific Committee (SUSC)

Examination Note - 4th year P a g e | 9

5. The C.P. of cushing syndrome (Nov. 77) 6. Give an account on ttt of diabetic ketoacidosis 7. Treatment of thyrotoxicosis 8. Causes of hypercalcaemia 9. What are the C.P. of hypothyroidism? (Nov. 79, 80) 10. What are the types of diabetes? (Nov. 79) 11. How would you differentiate between ketoacidemic & hypoglycaemic coma? (Nov. 80) 12. Describe the anatomy, blood supply and histology of the Rt. suprarenal gland. (Nov. 80) 13. Discuss the diagnosis features, aetiology, differential diagnosis and management addison's disease.

(Nov. 81) 14. Write short notes on management of diabetic ketoacidosis. (Nov. 81). 15. Describe the C.P., causes and management of tetany. (Nov. 82) 16. Discuss causes, C.P., investigations and laboratory functions in patient presenting with cushing's

syndrome. How can you proceed to manage such patients? (May 83) 17. Discuss:

• Causes and dangers of hypokalaemia • Use of corticosteroids in endocrinal disease. • Bed side and laboratory investigations done in a case of tetany (Nov. 83)

18. Describe the clinical features of: • Tetany • Addison's disease. (Nov. 84)

19. Give a full comment on ttt of hyperthyroidism. (May 85) 20. Describe details of dietary instructions to a patient with diabetes mellitus. (May 85) 21. Briefly discuss your management in:

• Diabetic ketoecidosis • Thyrotoxic crisis • Tetany • Enumerate 5 endocrinal diseases associated with pigment changes • Give 4 causes of male infertility

22. Give short account on diabetic microvascular complications. (May 89) 23. Mention 5 symptoms of diabetic autonomic neuropathy. (May 89) 24. Enumerate the main clinical manifestations of addison's disease. Give expected results of three

investigations that may be done to confirm the diagnosis. (Nov. 89) 25. Differentiate between diabetic neuropathy and radiculopathy as regard

• Type of pain • Distribution of lesion • Response to ttt

26. Give the possible clinical presentations of a case of adult hypothyroidism. Discuss the investigations that help to reach proper diagnosis. (Nov. 89)

27. Discuss the dietary recommendation in the management of type II diabetes mellitus. (Nov. 90) 28. What are the role of thyroid binding proteins

• Mention main complications of antithyroid drugs. (Nov. 90) 29. How would you suspect the diagnosis of diabetes insipidus? Give its differential diagnosis. (Nov. 90) 30. Describe the pathogenesis of diabetic neuropathies

• What are the clinical manifestations of each? (Nov. 91) 31. Give the clinical presentation of hypothyroidism below the age of 16 years. (Nov. 91) 32. The differential diagnosis of stunted growth. (Nov. 92) 33. Describe the diagnostic features of 2 endocrine conditions causing weight loss. (Nov. 92) 34. Mention 4 causes of hypogonadism. Describe how you would arrive at the diagnosis in each.

(May 93)

Page 10: ˘ˇˆ˙ - Ain Shams Universitymed.asu.edu.eg/uploads/med/Student Affairs/IEE_IO__2.pdf• Lt. Lower lobe bronchiectesis • Recurrent pul. Embolism • Chronic broncitis (May 96)

Faculty Of Medicine - ASU

Student Union Scientific Committee (SUSC)

Examination Note - 4th year P a g e | 10

35. Describe the diagnostic approach to a diabetic pt with coma. (May 93) 36. Discuss the management of diabetic ketoacidosis. (May 94) 37. Mention 5 complications of D.M. & Discuss the management of one of these complications. (Dec

94) 38. Discuss the causes, CI/P & ttt of K depletion (Dec. 94) 39. Classify the causes of stunted growth. Select one cause from each class you stated

mentioning its essential diagnostic features, the main l ine of ttt. (95)

TROPICAL 1. Give a short account an amoebic hepatitis (Nov. 75) 2. Discuss the ttt of acute bacillary dysentry. (Nov. 75) 3. Discuss the C.P., diag., differential diag., and ttt of malaria. (Nov. 75) 4. Give a full account of typhoid fever (Nov. 76) 5. Describe ttt of urinary bilhariziasis. (Nov. 77) 6. Write short notes on signs and symptoms of malaria. (Nov. 77) 7. Give an account on ttt of schistosomiasis. (Nov. 78, II) 8. Give an account on ttt of malaria. (Nov. 78) 9. Discuss the C.P. of glandular fever. (Nov. 78) 10. Describe the ttt of acute amoebic dysentry. (Nov. 79) 11. A young man aged 25 years is suffering from 2 weeks pyrexia, only physical sign was enlarged

spleen. Discuss how would you arrive at the diagnosis. (Nov. 79) 12. How would you treat a case of ankylystomia? (Nov. 79) 13. A patient is suffering from pyrexia with glandular enlargement. How would you arrive at the

diagnosis? (Nov. 80) 14. How can you treat a case of ankylystoma anaemia? (Nov. 80) 15. Discuss the management of brucellosis [Malta fever]. (Nov. 80) 16. Discuss

• Tertian malaria • Cerebrospinal meningitis • Hepatic amaebiasis (Nov. 81)

17. Give short notes on periodic pyrexia. 18. Describe the management of cerebrospinal fever. 19. Give an account on C.P. of infections mononucleosis. (Nov. 83) 20. How would you treat a patient with intestinalschistosomiasis. (May 84) 21. Discuss the causes and differential diagnosis of fever with generalized lymphadenopathy. (Nov. 84) 22. Discuss the management of amaebiasis. (Nov. 84) 23. Give:

• Complications of malignant malaria • Treatment of schitosoma mansoni • Investigations required for a suspecious case of malaria (Dec. 88)

24. Give the ttt of ascariasis 25. Give complications of filariasis. (May 89) 26. Enumerate 3 parasitic infections, each can lead to anaemia

• Describe the blood picture of one these anaemia • Give the dose and side effects of one of drug commonly used in the ttt of its causative parasitic

infection. (Nov. 90) 27. Give the doses of drugs given orally for the ttt of bilhariziasis

• What type(s) of bilharizia each drug acts upon. (Nov. 91) 28. Write short notes on ttt of cerebrospinal fever. (Nov. 77) 29. Explain why:

• Spleen is enlarged in B

Page 11: ˘ˇˆ˙ - Ain Shams Universitymed.asu.edu.eg/uploads/med/Student Affairs/IEE_IO__2.pdf• Lt. Lower lobe bronchiectesis • Recurrent pul. Embolism • Chronic broncitis (May 96)

Faculty Of Medicine - ASU

Student Union Scientific Committee (SUSC)

Examination Note - 4th year P a g e | 11

• Diarrhea occurs in giardiasis. (May 93) 30. Describe the usual course of typhoid fever

• What are the specific drugs that can cure the disease mentioning their doses. (Nov. 93) 31. Discuss the complications of urinary bilhariziasis (Dec. 94) 32. Enumerate the disease that can be caused by unicellular parasitees. Mention the specific drug.

(95) 33. Mention the ttt its dose & duration in the brucellosis (May 96) 34. Mention the definition of acute desentry. Enumerate the causes of such condition, mention the ttt of

only one of them. (May 96) 35. Give in detail the chemoprophylaxis against malaria. Describe the Cl/P in relation to the different

organs affected by plasamodium falciparum. (Nov. 93)

NEPHROLOGY

1. Describe the C.P. of acute nephritis. (Nov. 75) 2. Clinical importance of 10 specific gravity of urine. (Nov. 77) 3. Give short notes on:

• Complications of acute nephritis • Clinical manifestation of chronic renal failure. (Nov. 78)

4. What are the causes of nephrotic syndrome? 5. How can you treat a case of acute pyelitis? (Nov. 79, 83) 6. What information can you gain by naked eye examination of the urine? (Nov. 80) 7. Discuss diagnosis and management of pyelonephritis. (Nov. 81) 8. Describe the cause, management and clinical picture of pyouria.

(Nov. 80) 9. Describe the cause, differential diag., and management of acute renal failure. (Nov. 82) 10. Adult was found to pass 11gm of proteins in the urine per 24 hours. Discuss the causes,

investigations and short notes on ttt. (Nov. 83) 11. Give an account on causes and diagnosis of proteinuria. (May 84) 12. How would examination of urine (excluding the microscopic examination of the urinary sediment)

help you in diag. & follow up of diseases. (May 85) 13. Describe the details of the dietary instructions to patient with nephrotic syndrome. (May 85) 14. How would you suspect that a patient brought to seek you medical advice suffering from renal

failure? (May 86) 15. A patient with chronic renal failure may first consult the gastroentrologist, cardiologist, dermatologist

or haematologist. Give the clinical manifestation that may lead to these consultations. (Nov. 89) 16. Enumerate the clinical manifestation laboratory findings and possible complications of acute

glomuralonephritis. (Nov. 90) 17. Discuss the management of chronic renal failure (Nov. 84) 18. How does anemia occur in Ch.RF. Enumerate the condition which may be essential with reversible

uraemia. (Nov. 93) 19. Enumerate the extra renal manif. Of renal tumours & explain how they occur. (Nov. 93)

Page 12: ˘ˇˆ˙ - Ain Shams Universitymed.asu.edu.eg/uploads/med/Student Affairs/IEE_IO__2.pdf• Lt. Lower lobe bronchiectesis • Recurrent pul. Embolism • Chronic broncitis (May 96)

Faculty Of Medicine - ASU

Student Union Scientific Committee (SUSC)

Examination Note - 4th year P a g e | 12

20. Describe the suitable diet for • Hypercholesterolaemia & give your reasons in each • Ch renal insufficiency. (May 94)

21. Enumerate 4 causes of polyurea • Explain the mech. of its production in each (May 94)

22. How would you suspect & then confirm that a pt. is suffering from rapidly progressive GN? (May 94) 23. What is the definition of nephritis $, enumerate 10 causes of this $ (Dec. 94) 24. Enumerate various types of nephritis, mentioning the commonest cause & the most imp. Line of ttt

for each type. (95) 25. Enumerate only, all causes of suprarenal insufficiency (96) 26. Enumerate 5 of the most reliable parameters with which to determine the prognosis in lupus

nephritis. Describe the significance of each. (May 93)

RHUMATOLOGY

1. Discuss the ttt of rhumatoid arthritis. (Nov. 75) 2. Describe ttt of gout (Nov. 77) 3. Enumerate causes of transient arthritis. (Nov. 79) 4. Management of acute gouty artharitis (Nov. 82) 5. Describe the various types of arthritis. How can you diagnose and manage a patient with arhumatoid

arthritis. (May 83) 6. Drugs used in rhumatoid arthritis [name of drug, dose and side effects]. (Nov. 83) 7. Give a full account on the ttt of rhumatoid arthritis. (May 86) 8. Give the details of the emergency ttt of acute gouty arthritis. (May 86) 9. Give an account on the clinical types of a rhumatoid arthritis. Give investigations and ttt. (May 89) 10. Skin manifestation of systemic lupus erythromatosis. (May 89) 11. Discuss how can you clinically differentiate between rhumatic and rhumatoid arthritis. (Nov.

89) 12. Discuss the dietary recommendation in the management of hyperuricaemia. (Nov. 90) 13. Give the possible serologic and haematologic abnormalities in S.L.E. (Nov. 90) 14. Which organ can be involved in the extras-articular manifestations of rhumatoid arthritis. Name 2

manifestations in each organ. (93) 15. Describe the hand deformities in rhumatoid arthritis. Describe the CI/pof SLE & Mention the

prognosis (May 94) 16. How could you diagnose polyarteritis nodosa & ttt (Dec. 94) 17. Enlist the causes of polyarthrritis. Mention the lab tests be done for diagnosis, stating whether each

of the tests you mention is specific, equivocal or non-specific. (95)

PSYCHIATRY

1. Give an account on the types of schizophrenia. (Nov. 77, 79) 2. Discuss the different types of depression and their management.

(Nov. 80) 3. What are the physical signs and symptoms of anxiety neurosis and its differential diagnosis? (Nov.

81) 4. The ttt of manic-depressive psychosis. (Nov. 82) 5. Describe the C.P. of anxiety neurosis. Enumerate other medical disease giving a similar clinical

picture. (Nov. 83) 6. Write short notes on types of hysterical conversion reaction. (Nov. 84) 7. Give the details of emergency treatment of a patient with acute mania [excitement] (May 85)

Page 13: ˘ˇˆ˙ - Ain Shams Universitymed.asu.edu.eg/uploads/med/Student Affairs/IEE_IO__2.pdf• Lt. Lower lobe bronchiectesis • Recurrent pul. Embolism • Chronic broncitis (May 96)

Faculty Of Medicine - ASU

Student Union Scientific Committee (SUSC)

Examination Note - 4th year P a g e | 13

8. How would you suspect that a patient brought to seek your medical advice suffering from schizophrenia? (May 86)

9. Discuss causes and C.P. of acute mania. (May 90) 10. Enumerate the clinical presentations of anxiety disorder and the differential diagnosis of medical

conditions presenting as anxiety. 11. Describe the symptoms and management of severe endogenous depression in a female aged 45

years. (Nov. 91) 12. Describe the clinical presentations of anxiety disorder. (Nov. 92) 13. Describe the management of the main psychatric emergency.

(Nov. 92)

14. Discuss the D.D. signs and symptoms between: • Pheochromocytoma and anxiety neurosis (anxiety disorder) • Hysterical and general mal fits • Withdrawal symptoms of heroine and barbiturate abuse. (May 93, 94)

15. Describe the known types of personality disorders and management (Nov. 92) 16. Discuss the differential clinical symptoms, signs between:

• Hysterical & hypoglycemic coma. • Mitral valve prolapse & anxiety • Panic attack & myocardial infraction (Nov. 93)

17. A patient gets paroxysmal auditory hallucinations, disturbed and aggressive behavior & then amnesia to attack. Discuss the DD when it's paroxysmal & when it's permanent (May 94)

18. Mention, shortly the various clinical presentations considered as psychiatric emergencies. Mention shortly the immediate management in each presentation. (95)

19. Describe the management of suicidal attempt in a medical ward. (96)

INFECTIONS

1. Discuss the treatment of: • Acute follicular tonsilitis • Acute bacillary dysentry (Nov. 75)

2. Write short notes on the rash in small pox 3. Complications in scarlet fever 4. Discuss the differential diagnosis of acute throat lesions with short notes on their management.

(Nov. 82) 5. Enumerate 3 viral infections that may follow a blood transfusion. Discuss the laboratory diagnosis of

two of them. (Nov. 89) 6. How would you proceed to arrive at the definitive diagnosis of suspected case of meningitis. (Nov.

90) 7. Describe the CI/p & complications of measles. (94)

IMMUNOLOGY & THERAPEUTICS 1. Give a short account on urticaria (Nov. 75) 2. Treatment of acute anaphylaxis 3. A patient developed acute anaphylactic reaction after penicillin injection. How would you can treat

him. (Nov. 90) 4. Give an account on anaphylactic shock. (Nov. 83, II) 5. Describe the details of emergency ttt of a patient with anaphylactic shock. (May 85) 6. What is emergency ttt of acute penicillin anaphylactic shock. (May 89) 7. Give briefly the mechanism of immediate hypersensitivity reaction (type I reaction). Give 2 examples.

(Nov. 89)

Page 14: ˘ˇˆ˙ - Ain Shams Universitymed.asu.edu.eg/uploads/med/Student Affairs/IEE_IO__2.pdf• Lt. Lower lobe bronchiectesis • Recurrent pul. Embolism • Chronic broncitis (May 96)

Faculty Of Medicine - ASU

Student Union Scientific Committee (SUSC)

Examination Note - 4th year P a g e | 14

8. Discuss the pathogenesis of decreased resistance to infection, the prophylactic measures for their avoidance. (Nov. 90)

9. How would you suspect then confirm the diagnosis of acquired immune deficiency syndrome (AIDS). (Nov. 91)

10. Discuss the management of barbiturate poisoning (Nov. 81, May 83) 11. Discuss indications and complications of diuretics. (May 90) 12. Give 3 reasons against empirical prescription of antibiotics in pyrexia. 13. Enumerate 3 vitamins whose over dose can cause side effects. Mention the side effects of

hypervitaminosis of one of them. (Nov. 89) 14. Mention the various drugs used by the inhalation route in therapy for bronchial asthma and their

mechanisms of action. (Nov. 90) 15. Describe the adverse drug reactions of:

• Asprine • Digoxin (Nov. 91)

16. Give the indecations for ttt with and the dose of one: • Beta-blocker • H-receptor blocker (Nov. 92)

17. What are the hazards of the following & how do they presents: • Fe overload • Excess serum Cu. (Nov. 92)

18. What are the lines of defense present in the body against infection. Describe on clinical situation as an example for the defensive function of each of them.

19. Write short essay on: • Immunopotentiating drugs • Desensitization therapy • IV-Gamma globulin therapy (Dec. 90)

NUTRITION 1. Describe the manifestation of vit.B complex deficiency. (Nov. 76) 2. Describe the manifestations of pellagra. (Nov. 77, 78, 79, 80) 3. Describe the clinical manifestations of:

• Hypovitaminosis A • Hypovitaminosis D (Nov. 91)

4. What are the causes of generalized oedema. Give at least one diagnostic feature of each. (Nov. 92) 5. Describe the eye changes seen in:

• Vit. A deficiency • Vit. B deficiency (May 93)

BLOOD 1. Describe the C.P. of chronic myeloid leukaemia. (Nov. 75) 2. What is blood picture of pernecious anaemia? (Nov. 75) 3. Discuss the causes of haemolytic anaemias. Discuss the ttt. (Nov 76, 78) 4. Discuss the ttt of acute leukaemia. (Nov. 78) 5. Describe the C.P. of hodgkin's disease (Nov. 79) 6. What is blood picture of chronic myloid leukaemia? (Nov. 79) 7. Give an account on the C.P., investigations and ttt of iron deficiency anaemia. (Nov. 83) 8. Give an account on haemolytic anaemias. (Nov. 84) 9. Give an account on leucopenia (May 89) 10. How would a full blood picture help to differentiate the causes of anaemia?

Page 15: ˘ˇˆ˙ - Ain Shams Universitymed.asu.edu.eg/uploads/med/Student Affairs/IEE_IO__2.pdf• Lt. Lower lobe bronchiectesis • Recurrent pul. Embolism • Chronic broncitis (May 96)

Faculty Of Medicine - ASU

Student Union Scientific Committee (SUSC)

Examination Note - 4th year P a g e | 15

11. Discuss the importance of different shapes of the RBCs. What are the role of the MQ in the immune defense system? (Nov. 92)

12. Describe the CI/p and management of incompatible blood transfusion. (May 93) 13. Enumerate the cause of agranulocytosis & give its CI/p. (May 93) 14. How would you suspect & then confirm the diagnosis of acute leukemia (Nov. 93) 15. Describe the ttt of Hodgkin's disease based on its clinical staging (May 94) 16. Discuss the management of ch.lymphocytic leukemia (Dec. 94) 17. Write short essay on ttt of non-thrombocytopenic purpura (Dec. 94) 18. Mention the ttt, its dose and duration of the pernicious anaemia.

� �� �� �� �

� �� �� �� �

� �� �� �� �

� �� �� �� �

� �� �� �� �

يص لمحاضرة أقوى الكبسو�ت لمتحاناتتلخ

ندوة أعدتھا أسرة الفرقان

شيماء أسامة/ د : بواسطة

--------- من امجل االشياء ىف ندوة االختالف ىف طرق التفكيري الكل عرض ما ميكن ان يفيد غريه و انت ختتار اللى يناسبك -------------

د السكرى: اوال

************* لى حاجتنياكد ع

Page 16: ˘ˇˆ˙ - Ain Shams Universitymed.asu.edu.eg/uploads/med/Student Affairs/IEE_IO__2.pdf• Lt. Lower lobe bronchiectesis • Recurrent pul. Embolism • Chronic broncitis (May 96)

Faculty Of Medicine - ASU

Student Union Scientific Committee (SUSC)

Examination Note - 4th year P a g e | 16

التكرار مهم جدا -1مهما كان اللى فاتك املهم ابدأ دلوقىت و كفاية نزيف: اوقف الرتيف -2

------------------------

د كفراوى: ثانيا

*********** الزم حتط هدف من دراستك للطب و الزم حتب اللى انت بتعمله -1 ذاكر خبط ايدك عمرك ما هتنسى -2اجلماعية غري مفيدةاملذاكرة -3

مش مشكلة لو سرحت و انت بتذاكر بس متطولش ىف السرحان عشان متقفدش تركيزك -4 متفتحش النت اال و انت حمتاج حاجة مش ع الفاضية و املليانه النك هتضيع بيه وقت من غري ما حتس -5 متنساش حافز انك بتعمل دا الرضاء والديك اللى اكرب هدية ليهم جناحك -6 ---------------------

د حممد جمدى معيد هستولوجى: ثالثا

******** مرحلة ما قبل االمتحانات*_*_*_*_*_ اياك تغري مصدر مذاكرتك اوعى اوعى اوعى -1 حاول ختلى مرة او اتنني من مذكراتك لنفس الشابتر تبدا من قدام لورا عشان املواضيع اللى دميا بتقع ىف االخر -2ناوين االولذاكر ع -3

حط راحة يومية و اسبوعية الزم -4وقت الراحة ارتاح و انسى كل حاجة و وقت املذاكرة ركز فيها فقط...ركز -5

مرحلة االمتحانات*_*_*_*_*_ قبل االمتحان صلي ركعتني قضاء حاجة -1اهم حاجة دعائهم...اوعى تزعل والديك -2

حىت لو غشيت انت مش عارف مني ممكن يصححلك ...االمر كله بايد ربنا افتكر ...اوعى اوعى اوعى اوعى تغش -3 و هينجحك و ال ال اقرا االسئلة كويس جدا ىف مخس دقايق دى فيهم -4 مرحلة ما بعد االمتحان*_*_*_*_*_ متراجعش -1النص يوم دا اجازة"متذاكرش -2

ارضى بقضاء ربنا -3 تذكر*_*_*_*_*_ بدا بعلمك او درجتكاياك تتغر ا -1

تذكر امسى اهللا احلكيم و العدل -2 انت عاوز امتياز عاوز تعيني بس هل دا اخلري ؟ ربنا له حكمة قد ال تعلمها

Page 17: ˘ˇˆ˙ - Ain Shams Universitymed.asu.edu.eg/uploads/med/Student Affairs/IEE_IO__2.pdf• Lt. Lower lobe bronchiectesis • Recurrent pul. Embolism • Chronic broncitis (May 96)

Faculty Of Medicine - ASU

Student Union Scientific Committee (SUSC)

Examination Note - 4th year P a g e | 17

بس امىت النصيب دا مش بتاعتك...لكل جمتهد النصيب

--------------------------

٦حممد موسى سنة : رابعا

**************

الكرب و البخل...اياك و افات العلم ...تذكر ابدا

ظبط اى مصدر املهم تثبته...تذاكر منني

جرب كذا طريقة و اختار االنسب...تذاكر ازاى

افضل تدريب هو اخلشوع ىف الصالة...اركز ازاى

خر لسنة اكتب دميا كل اللى تذاكره حبيث ا+ حط جدول للشهر على بعضه من اول السنة ...استراجيية املذاكرة تكون عارف ترتب اولوياتك تبدأ باحلاجة اللى اتذاكرك اقل عدد مرات سابقا

ال تنسى االستخارة و االستشارة+ اهلدف لينا واحد الفرق الغاية و الطريقة ...روحانيات االمتحان

psychological reilief ...التوكل على اهللا باالخد بالسباب و الثقة بالل خري التجارة مع اهللا...ن صفقات خسارة االوائل ال يعقدو

----------------------------- حممد احلميلي

**************** no one size fits all every 1h planning saves 8 h of waste later لعمل اجلدول

يوم قبل االمتحان االول ١١اوال فترة بني اخر يوم ىف الراوند و خلد

*_*_*_*_*_*_*_*_*_*_ (: صفى ذهنك متاما و خلص كل مشاكلك النفسية و العاطفية قبل دخول الفترة دى -

خد اجازة يومني قب ما تبدأ - خد قرارك هتأجل و ال ال قبل ما تبدأ الفترة دى - جدد نيتك تتعلم باتقان -

ممكن تضيع يوم كامل فيه متستخرش: حط اجلدول

رتب شابترز املادة -1a حاجة اتذاكرت كويس و اتراجعتاقل . b . cاكتر حاجة مذاكرها حلو قبل كدا و مراجعها ايام ىف االسبوع و يوم ريزرف ٦اعمل اجلدول على -2

اححسب عدد صفح املادة على عدد االيام هيطلع كام صفحة تتذاكر ىف اليوم -3 يفضل متشي مادتني سوا منعا للملل -4ى تقدر تعمله عشان متتزنقشحط عدد صفح اقل ما الل -5

Page 18: ˘ˇˆ˙ - Ain Shams Universitymed.asu.edu.eg/uploads/med/Student Affairs/IEE_IO__2.pdf• Lt. Lower lobe bronchiectesis • Recurrent pul. Embolism • Chronic broncitis (May 96)

Faculty Of Medicine - ASU

Student Union Scientific Committee (SUSC)

Examination Note - 4th year P a g e | 18

يوم ١١ال

*_*_*_*_*_*_*_ dont change ur source accept u cant dont in best way مش مهم خلصت كام مرة املهم انك متعود دميا انك ممكن ختلص مرتني و بتعرف حتل كويس و تظبط بيهم و جتيب تقدير ---------------- حممود نبيل

*********** نا مش زى غريي متضغطش نفسك عصبياا -1 مذاكرة االصحاب زى املورفني بتريح بس ملهاش فايدة جبد -2 حط اهاداف صغرية لكل مرة مذاكرة -3 اخر يوم قبل االمتحان الزم عيىن تشوف كل ورقة ىف املنهج -4 (: اضغط اجلدول و حط اكتر كل يوم -5حاجة و بني اللعبىف فرق بني الراحة انك تقعد ترتاح متعملش -6

خالد عفيفي********** fatal mistakes بالش -1 شكل و صياغة االجابة -2 وضع االهم فاملهم املميزة bonusزود النقط ال -3 لو ىف الشفوى الدكتور حس انك دكتور جبد هيقدركك -4

Page 19: ˘ˇˆ˙ - Ain Shams Universitymed.asu.edu.eg/uploads/med/Student Affairs/IEE_IO__2.pdf• Lt. Lower lobe bronchiectesis • Recurrent pul. Embolism • Chronic broncitis (May 96)

Faculty Of Medicine - ASU

Student Union Scientific Committee (SUSC)

Examination Note - 4th year P a g e | 19

CASESCASESCASESCASES

Case 1

A young man was admitted to hospital on account of collapse after passing black soft stools. His

pulse was 110/ min regular blood pressure (90/60 mm Hg).

a) What are the points in history and clinical examination will help you reach the diagnosis? b) Inspite of treatment the patient developed bouts of irritability and coma. How would this

development affect management. (May 81)

Case 2

A 23 years old man was admitted in shock after vomiting a large amount of dark brown vomitus. He was transfused with three units of whole blood. Next morning he was found confused and irritable;

his BP was 110/70.

a) What are the points in history of this patient would you like to enquire about? b) What signs would help you to arrive to a diagnosis? Explain how? c) How would you manage this patient? (May 84)

Case 3

A 17 years old girl suffered from 3 attacks of fever and sore throat from January till April 1984. She used to respond to chloramphinicol capsules 250mg q.i.d. with a few days which was prescribed by her doctor in the first attack and as taken by herself in subsequent attacks on 14 April she had fever 38.3°C and sore throat inspite of self medication with chloramphinicol in the same dose. She did not

respond and you were called to see her on the seventh day.

a) What are particular signs would you like to look for in the clinical diagnosis of the girl? Explain the value of each.

b) Select the most important three tests which should be ordered now. c) Comment on the doctor's prescription on the first occasion and the patients self medication on the

subsequent attacks. (May 84)

Case 4

A 33 years old man presented with severe headache for the previous 12 hours. He was unable to bend his neck without marked pain and he vomiting four times since the start the headache.

Temp. 37.8, BP 170/90 and he was a bit confused.

a) What are particular signs would you like to elicit during you examination? b) Choose one essential bed side examination that should be done and explain the significance of the

different findings obtained and how would these findings determine your future line of treatment? c) To which particular consultant would you like to refer this patient? Indicate reasons for consultation.

(May 84)

Case 5

A 6 years old girl complained of a pain in her left shoulder and right knee for 10 days she was febrile. She presented to you a laboratory result of ESR of 100 mm.

a) What particular points in the history should you enquire about? And what should you specifically look for in the physical examination?

Page 20: ˘ˇˆ˙ - Ain Shams Universitymed.asu.edu.eg/uploads/med/Student Affairs/IEE_IO__2.pdf• Lt. Lower lobe bronchiectesis • Recurrent pul. Embolism • Chronic broncitis (May 96)

Faculty Of Medicine - ASU

Student Union Scientific Committee (SUSC)

Examination Note - 4th year P a g e | 20

b) Select the most important four additional laboratory tests that should help you in follow up. c) Comment on the immediate and remote prognosis.

Case 6

A 54 years old man presented to the reception room of a general hospital because severe pain across the chest that lasted for 40 min. clinical examination revealed no abnormalities. An electrocardiogram

was also normal.

a) What advise would you give to the patient? b) What particular signs would you look for if you were asked to follow up? Explain the significance of

each. c) Select the most relevant 3 investigations that you would like to order giving reasons. (May 85)

Case 7

A 12 years old girl presented after a fit. On examination she was puffy BP170/100 temp. 37.5, examination of the chest, heart, abdomen and nervous system were unremarkable. Her mother gave

a history of fever and sore throat 10 days previously for which the girl received 3 injections of penicillin.

a) What is the most important bed-side test you should do? Explain the significance of the findings. b) Explain the line of treatment you should advise. c) What particular complications would you be on the look for? Describe their early diagnosis and your

response to their development. (May 85)

Page 21: ˘ˇˆ˙ - Ain Shams Universitymed.asu.edu.eg/uploads/med/Student Affairs/IEE_IO__2.pdf• Lt. Lower lobe bronchiectesis • Recurrent pul. Embolism • Chronic broncitis (May 96)

Faculty Of Medicine - ASU

Student Union Scientific Committee (SUSC)

Examination Note - 4th year P a g e | 21

Case 8

Five days after an operation for fracture neck of femur, a 73 years old man developed fever and shivering. A week later he started to cough and brought up haemorrhagic sputum. He had sharp pain

in the lower right side of the chest and right upper abdomen.

a) What are points in the clinical examination would you specifically look for, and why? b) How would you manage the patient? c) What is special advise would you give him in order to avoid recurrence if he is to have a further

operation?

Case 9

A 50 years old diabetic patient controlled on 25 mg/day chlorpropamid for the last 3 years, noticed that he started to loose weight, to pass urine 3-4 times by night and to get thirsty over the last 10

days. On examination he was found febrile 37.3°C.

a) What points in the physical examination he was found febrile 37.3°C. b) Choose 4 relevant procedures that might be of help to explain his pyrexia. Explain how. c) What changes and additions in the line of treatment would you advise to control his recent

complaints?

Case 10

A 12 years old boy presented on account of fever and mild jaundice for 5 days. He has history of sore throat but no anorexia or nausea in the few days that preceded his complaint. O/E the patient looked

well. Temp. 38 and the lymph nodes were enlarged in both sides of the neck and both axillae.

a) What additional signs would you look for? Explain their significance. b) Select the minimum number of test that would let you reach a definite diagnosis. c) What answer would you give to the parents if you were asked about the prognosis.

(May 86)

Case 11

You were called to see a 36 years old hypertensive woman on irregular hypertensive therapy who developed sudden inability to move her left upper and lower limbs. O/E you found the BP 130/100 and she stated that she did not receive her antihypertensive treatment for several weeks. He pulse showed an irregular irregularity. 5 days later she complained of severe pain in the right leg, the foot

became swollen pale and quite cold. 4 days later she developed diarrhea and passed about 10 motions of soft, climy, black stools.

a) What particular signs would you look for in examination of the nervous and cardiovascular systems when yot, first see her? Explain their significance.

b) What particular bed side investigation would order? Why? c) What is the possible explanation of the features, which developed one after the other. What treatment would you advise? How would you monitor your treatment? (May 86)

Case 12

A 55 years old man, smoking for 30 years and having "smokers cough" developed acute chest

infection last September and another one last December. After the second episode cough changed in

character and the sputum started to become blood tinged. He presented to you on account or

Page 22: ˘ˇˆ˙ - Ain Shams Universitymed.asu.edu.eg/uploads/med/Student Affairs/IEE_IO__2.pdf• Lt. Lower lobe bronchiectesis • Recurrent pul. Embolism • Chronic broncitis (May 96)

Faculty Of Medicine - ASU

Student Union Scientific Committee (SUSC)

Examination Note - 4th year P a g e | 22

progressive weakness and tingling and numbness in hand and feet which were progressing over the

previous 8 weeks.

a) What particular points in the general examination and examination of the various system should you look for? Why?

b) Select 3 important investigations that would help you to reach a definitive diagnosis and might help your choice of the line of management. Explain how. c) What particular consultant might be needed for further management?

d) What additional investigations would you prepare before calling this patient? (May86) Case 13

An obese lady 65 years old presented with left sided abdominal pain & bleeding per rectum. She gave a history of more than 10 years of constipation with chronic use of laxatives.

a) What is the most likely diagnosis? Why?

b) What is the differential diagnosis?

c) How can you prove the diagnosis? (Dec 88) Case 14

A lady 58 years of age presented with jaundice, for 1-week duration she has been complaining of flatulent dyspepsia over the last two years occasional abdominal pain. She had marked anorexia for

the last 2-weeks

a) What are the symptoms & signs are care to look for in this patient?

b) What are the investigations required proving this diagnosis?

Case 15

A 55 years old man has been diabetic for the last 20 years. He developed ankle edema. His BP was 180/100.

Last month he got an attack of pain in the right loin with fever 38.5 & dysuria. A doctor examined him & prescribed tetracycline & analgesic and he asked for intravenous pyleography and urine culture

& sensitivity. a) Comment on the mistake of the doctor.

b) What would you do if you were faced with such a case? The IVP showed a stone in the lower pole of both kidneys. The urine culture showed Gram's negative bacilli highly sensitive to

gentamycine. So the doctor stopped the tetracycline & prescribed gentamycine IM in a dose of 80 mg / 8 hours. Over the next 5 days the general condition of the patient deteriorated markedly and lie

became semicomatosed with marked air hunger and severe oliguria. His blood urea was 420 mg and serum creatinine 18 mg/100 ml.

c) Comment again on the present situation and try to explain this deterioration.

d) How would you manage this patient now. (Dec. 88) Case 16

Female patient complaining of amenorrhea for 2 years following the last difficult labor complicated by severe postpartum from that time she was told that she is hypotensive. She has a

well marked asthenia, and cannot tolerate cold weather.

a) What are other symptoms to be asked for?

b) What is the diagnosis?

c) What are the necessary investigations to prove the diagnosis?

d) What are the main lines of treatment?

Page 23: ˘ˇˆ˙ - Ain Shams Universitymed.asu.edu.eg/uploads/med/Student Affairs/IEE_IO__2.pdf• Lt. Lower lobe bronchiectesis • Recurrent pul. Embolism • Chronic broncitis (May 96)

Faculty Of Medicine - ASU

Student Union Scientific Committee (SUSC)

Examination Note - 4th year P a g e | 23

e) What is the prognosis? (Dec 88)

Case 17

A young girl aged 6 years presented with pyrexia, rigors, headache, vomiting, and left loin pain. She had a history of two similar attacks in the last few months.

a) What is the only important investigation you recommend in such circumstances? b) What is the probable cause of fever in this case? c) What further investigations you advise to reach the final diagnosis? (May 89)

Case 18

A male patient aged 35 years presented with asthenia. He has been under therapy for his low blood pressure for sometimes. His dermatologist had a long run of therapy for discoloration on his trunk

and for the deep color of skin.

a) What do you expect to find in his history and on full clinical examination?

b) What are the steps needed for the diagnosis?

c) What do you suggest for treatment? (May 89) Case 19

A male patient 45 years old presented to the reception room in hospital delerium. His wife told you that he caught influenza and chest infection three days ago. She also told you that in the last few months he was irritable and lost concern, to the family. On examination you found that lie had

tremors. The liver was left 10 cm below costal margin.

a) What is the diagnosis?

b) What is the ppt factor for present condition?

c) Which types of tremors did the patient have?

d) Mention other 5 causes of tremors you know?

e) What are the essential lines of treatment of the patient's emergency condition?

f) Can you give him morphia to sedate? (May 89)

Case 20

A 25 years old man suffered from cough, hemoptysis & night fever. X-ray of chest showed a cavity in the right upper lobe.

a) What further investigations would you ask for?

b) Assuming your suspected diagnosis is correct, what would be the line of treatment?

The patient received (lie proper treatment and his condition improved markedly, but he stopped the treatment after 4 months. Few months later he received irregular course of therapy upon recurrence of symptoms. 10 years later he developed marked general weakness, ankle edema, puffy eyelids &

proteinuria 6 gm/lit.

c) What investigations are needed now? d) What is the most probable diagnosis? e) Comment on the prognosis? (May 89)

Case 21

Page 24: ˘ˇˆ˙ - Ain Shams Universitymed.asu.edu.eg/uploads/med/Student Affairs/IEE_IO__2.pdf• Lt. Lower lobe bronchiectesis • Recurrent pul. Embolism • Chronic broncitis (May 96)

Faculty Of Medicine - ASU

Student Union Scientific Committee (SUSC)

Examination Note - 4th year P a g e | 24

A 14 years old girl presented by rapidly developed asthenia, pallor sore throat & pyrexia. On

examination she looked pale, had blood spots on her arms, few throat ulcers and systolic murmur on the heart.

One week later she suddenly passed into coma and right-sided paralysis of the upper limb and the lower limb extensor planter response and died three days later.

a) What is the most possible diagnosis of the girl's original condition?

b) Give two investigations that will help to establish the diagnosis.

c) What is the complication that occurred in the last week? (May 89) Case 22

A 20 years old girl presented with ankle edema and puffiness of the eyelids, fever 37.8, and joint pain. Urine analysis showed a proteinurea of 5.5 gm/l. she gave the history of an erythematus rash

on the cheeks and nose.

a) What is your provisional diagnosis?

b) State the appropriate investigations, and their expected results?

The condition of the pt. Markedly improved by proper treatment, and the dose of the main drug was reduced gradually to a maintenance smaller dose. 2 years later the pt developed severe stitching pain

in the right side of the chest. Examination revealed dullness on the right side of the chest & a rub heard at its upper level.

c) Give an explanation for this data?

d) What are the investigations needed now?

Six years later the patient was admitted to a hospital in a semicomatosed state with marked air hunger and blood pressure of 180/120 mm Hg. Her relatives stated that she suffered from marked

anorexia, vomiting and general weakness during the last month.

e) What is your diagnosis in this stage?

f) What are the urgent investigations to be done?

g) Outline the necessary treatment at this stage? (Nov. 89) Case 23

A male patient aged 22 years, presented with sudden left sided hemiplegia and left Jacksonian fits, cardiovascular examination showed evidence of mitral stenosis with atrial fibrilation. Two months

later the patient developed hematuria and plain radiology of the urinary tract showed no abnormality.

a) What further symptoms and signs will help you to define the site of the brain lesion? b) What can you explain the attack of hematuria, on me background brain lesion. c) Discuss factors in favor of anticoagulent therapy versus contraindications in this patient. (Nov. 89)

Case 24

A 15 years old developed fever with cough and chest discomfort. He suddenly became severely dyspnic. Examination showed engorged neck veins with inspiratory filling, pulse of 120/ min with a

paradoxical character, BP 80/50 mm Hg and distant heart sounds.

a) What is the most likely diagnosis?

b) Give 2 investigations to prove the diagnosis

Page 25: ˘ˇˆ˙ - Ain Shams Universitymed.asu.edu.eg/uploads/med/Student Affairs/IEE_IO__2.pdf• Lt. Lower lobe bronchiectesis • Recurrent pul. Embolism • Chronic broncitis (May 96)

Faculty Of Medicine - ASU

Student Union Scientific Committee (SUSC)

Examination Note - 4th year P a g e | 25

c) Give 2 possible aetiological conditions

d) What emergency management do you suggest? (Nov. 89)

Case 25

A young 12 years old girl presented with pyrexia of 38, malaise anorexia and hypochondrial pain. Examination revealed mild jaundice and enlarger tender liver. Blood picture showed eosinophilia of 20

%.

a) Give me most likely parasitic infection giving this picture.

b) Give any further investigations you would like to do.

c) What is the best drug for treatment? (Nov. 89)

Case 26

A man aged 52 years was brought to hospital within one hour of having severe chest pain. After

doing an ECG, he was admitted to the coronary care unit.

A blood sample was sent to the laboratory and proper treatment was started. Two hours later the patient felt sudden severe palpitations. Few minutes later lie became severely dyspnic and cyanosed

with audible chest bubbling.

a) Describe the possible ECG findings that urged admission to the ICU.

b) Choose 3 investigations to be ordered in the laboratory request sent with the blood sample, in order

to prove the diagnosis and give their possible results.

c) Outline the schedule of treatment that he received in the ICU.

d) What possibly happened two hours later & how would you manage? (Nov. 90)

Case 27

A male patient aged 65 years presented with gradually increasing difficulty in signing his name. On ex.: no weakness, reflex changes or sensory abnormalities were recorded. His blood pressure was

100/60mmHg. And his pulse showed multiple extrasytoles.

a) Would you expect any change of muscle tone? If so which tone abnormality would you expect

mostly?

b) What further signs would you expect to detect in this patient during the next year if he is not treated?

c) During the management of this case, is percussion of lower part of the abdomen important, and why?

(Nov. 90)

Case 28

In 1980, a 38 years old lady had an attack of severe epistaxis for which she was transfused by 2 units of blood. Four months later, she developed jaundice. In 1985, her abdomen became distended with

dull flanks. A thiazide diuretic was given but response was not satisfactory. 1990, a routine abdominal sonogram showed a round area 3*3 cm of hypoechogenisity in the right lobe of the liver.

a) Give 3 medical conditions that can lead to severe epistaxis.

b) Give 4 factors that have probably results in the unsatisfactory response to the thiazides.

c) What is the most probable cause of jaundice in this case and how can you prove it?

d) How can you explain the sonographic finding in relation to the patient's history? (Nov. 90)

Page 26: ˘ˇˆ˙ - Ain Shams Universitymed.asu.edu.eg/uploads/med/Student Affairs/IEE_IO__2.pdf• Lt. Lower lobe bronchiectesis • Recurrent pul. Embolism • Chronic broncitis (May 96)

Faculty Of Medicine - ASU

Student Union Scientific Committee (SUSC)

Examination Note - 4th year P a g e | 26

Case 29

A 60 years old man with chronic cough that he attributes to heavy smoking for many years

started to get frank haemoptysis.

a) How can sputum analysis and chest radiology suggest the most important causes of this recent

haemoptysis.

b) Mention another additional investigations necessary to reach the proper diagnosis. (Nov. 90)

Case 30

A 24 years old lady presented with headache, blurring of vision and right

loin pain. Examination revealed a blood pressure of 240/150mmHg, and a systolic bruit over right

quadrant of the abdomen.

a) What is the probable diagnosis?

b) Discuss the investigations to be done.

c) What is the suitable treatment? (Nov. 90)

Case 31

A 56 years old male was admitted to the CCU because of severe agonising chest pain that lasted

for 30 minutes BP was 180/100, pulse 100/mm with infrequent ectopics.

a) Mention 3 diagnostic possibilities

b) How can you differentiate between them?

c) What treatment would each condition need? (Nov. 91) Case 32

A 40 years old woman suffered from arthraglia affecting the small joints for the last 15 years received various drugs for treatment of her disease. Last month she developed generalized oedema and heavy

proteinuria.

a) Discuss the possible diagnostic diagnosis

b) What investigations are required to reach the diagnosis?

c) Describe the management according to the diagnosis. (Nov. 91) Case 33

A married female aged 20 years developed irregular involuntary movements of the left upper limb. The proximal muscle was more affected than the distal. Her

husband reported that she became irritable and dropped objects from her hand.

a) What would you expect to find on neurological examination? b) What further clinical manifestations would you. like to search for to help you define the type and

cause of this complaint? c) What treatment would you give her? d) What is the prognosis in this case? (Nov. 91, 92)

Page 27: ˘ˇˆ˙ - Ain Shams Universitymed.asu.edu.eg/uploads/med/Student Affairs/IEE_IO__2.pdf• Lt. Lower lobe bronchiectesis • Recurrent pul. Embolism • Chronic broncitis (May 96)

Faculty Of Medicine - ASU

Student Union Scientific Committee (SUSC)

Examination Note - 4th year P a g e | 27

خواطر كريم كامل

!من حوايل سنة كنت عيل املتورسيكل والسكة زحمة و الدنيا حر مولعة !ع*لة بتمىل منه براحتها لقيت كولدير يف الشارع والناس

!واحد جنبي القيته جايب ازازة كب1ة.. كان معايا كباية فملتها جركن بأه! قعد :يل براحته! بعدين جه واحد ظريف معاه جركن

!كل* افاض هللا عليك بالتوفيق.. كل* كربت! بتتفاوت.. سبحان هللا الهمم كده برده ! ازازة وال جركن؟كباية وال.. الفكرة بس احنا معانا ايه

!و عىل قدر االستعداد يكون االمداد

العرس يرسا لتفائلت واستبرشت ) بعد(لو قيل يل من أحدهم إن "إن مع العرس يرسا! "؟) مع(فكيف وقد قال سبحانه وتعاىل Yبل كيف وإن تكررت اآلية مرت! العرس يرسا لتفائلت واستبرشت ) بعد(لو قيل يل من أحدهم إن .وهللا لن نعدم من رب يضحك خ1ا "إن مع العرس يرسا! "؟) مع(فكيف وقد قال سبحانه وتعاىل Yبل كيف وإن تكررت اآلية مرت! .وهللا لن نعدم من رب يضحك خ1ا

قد تضيق الصدور بتكالب الناس عليك وشدة ظلمهم لك جدا وتنرشح.. ولكنها تتسع وتلتجئ إىل هللا وحده.. أ من كل حول وقوة لك عندما تترب

ثم تقرأ يف كالمه عز وجل كل*ت مثل هذه ٤٨الطور{ واصرب لحكم ربك فإنك بأعيننا وسبح بحمد ربك حY تقوم }

يخيل إليها أنه من الجنة.. حتى لتجد يف ذلك نعي* ينبغي لجالل وجهك وعظيم سلطانكلك الحمد ك* .. يا رب نحبك.. يارب

Page 28: ˘ˇˆ˙ - Ain Shams Universitymed.asu.edu.eg/uploads/med/Student Affairs/IEE_IO__2.pdf• Lt. Lower lobe bronchiectesis • Recurrent pul. Embolism • Chronic broncitis (May 96)

Faculty Of Medicine - ASU

Student Union Scientific Committee (SUSC)

Examination Note - 4th year P a g e | 28

EXAMS . . .EXAMS . . .EXAMS . . .EXAMS . . .

Page 29: ˘ˇˆ˙ - Ain Shams Universitymed.asu.edu.eg/uploads/med/Student Affairs/IEE_IO__2.pdf• Lt. Lower lobe bronchiectesis • Recurrent pul. Embolism • Chronic broncitis (May 96)

Faculty Of Medicine - ASU

Student Union Scientific Committee (SUSC)

Examination Note - 4th year P a g e | 29

1992-11

Paper I

All questions to be answered 1. Give the management of sever acute bronchial asthma

• What are the differentiating point between bronchial asthma and cardiac asthma

2. Describe the diagnostic features of TWO endocrine conditions causing loss of weight.

3. Describe the clinical presenta.on of anxiety disorders

• Describe the management of the main psychatric emergencies.

4. A 52 years old lady presented with disturbed generalized itching over a period of 2 weeks. The

dermatologist found no skin lesions.

- What would you particularly care to look for in your clinical examination? Why?

• What investigations would you suggest?

• Discuss the significance of the results expected from your investigations?

5. Discuss the importance of the different shapes of the RBCs

• What is tlie role of the macrophage in the immune defense system?

6. Enumerate 3 diseases of different ae.ologies characterized by the presence of a granulomatous reac.on in

the affected organs. <» Give the SPECIFIC laboratory test for each

• What in your opinion is the best drug therapy for each?

7. What are the hazards of the following and how do they present?

- Iron overload

- Excess serum copper

8. How can you differentiate clinically between rheumtic & rheumatoid arthritis?

9. A 34 years old lady presented with one week's history of severe vomi.ng, malaise, hiccups, nocturia, and

bilateral loin pain. Little urine had been passed over the last two days. These loin pains had been

occurred during her two pregnancies, together with dysuria, and were also repeated three times over

the last two years. On each occasion she was given an antibiotic. On examination she looked ill, flushed,

dehydrated, but afebrile. BP = 100/70. Pulse 100/min, regular, bilateral loin tenderness was present.

Heart & lungs were free. Lab inves.ga.ons showed : Hb 14.2 gm%- WBCs 16000/ cmm- urea 412 mg % -

plasma sodium 120 mEq/L - potassium 8.3 mEq/L - bicarbonate 10 mEq/L.

What are the four essential points that necessitate immediate management in this patient?

- How will you achieve your aim to correct them?

- Could you advise I.V.P.? Give reasons?

10. What are the important points that determine drug therapy in epilepsy?

- Enumerate 4 benign intracranial tumors, giving their usual sites.

Paper II

1- A right handed male aged 22 years, presented with sudden right sided hemiplegia and inability to peak at

all. CVS. Examination revealed a double mitral lesion:

- What other signs & symptoms you look for?

- What in your opinion the first treatment of choice?

2- Enumerate the factors precipitating hepatic encephalopathy in a patient with chronic liver disease.

- Explain the mechanisms by which GIT bleeding causes it.

3- Enumerate 3 causes for hemoptysis of different etiologies.

- Describe the mechanisms of its occurrence in each.

Page 30: ˘ˇˆ˙ - Ain Shams Universitymed.asu.edu.eg/uploads/med/Student Affairs/IEE_IO__2.pdf• Lt. Lower lobe bronchiectesis • Recurrent pul. Embolism • Chronic broncitis (May 96)

Faculty Of Medicine - ASU

Student Union Scientific Committee (SUSC)

Examination Note - 4th year P a g e | 30

4- Describe the known types of personality disorders

5- How to differentiate between organic pyloric obs & pylorospasm?

• Enumerate the risk factors that may cause acute pancreatitis.

6- A Nigerian girl 11 years old visi.ng Egypt was caught with fever & shivering. On examina.on she showed pallor

& splenic enlargement. Blood examination clenched the diagnosis, and fever subsided after specific therapy.

- What were the possible diagnostic features on blood examination?

- What was the specific management given?

- Is any further management needed? Why?

7- Describe the different non-surgical therapies available for gallstones.

5-1993

Paper I

All questions to he answered

Describe the different non-surgical therapies available for gallstones.

1- Define malignant hypertension

• Describe its manifestations

2- Enurnerate five of the most reliable parameters with which to determine the prognosis in lupus nephrits.

- Describe the significance of each

3- Describe the eye changes seen in

- Vitamin A deficiency

- Vitamin B deficiency

4- Mention four causes of hypogonadism describe how you would arrive at the diagnosis in each.

5- Enumerate the sites of lesions causing vertigo and the characteristic features of each.

6- Explain why :

• The spleen is enlarged in Bilharziasis

• Diarrhea occurs in giardiasis

7- Discuss the differentiating signs and symptoms between:

- Pheochromocytoma and anxiety neurosis

- Hysterical and grand mal fits

- Withdrawal symptoms of heroine and barbiturate abuse

8- Give the diagnostic value of sputum

9- A 30 years old man had felt unwell and liad a fever for several weeks. He also had a second episode of

painless haematuria. He had no relevant past medical history. On examina.on he was pyrexial (38°C) and his

blood pressure was 120/70 mmHg. The rest of the examina.on was normal. Inves.ga.ons: bl.urea 35 mg,

plasma sodium 136 mEq/L, potassium 3.6 mEq/L, urine analysis: trace of blood and protein.

Microscopically: 10 RBCs and 2 WBCs per high power field, no casts, plain X-ray of the abdomen showed

calcification in the renal area. IVP was normal apart from calcification in the lower pole of the left kidney.

- Give three possible diagnosis

- What five investigations would you choose to perform? And why?

- What is the follow up in 2 of your differen.al diagnoses

10- What are the line of defense present in the human body against infection

- Describe one clinical situation as an example for defective function of each of them.

Paper II

1- Describe the diagnostic approach to a daibetic patient with coma

2- Describe the clinical features and management of incompatible blood transfusion.

Page 31: ˘ˇˆ˙ - Ain Shams Universitymed.asu.edu.eg/uploads/med/Student Affairs/IEE_IO__2.pdf• Lt. Lower lobe bronchiectesis • Recurrent pul. Embolism • Chronic broncitis (May 96)

Faculty Of Medicine - ASU

Student Union Scientific Committee (SUSC)

Examination Note - 4th year P a g e | 31

3-A 67 years old man complains of difficulty of swallowing of 3 months dura.on. The condi.on was mild at the

beginning with difficult semisolid foods only. At the .me of presenta.on he could only swallow fluids. He lost 10

kg of his weight during the illness and started to complain of hoarseness of voice.

- What is the most likely diagnosis?

- Mention 3 tools of inves.ga.on to reach definite diagnosis.

- Men.on 5 condi.ons that may lead to this defini.ve condi.on

- Give 3 diff diagnosis and how they can be excluded

4- Give the causes and clinical picture ofhyponatraemia

5- Describe the management of:

- Pulmonary embolism

- Acute pulmonary oedema

- Cyanotic spells

6- Describe the neuropsychiatric manifestation of hepatic encephalopathy

7- Enumerate the cause of agranulocytosis and give its clinical picture.

11-1993

Paper I

All questions to he answered

1- Enumerate the endocrine disorders that may be accompanied by hypertension

- Give the causes of hypertension in each

2- Give the causes of bloody diarrhea and describe how they can be differentiated.

3- Enumerate the causes of bilateral facial nerve palsy

- Describe the clinical features suggestive of each cause

- Give one investigation specific for each cause

4- A 43 years old woman ate shellfish, 30 minutes later she developed crampy abdominal pains and profuse

vomi.ng and lost consciousness for about 15 minutes. She then regained consciousness slowly and resumed

vomiting in attacks but was unable to remain in the upright position because other weakness. She noticed

swelling of her hands and itchy rash on her body.

- What is the diagnosis

- What are the manifestations of this same condition in other body systems

- What is the immediate treatment?

5- Discuss me differential diagnosis and management of regular tachycardias.

6- Discuss the differential clinical symptoms and signs between:

- Hystepical and hypoglycoemic coma.

- Mitral valve proiapse and anxiety.

- Panic attack and myocardial infraction.

7- Describe the aetiology, clinical features and management of osleomalacia. State the differences between

osteomalacia and osteoporosis.

8- What are the risk factors favouring of gallstones. How do they act?

9- How does anaemia occur in chronic renal failure?

- Enumerate me conditions, which may be associated with reversible uraemia.

10- What dietary advice would you give to a patient wlio lias recovered from a myocardial infraction. Give your

reasons.

Paper II

1- Give in detail the chemoprophylaxis against malaria

Page 32: ˘ˇˆ˙ - Ain Shams Universitymed.asu.edu.eg/uploads/med/Student Affairs/IEE_IO__2.pdf• Lt. Lower lobe bronchiectesis • Recurrent pul. Embolism • Chronic broncitis (May 96)

Faculty Of Medicine - ASU

Student Union Scientific Committee (SUSC)

Examination Note - 4th year P a g e | 32

- Describe the clinical features in relation to the different organs affected by plasmodium

falcipamm

2- How would you suspect and then confirm the diagnosis of:

- Acute leukaemia

- Cancer head of the pancreas.

3- Describe the different mechanisms by which drugs can cause jaundice. Giving two drugs as an example for

each mechanism.

4- A 40 years old man was brought to the emergency room with acute sudden dyspnea.

- Enumerate the possible differential diagnosis

- Give the diagnostic clinical signs of each.

- Describe the emergency treatment for each.

5- Which organs can be involved in the extra-articular manifestations of rheumatoid arthritis.

- Name two manifestations in each organ

6- Enumerate the extrarenal manifestations of renal tumours and explain how they occur.

7- Describe the usual course of typhoid fever

- What are the specific drugs that can cure the disease, mentioning their doses?

5-1994

Paper I

All questions to be answered

1- How can you differentiate clinically between gall bladder pain and duodenal ulcer pain?

- Enumerate the possible clinical manifestation of functional disorders in different parts of the

gastrointrstinal tract.

2- Describe the clinical picture and complications of measles

3- Describe the treatment of liodgkin's disease in its clinical staging

4- Describe the suitable diet for :

- Hypercholesterolemia

- Chronic renal insufficiency

Give reasons in each

5- Enumerate 4 reasons ofpolyurea

- Explain the mechanism of its production in each

6- Discuss the differential clinical features between:

- Pheochromocytoma and anxiety disorder

- Hysterical and grand mal fits

- Withdrawal symptoms of herione and barbiturate abuse

7- Discuss the treatment of parkinson giving the role of each drug used

8- Discuss the diagnostic value of ascitic fluid analysis

9- Describe the hand deformities in rheumatoid arthritis

- Describe the clinical features of systemic lupus erythematosus. Mention the prognostic parameters.

10- An obese man 56 years old presented, with a syncopal aOack associated with swea.ng. Results of laboratory

inves.ga.ons were: HB 14.5 gm, WBCs 12000/cmm, ESR 32 at first hour, GOT 65 (<25), GPT 18 (<20).

- What diagnosis is suggested by these results?

- What further investigations would confirm your diagnosis?

- Discuss in short the lines of management.

Page 33: ˘ˇˆ˙ - Ain Shams Universitymed.asu.edu.eg/uploads/med/Student Affairs/IEE_IO__2.pdf• Lt. Lower lobe bronchiectesis • Recurrent pul. Embolism • Chronic broncitis (May 96)

Faculty Of Medicine - ASU

Student Union Scientific Committee (SUSC)

Examination Note - 4th year P a g e | 33

Paper II

1- A 25 years old man presented with generalized edema, pallor and a blood pressure of 170/110. He also had

chronic cough of 12 years dura.on with expectoration of more than glassful! of mucopurulent sputum during

exacerbations of the illness.

- How to investigate this case.

- How to investigate the possible relationship between the presenting features and its original

disease condition.

- Discuss the lines of management.

2- What parasites can enter the body through "skin penetration"? Describe their route after their

entrance.

- Discuss the laboratory methods available for diagnosis of entameba histolytica infection.

3- A patient gets proxysmal auditory hallucinations, and aggressive behaviour and then amnesia to the attack.

Discuss the differential diagnosis when it is proxysmal and when it is permanent.

4- How would you suspect and then confirm that a patient is suffering from rapidly progressive

glomerulonephritis?

5- Discuss the management of diabetic ketoacidosis.

6- Discuss the treatment of sever acute bronchial asthma

7- Enumerate the cause of pericarditis

8- Discuss the diagnostic value of ascitic fluid analysis.

5-1995

Paper I

All questions In be answered

1- A 35 years old man presented with chest pain, marked dyspnea and fa.gue of one week dura.on. Three

months earlier lie had anorexia, vomiting and hicough. Ten years earlier he was operated up on for bilharzial

ureteric strictures. Examination revealed marked pallor, air hunger, tremors of hands, blood pressure 160/120

and an extensive pericardial rub heard.

- What is your diagnosis?

- How would you investigate this patient?

- Discuss the management

2- Men.on 5 complica.ons of diabetes mellitus. Discuss the management of one of these complications

3- Discuss differential clinical symptoms and sign between:

- Hysterical and organic hemiplegia

- Thyotoxicosis and anxiety disorder

- Panic attack and myocardial infraction

4- Write a short essay on:

- Classification of bronchial asthma

- Treatment ofnon-thrombocytopenic purpura

- Immunopotentiating drugs

- Desensitization therapy

- Intravenous gamma globulin therapy.

5- A 68 years old man with history of progressive dysphagia to solid food of 6 months dura.on was admiOed

to hospital. On admission he could only swallow fluids. He complained also of irrita.ve cough and has lost 12

kg during the last 6 months. His hemoglobin was 6.8 gm/dl, and ESR 92 mm in the first hour.

- What is your provisional diagnosis

- Men.on 5 points in favour of your diagnosis

Page 34: ˘ˇˆ˙ - Ain Shams Universitymed.asu.edu.eg/uploads/med/Student Affairs/IEE_IO__2.pdf• Lt. Lower lobe bronchiectesis • Recurrent pul. Embolism • Chronic broncitis (May 96)

Faculty Of Medicine - ASU

Student Union Scientific Committee (SUSC)

Examination Note - 4th year P a g e | 34

- How would you investigate this case?

6- Discuss the causes, clinical features and treatment of potassium depletion.

7- Discuss the causes, clinical features and management of infective endocarditis.

8- How would you diagnose polyarteritis nodosa? Discuss the treatment.

Paper II

1- Enumerate 5 psychiatric emergencies and their management.

2- What is the definition of nephrotic syndrome? Enumera causes of this syndrome.

3- Discuss the causes of upper gastrointestinal bleeding

4- How can you differentiate between organic and hysterical hemiplagia?

5- Discuss the complications of urinary schistosomiasis.

6- Enumerate 5 lines of management ofvariceal bleeding.

7- Discuss the treatment of amebic liver abscess.

8- Discuss the management of chronic lymphocytic leukaemia.

9- Men.on 6 main symptoms of depression.

10- Discuss the causes of hemoptysis.

12-1995

Paper I

1- In a patient complaining from cough. Mention how can you from the history alone suspect that he may be

having:

- Lt. lower lobe bronchiectasis

- Recurrent pulmonary embolism

- Chronic bronchitis

2- Men.on 2 main causes of raised venous pr. How can you differen.ate between the 2 causes from

examination of the neck veins alone.

3- Mention the definition of acute dysentry. Enumerate the causes of such condition. Mention the treatment of

them only.

4- What are the causes of an enlarged tender liver. What are the clinical examination; date that only can

differentiate them from each other.

5- Mention the specific value of the following lab tests in the clinical medicine:

- Serum LDH

- Partial thromboplastin time

- Antimitochondrial abs

- 24 hrs albumin in urine

6- In a patient with rt. sided hemiplegia. Mention the clinical data that can make you define its level as :

- cortical

- a lower pontine level

- spinal

7- Mention the prophylactic treatment; its dose and indication in the following conditions:

- Malaria

- SBE

- Type A hepatitis

8- Mention the treatment; its dose and duration in the following:

- Pcrniciuos aneamia

- Brucellosis

Page 35: ˘ˇˆ˙ - Ain Shams Universitymed.asu.edu.eg/uploads/med/Student Affairs/IEE_IO__2.pdf• Lt. Lower lobe bronchiectesis • Recurrent pul. Embolism • Chronic broncitis (May 96)

Faculty Of Medicine - ASU

Student Union Scientific Committee (SUSC)

Examination Note - 4th year P a g e | 35

9- Enumerate all the cases of SR insufficiency

10- Describe the management of a suicidal attempts in a medical word.

Paper II

Comment on the following case histories by stating the following items for each question:

A: the most probable diagnosis

B: the most probable diagnosis that can share the same history

C: the most useful single investigation that can confirm the diagnosis, non-invasisve investigation being preferred

if possible.

D: a short summary for the treatment of the case based on your most probable diagnosis only.

1- A 22 years old girl presented with fever and joint pains since one month" Examina.on .revealed T 38 °C,

pulse 110/min, malar flush, and silky light hair. Her skin shows reddish skin moOling. The knees, ankles and

wrists are swollen and tender. Urine examination revealed heavy albuminuria.

2- A 50 years old male came complaining from dark brown urine since 2 weeks. He could no.ce that he has lost 8

kg of weight in the last 4 months. Examina.on revealed that he is markedly jaundiced, the liver is enlarged

and not tender. The gall bladder could be palpated.

3- A traffic police soldier 40 years old came complaining from cough accompanied with stabbing pain in the chest

& liOle blood .nged sputum since 3 days. Past history revealed 2 similar attacks that occurred since one

month and since 3 months, though not accompanied by haemoptysis & were cured spontaneously in a week

time.

4- A mother presented her male child 12 years old to the clinic. She no.ced that he climbs the stairs slowly & with

difficulty & that he tries to support himself by grasping his legs with his hands on standing. Examination

revealed that the muscles of the lower limbs are bulky, but the ankle & knee reflexes are very weak.

5- A 38 years old lady, no.ced that she is loosing weight and suffers from anxiety & insomnia at night.

Examina.on revealed a T 36.3 °C, pulse 135/mm, and she has warm trembling hands. The sedimenta.on rate

is normal.

6- A Sudanese student 21 years old, studying in Egypt was broughtby his friends to the R.R in a semicomatosed

state since one day. They noticed that lie passed too dark urine this day. History taking revealed that he

returned from vacation which he has spent in his country since 3 weeks.

7- A 28 years manual worker presented with a sense of fever & marked general fa.gue in the last 3 weeks.

Examina.on revealed pallor & a toxic look, his T 37.8 °C, BP 350/50, pulse is 100/min & he has clubbing of

fingers. Urine examination revealed many red cells & one plus alb.

8- A 10 years old child was referred by the ENT specialist to the medical clinic. His disease started since 3 weeks

by a tonsillitis like pictures but was not responding to proper AB in adequate doses. In the last week the ENT

specialist noticed that his gums became swollen & bleeding & that he is having very frequent epistaxis.

Examination revealed that the child looks ill, lie has enlarged cervical LN. The blood examination revealed a

WBC count of 6000/mm with some abnormal ceils. The other lymph glands & spleen are not enlarged.

5-1996

Paper I

1) Classify the causes of dyspnea & Select two causes you men.oned & state:

- How can you recognize each of 2 from the other causes of dyspnea

- How would you manage each of the 2 cases

2) Men.on shortly the various clinical presentations considered as psychiatric emergencies.

Page 36: ˘ˇˆ˙ - Ain Shams Universitymed.asu.edu.eg/uploads/med/Student Affairs/IEE_IO__2.pdf• Lt. Lower lobe bronchiectesis • Recurrent pul. Embolism • Chronic broncitis (May 96)

Faculty Of Medicine - ASU

Student Union Scientific Committee (SUSC)

Examination Note - 4th year P a g e | 36

- Mention shortly the immediate management in each presentation

3) Men.on the essen.al symptoms & signs of acute myocardial infrac.on.

- Select 3 serious signs & discuss the immediate management for each

4) State the ques.ons that can help you to reach the possible causes in a pt presen.ng mainly with dysphagia

mentioning the reason in each question.

5) Enlist the cause of polyarthri.s

- Mention the lab tests that may be done for diagnosis stating whether each of the tests you mentioned is

specific, equivocal or non-specific.

6) Discuss the causes of diete.c deficiencies that are not due to food lack (condi.oned deficiencies).

- Mention the missing dietetic factor in each cause & how you restore it therapeutically.

7) Classify the causes of the stunted growth

- Select one cause of each class you stated mentioning its essential diagnostic features & the main line of

ttt.

8) Enumerate various types of nephri.s men.oning the commonest cause & most important line of ttt for each

type.

9) Classify the causes of paraplegia developed in less than 3 weeks dura.on

- Mention one investigation essential for each cause

10) Enumerate the disease that can be caused by unicellular parasites

- Mention the specific drug therapy for each.

Paper II

"In a maximum space of one page of your answer booklet write short comment on the following"':

Comment on the following case histories bystating the following items for each question:

1. The most probable diagnosis

2. The most Probable diagnosis mat can share the same history

3. The most useful single inves.ga.on that can confirm the diagnosis, non-invasive investigation being preferred

If possible.

4. A short summary for the treatment of the case based on your most probable diagnosis only.

1- A 45 years male complaining from difficulty in swallowing of 4 years dura.on, more to fluids than solids & felt

behind the lower part of the sternum, commonly accompanied by retrosternal pain with radiation to both

arms.

2-A 45 years female used to self administer pain killers for her joint pain in the last 6 months started to no.ce

yellow discoloura.on of her eyes. In the last 2 weeks examina.on revealed hepa.c enlargement four fingers

below the costal margin. In addition to the jaundice, there are no spider nevi & no oedema other lower

limbs.

3- A male 45 years, moderate built, diabe.c, was controlled on oral hypoglycemics the last 3 years, stated to have

cough with liOle sputum in the last 4 weeks & no.ced that he is having polyuria & thirst. he could notice that

he lost 4 kg of weight in the last month.

4- A 25 years old male who was working abroad for 2 years started to complain from irregular fever since one

moth with joint & bone pain. Examination revealed fever of 38 °C together with generalized LAD (1-2 cm in

size) &. a palpable spleen.

Page 37: ˘ˇˆ˙ - Ain Shams Universitymed.asu.edu.eg/uploads/med/Student Affairs/IEE_IO__2.pdf• Lt. Lower lobe bronchiectesis • Recurrent pul. Embolism • Chronic broncitis (May 96)

Faculty Of Medicine - ASU

Student Union Scientific Committee (SUSC)

Examination Note - 4th year P a g e | 37

5- A 10 years old child presented complaining from stunted growth & bouts of fever, which recurs frequently

since a long .me. Examina.on revealed pallor, enlarged liver 4 fingers below the costal margin & an enlarged

spleen 5 fingers below the costal margin.

6- A heavy smoker lady on CP used to feel dyspnea on moderate exertion & frequent bronchitis in winter, which

she aOributed to smoking in the last 2 years. She came to emergency room with sudden marked weakness in

her rt arm with difficulty in walking and difficulty in expressing herself in words.

7- A 54 years old male on irregular Ot of hypertension presented complaining from general fa.gue dyspnea,

polyuria & mild diarrhea in the last 2 weeks. Examina.on revealed BP 210/120 & a heaving apex, in addi.on

to mild skin dehydra.on. Urine examina.on revealed pale color & 1 + Alb.

11-1996

Paper I

1- Mention the treatment of an acute attack of bronchial asthma in an adult, taking in consideration the order of

drugs to be used, their doses, and the route of administration.

2- Mention in short the early management of a case of haematemesis

3- Describe the early symptoms and signs of incompatible blood transfusion

4- Enumerate the medical conditions that can present with an anxiety state - like clinical picture. Suggest a

differentiating test for each condition.

5- Describe in short, the main clinical signs and lab findings of what you consider to be the minimum data

compatible with diagnosis of acute nephritic syndrome.

6- Mention the most effective drug treatment for each of the following conditions, including the doses, and

duration of treatment:

- Hepatic amebiasesS

- Tenia saginata infestation

- Enterobious vermicularis infestation

7- Men.on the treatment for a diabe.c adult who was controlled on forty units of mixed insulin 1 day, BUT

developed sterpococcal pneumonia. The doses, route of administration and duration for your treatment will

be appreciated.

8- Describe in short, the secondary propylac.c medical therapy for an adult mole, 45 years old, who is discharged

from the cardiology department after cure from a small myocardial infract.

Paper II

Dear student, read the following cases histories slowly, then comment on each case regarding:

- The most probable diagnosis.

- Other diagnosis That can share the same clinical picture.

- The minimum investigations necessary to prove your diagnosis, (non-invasive investigation are better if

sufficient), and finally,

- The treatment in short for the case based on your most probable, diagnosis.

1- A 25 years old female presented with irregular fever, and pain in the right upper part of the abdomen of one

month dura.on. Examina.on revealed temp. 37.7 °C, enlarged tender liver, and no palpable spleen. Blood

picture showed EBC 15000 / cm, 75 % of the WBC were eosinopils, and serum bilirubin is 2 mg/dl.

2- A 12 years old child presented wit fever and joint pains of two weeks dura.on. Examina.on revealed that his

temp is 37.5 °C, pulse 110 / min, and the tonsils are enlarged and congested. Cardiac auscultation revealed

faint heart sounds, and a soft systolic murmer over the apex. ECG revealed prolonged P-R interval.

Page 38: ˘ˇˆ˙ - Ain Shams Universitymed.asu.edu.eg/uploads/med/Student Affairs/IEE_IO__2.pdf• Lt. Lower lobe bronchiectesis • Recurrent pul. Embolism • Chronic broncitis (May 96)

Faculty Of Medicine - ASU

Student Union Scientific Committee (SUSC)

Examination Note - 4th year P a g e | 38

3- A 25 years old male felt acute s.tching pain in the leV side of the chest while driving a bicycle, followed shortly

by marked dyspnea. He presented himself to the doctor, who discovered that the left side of the chest is

larger than the right and shows decreased air entry.

4- A 65 years old man presented to R.R. in coma, examina.on revealed that his blood pressure is 130 / 80 the

heart, chest abdomen are free. Neurological examination deviation of the mouth to the left side, and rt

extensor planter response.

5- A 45 years old lady presented to the medical clinic complaining from an increasing sense of general fatigue and

intolerence to colds since three months. In the last two weeks her voice became increasingly hoarse.

Examina.on revealed that her temp is 36.9 °C, pulse 60/min, BP 140/80, her skin is dry. Neurological

examination was negative but the knee reflexes were slow in their response.

6- A. 30 years old unmarried male presented complaining from joint pains in both knees, and fingers since three

weeks, together with some dysuria during micturition. Examination revealed that her temp is 37.3°C, he has

congested eye, a mild maculae skin rash, and the sacro-iliac joints are tender. The WBC count is 11000/cmm

and the sedimenta.on rate is 85/mm in the first hour.

7- A 20 years old newspaper seller presented to the R.R. complaining from general weakness, swelling of lips and

eye lids together with itchy skin rash since one day. Examination revealed that the skin rash is urticarial, the

temp is 37.5°C. There is generalized lymphadenopathy, and the chest is wheezing. Detailed history taking

revealed that he suffered a street cut wound in his foot, which was treated in the R.R. since 6 days.

Page 39: ˘ˇˆ˙ - Ain Shams Universitymed.asu.edu.eg/uploads/med/Student Affairs/IEE_IO__2.pdf• Lt. Lower lobe bronchiectesis • Recurrent pul. Embolism • Chronic broncitis (May 96)

Faculty Of Medicine - ASU

Student Union Scientific Committee (SUSC)

Examination Note - 4th year P a g e | 39

May 1997

Paper I

1- Give an account on

- Tachycardia

- Facial palsy

- Increased ICT

2- Causes, CI/p & management of hepatic encephalopathy

3- Discuss the management of:

- Thyrotoxicosis

- Rheumatoid arthritis

- Pulmonary TB

4- Give an account

- Chronic diarrhea

- Pulmonary oedema

Paper II

Time allowed 2.5

All questions to be answered with comment on each case according to questions proposed

1- A 19 years old medical secretary-was admitted to hospital after she had fainted at work. She mentioned feeling

.reness in the last 6 months. On examina.on she was pale wiih no other remarkable physical signs. Her

pulse was 100/min, BP was 100/50 &. temp 37°C. Inv.: Hb 7 gm/dl, RBCs hypochromic & microcy.c, chest X-

ray & ECG were normal.

- What are other clinical & laboratory findings may help in the diagnosis of this girl's anemia.

2- A 60 years old male was recovering from a myocardial inf. in hospital. 5 days aVer transfer

from CCU to a general ward he woke up in the morning with severe pain in his left leg. There was no obvious

swelling or calf tenderness. The pulse was 125/min & ireg. His BP was 120/80 mmHg. His ECG showed AF &

signs of resolving infarction.

- Give 2 possible causes of the pain in the leg.

- What would be your further management & suggestions of appropriate ttt?

3- A 45 years old female was suffering from abdominal discomfort & fat intolerance for the last 2 years. She

developed upper abdominal pain, N&V in the last 10 days followed by abdominal disten.on & fever. On

examina.on, she was pale & tetanic spasm was easily evoked in her hands, Temp. 38.5°C a swelling was seen

& felt in the epigastric region. Immediate U/S showed calcular gall bladder & dilated common bile duct & a

large heterogeneous mass was seen inferior to the pancreas.

• How would you correlate the pt. history, physical signs & U/S findings.

• What is the cause of the patient carpal spasm?

• Do you accept the physician attitude upon conservative management of this case.

4. A 77 years old lady with 16 years history of DM which has always been perfectly controlled on strict diet &

glibenclamide 10 mg/d. She developed one day morning confusion & restlessness, on examination she was

disoriented with wet skin & inability to move her Lt upper & lower limbs with an extensor planter response on

that side. Her temp was 35°C. when she was given IV glucose her confusion & restlessnes improved & she

started to move her Lt arm & leg:

• What is your diagnosis

Page 40: ˘ˇˆ˙ - Ain Shams Universitymed.asu.edu.eg/uploads/med/Student Affairs/IEE_IO__2.pdf• Lt. Lower lobe bronchiectesis • Recurrent pul. Embolism • Chronic broncitis (May 96)

Faculty Of Medicine - ASU

Student Union Scientific Committee (SUSC)

Examination Note - 4th year P a g e | 40

• What is your further management

5. A 55 years old lady was referred to a medical clinic complaining of breathlessness on exer.on with plain &

colour change of her hands in cold weather for the last years. During the last year she started to complain of

difficulty in swallowing. On examination bluish discolouration & tapering of the fingers with tight skin.

Tachypnea & basal chest crepitations.

• What is the diagnosis of the colour changes in this lady's hand. Mention other causes of this phen.

• How do you account for the chest symptoms & signs.

• Why does this patient complain of dysphagia

6. A 8 years old man who smokes 40 cigareOes a day gives a history of chronic cough for many years. In

winter, he brings up purulent sputum with episodes of fever & ill health. Prior to the present admission

he had lost 8 kg of body weight and his breathing had become more diff. & he started to bring blood sp.

On examina.on he had pyrexia 38°C, finger clubbing & he was breathless at rest & chest expansion was

poor with signs of pleural effusion on the rt.side.

• What is your interpretation of the pt history

• Give 3 possible causes for this pts haemoptysis

• Suggest 3 important inv to diagnose the case.

7. A 40 years old house-wife started to notice that she feels more than usual fatigue specially at the end of

the day. The condi.on became progressively severe, so that within 6 months she can only open her eyes

with difficulty at the end of the day & is having difficulty in swallowing the food at the end of the meal &

her voice changes if she speaks long enough.

• What is the most probable diagnosis.

• Investigations necessary to prove your diagnosis.

• Treatment in short.

11-1997

Paper I

1. Define the clinical features of shocks. Mention the three main types of shock and the causes of each type.

2. Classify the DD of chest pain. Describe in details the characteristics of pain in any two diseases you choose

from your classification.

3. Enumerate the common lung diseases that occur in a patient with AIDS and Mention in short the

treatment for two of them.

4. Enumerate the causes of Atrial Fibrillation. Mention the treatment of the condition in details.

5. Mention the factors that predispose to DVT and the prophylactic precaution for this disease.

6. Discuss how can you diagnose a case of hepatocellular carcinoma.

7. Mention the tests that are used in a patient having bleeding disorder and the clinical disease in which

each test is abnormal.

8. Mention three medical diseases, which may present as psychiatric disorder. Write down the main

differentiating clinical features and essential investigations for each of them.

Paper II

Comment on each case regarding

(a) The most probable diagnosis

(b) Other diagnosis that can share the same clinical picture if any.

(c) The minimum investigations necessary to prove your diagnosis taking in consideration that non-invasisve

tests are preferable than the invasive.

(d) The treatment in short based on your most probable diagnosis.

1. A female pa.ent 40 years complains from exhaustion and asthenia in the last year, with loss of weight.

She noticed that her tongue and checks are becoming mottled with dark areas. Examination revealed she

is underweight, BP 90/60, dark pigmented area on checks and tongue were confirmed.

Page 41: ˘ˇˆ˙ - Ain Shams Universitymed.asu.edu.eg/uploads/med/Student Affairs/IEE_IO__2.pdf• Lt. Lower lobe bronchiectesis • Recurrent pul. Embolism • Chronic broncitis (May 96)

Faculty Of Medicine - ASU

Student Union Scientific Committee (SUSC)

Examination Note - 4th year P a g e | 41

2. 60 years male, who had very regular bowel habits allover his life, started to have constipation of

increasing severity in the last 6 months not responding easily to usual laxa.ves. He lost 6 kg of weight

during this period. Examination revealed exhausted face but no other signs.

3. 14 years boy presented by his mother complaining of increasing fa.gability and dyspnea on exer.on in

the last year. Examina.on revealed pulse 110/min, paradoxus character, neck veins are congested non-

pulsating. Local examination of the hear revealed that the apex in neither visible nor palpable and there is

protodiastoic gallop on apical auscultation.

4. 25 years female presented by her parents in a very weak state. They complaint that she is having fever

since more than 6 months with rapid loss of weight. The fever is not responding to general antibiotics.

Examina.on revealed severe cachexia, temperature 37.8°C, pulse 115/min, BP 110/65. There is

generalized enlargement of the lymph nodes in addition to moderate but definite and painful neck

rigidity. Chest examination revealed signs of infection in the left upper lobe.

5. 60 years old man presented in coma and urinary incon.nence since one week. His coma varies in severity

during this period with occasional hours of improved consciousness. He is neither diabetic nor

hypertensive, but one month ago he fell in the bathroom and wounded his head with a resulting

hematoma which resolved aVer 2 weeks. Examina.on revealed semi-coma and the reflexes are

exaggerated allover but there are no other signs.

6. A male child 13 years old presented with face and leg oedema, more in the morning since one month with

increasing severity. He used to have tonsilli.s in the last 2 years frequently. His trea.ng doctor, aVer one

of these tonsillitis noticed that he is having high BP which however returned to normal after two weeks.

Examination revealed the child is pale, swollen eyelids, soft pitting oedema of lower limbs, BP is normal.

7. 25 years male presented with marked polyurea, polydipsia since 2 weeks that made him markedly

exhausted. There is no family history of diabetes. Examination revealed generalized enlargement of

lymph glands, there is erythema nodosum on his legs. Chest X-ray revealed broad superior mediastinum.

And sedimenta.on rate is 40 mm/hr. urine examina.on revealed pale urine specific gravity 1006 and free

from sugar.

8. 22 years girl presented with fever and joint pains since one month. Later she developed leV sided chest

pain followed by progressive dyspnea. Clinical examination revealed temp. 38, pulse 110/min, malar

flush, reddish skin mottling, in addition to swelling and pain in the knee, ankles and wrists. Urine

examination revealed heavy albuminuria.

Final MBBCH

11-1998

Paper I

Time allowed: 3 hours

All ques.ons to be answered: 10 marks for each question

1. Classify the causes of tetany. Mention the methods by which you can provoke an attack in the clinic in a

patient complaining from the latent disease.

2. Enumerate the factors that can precipitate porto-systemic encephalopathy in a patient with liver

cirrhosis. Mention the principles of treatment of such condition when it occurs.

3. Mention how can you diagnose a case of systemic lupus erythematosis.

4. Enumerate the causes of reversible dementia in the elderly patient.

5. Enumerate the immunological emergencies that are based on type II (cytotoxic-cytolytic) pathogenesis.

Mention the general management for this kind of emergencies.

6. Discuss in short the treatment of the various hyperlipidaemias.

7. Mention the drug treatment including the doses for the following infestations:

• Taenia saginata

• Enterobius vermicularis

• Cutaneous lishmaniasis

Page 42: ˘ˇˆ˙ - Ain Shams Universitymed.asu.edu.eg/uploads/med/Student Affairs/IEE_IO__2.pdf• Lt. Lower lobe bronchiectesis • Recurrent pul. Embolism • Chronic broncitis (May 96)

Faculty Of Medicine - ASU

Student Union Scientific Committee (SUSC)

Examination Note - 4th year P a g e | 42

8. Enumerate the indications for anti-coagulant therapy, together with the expected duration of treatment

for each indication. Mention the contra-indications for this type of therapy.

Paper II

Time allowed: 2.5 hours

• Please read the following case histories slowly and carefully then comment on each case regarding

A- The most probable diagnosis (3 marks)

B- Other diagnosis that can share the same clinical picture if any (3 marks)

C- The minimum investigations necessary to prove your diagnosis taking in consideration that non-invasive

tests are preferable than the invasive. (2 marks)

D- Treatment based on your most probable diagnosis. (2 marks)

1- A 40 years old married woman came to the clinic complaining from vague abdominal pains of 3 months

duration, with the certainly that she has cancer. History taking revealed that she experiences early

morning awakening, loss of appetite and weight, inability to do home work, and is thinking always that

her family would be better off without her. Although examination and all investigations are normal. She

continues to believe that she has cancer.

2- A 52 years old obese and smoker male is under treatment for hypertension presented to the RR with

marked upper epigastric pain radiating to the chest and neck, together with palpitation and sense of

being about die. This occurred two hours after a heavy meal. Examination revealed an anxious facies,

pallor and swea.ng. The BP is 160/100, and the pulse is irregular.

3- A 60 years old male on oral treatment for diabetes got a sudden sense of coldness and chilling, followed

shortly by the development of pyrexia. Next day cough started to appear, increased rapidly in severity,

and became painful. By the fourth day, he became markedly ill, dyspnoeic, and the cough became very

distressing, rusty sputum started to appear, and increased progressively. Examina.on on the 6th day

revealed 38°C, 110/70, 110/min pulse, marked dullness on the right lung base with decreased air entry

and medium sized consona.ng crepita.ons. The WBC was 14000/mm.

Final MB B.Ch Exam

11-2001

1. What is meant by curable hypertension? Men.on 4 important causes and how to diagnose one of

them.

2. Enumerate: a- 10 causes of clubbing of the fingers.

3. Enumerate 5 sexually transmited diseases. Discuss how the liver is characteris.cally involved in one of them.

4. Discuss lung abscess: aetiology, clinical picture, diagnosis, complications and treatment.

5. Discuss brucellosis.

6. a) What are the medical causes of depression?

b)What are the reversible causes of dementia?

7. Discuss the value of the following serological tests in rheumatology:

a- Raised ESR c- positive rheumatoid factor

b- Raised anti-streptolysin O titre d- high serum urine acid.

8. A 53 years old pa.ent presented to the emergency room aVer the onset of right hemiplegia. Discuss the

steps for management of this case.

9. What is the definition of hypoglycaemia? Discuss the important clinical features and causes in diabetics and

non-diabetics.

10. Discuss how the kidneys are involved in the following:

a- Diabetes mellitus

b- Hepatitis

Page 43: ˘ˇˆ˙ - Ain Shams Universitymed.asu.edu.eg/uploads/med/Student Affairs/IEE_IO__2.pdf• Lt. Lower lobe bronchiectesis • Recurrent pul. Embolism • Chronic broncitis (May 96)

Faculty Of Medicine - ASU

Student Union Scientific Committee (SUSC)

Examination Note - 4th year P a g e | 43

c- Rheumatoid arthritis patients.

11. Discuss medical management of:

a- Peptic ulcer disease.

b- Thrombocytopenic purpura.

c- Acute amoebic dysentery.

5-2000

1- Mention five cause of A.F. Discuss its complications.

2- Mention three causes of enlarged tender liver and discuss how to diagnose each of them.

3- Enumerate 10 causes of Arthri.s.

4- a) What are the physiological manifestations of anxiety.

b) What is the clinical picture and management of Gullian Barre's syndrome?

5- Discuss drug treatment and complications of typhoid fever.

6- Discuss clinical picture and treatment of bronchial asthma.

7- Enumerate five cause of sore throat and their characteristics blood picture.

8- What are the oral anti diabetic drugs you know?

9- What are the indications, mechanism of action, does and complications?

10- What is meant by bleeding per rectum?

11- What are the characteristics and pathological features of Nephritic Syndrome? Enumerate its cause?

Case Histories 5-2000

1- A 25 years old female presented with sudden sever dyspnea and haemoptysis of frank blood following effort:

She mentioned history of rheumatic fever during childhood.

• What is your possible diagnosis?

• What are the clinical findings you expect to find on examination of the pericordium?

• Mention one investigation that properly assesses her illness.

2- A 50 years old male who is Sheisha smoker for 20 years and diabe.c for 10 years. He is on insulin but his

diabetes recently got out of control with cough, colored sputum night fever, anorexia and weight loss.

• What is the possible diagnosis?

• What are two investigations that help in the diagnosis?

• Mention your plan of treatment.

3- An adult Egyptian man came back recently from a visit to South Sudan. There, he suffered from fever, rigors

and sweating and was diagnosed as malaria but received irregular treatment. Now he complains of fever,

rigors and passage of very dark urine.

• What are the complications he developed after he returned back to Egypt?

• What is the most likely drug he received in Sudan?

• How can you manage this complication?

4- A woman 44 years old complaining of increasing weakness and cons.pa.on over the last three moths on

examina.on, she was pale with puffy eyelids. Her pulse was 60/minute and regular. Her serum TSH 201U/ml

(N from 1-6).

• What is the possible diagnosis?

• How can you explain the clinical and laboratory findings?

• What is the treatment and how can you follow up her condition?

5- An elderly man with established cirrhosis and ascites deteriorated recently with drowsiness, hand tremors

and temperature 37.8°C.

6- What is your aged 32 years consulted her general prac..oner because of geWng pale and unable to

perform her usual ac.vi.es. He found her Hb. 8.7 gm/dl, re.culocy.c count 0.3% and microcytosis in the

blood film. Serum iron was 28 Hg/100 and total iron binding capacity 498 Hg/100ml.

Page 44: ˘ˇˆ˙ - Ain Shams Universitymed.asu.edu.eg/uploads/med/Student Affairs/IEE_IO__2.pdf• Lt. Lower lobe bronchiectesis • Recurrent pul. Embolism • Chronic broncitis (May 96)

Faculty Of Medicine - ASU

Student Union Scientific Committee (SUSC)

Examination Note - 4th year P a g e | 44

• What is the diagnosis and why?

• Mention three common causes for this condition in this age group.

7- A 45 years old businessman taking aspirin for occasional headaches, presented with fainting and melaena.

• What is the diagnosis?

• Give one investigation to assess the case.

• What is your management?

• What is your management?

8- A 38 years old lady consulted her doctor for the past three years with painful joints. These joints were knees,

Shoulders and Wrist and later on the fingers and temperomandibular joints were affected. The morning. Her

doctor had prescribed NSAIDs for her with some benefit. Prior to admission she developed a fever and a

sharp pain in the left axillary region.

• What are the possible causes for her polyarthritis?

• How do you account for the last complication?

• What investigations are indicated?

11-2000

1- Men.on 5 causes of weight loss Define how to reach to the diagnosis?

2- Enumerate 10 causes of polyneuritis?

3- Men.on 4 groups of drugs used in the management of conges.ve heart failure and what the principle behind

the therapeutic value of each group?

4- Discuss manifestations and complications of portal hypertension?

5- Discus vascular complications of diabetes mellitus?

6- Men.on 5 causes of fever with arthri.s how to treat a case of acute gout?

7- What are diagnostic features of each of the following:

- Iron deficiency anaemia - Acute Leukaemia

8- Mention how to determine the level of the lesion in a case of Hemiplegia?

9- Discuss chest pain of non-cardiac origin?

10- Men.on 5 causes of pleural effusion and how to reach to their diagnosis?

11- Mention symptoms and signs of chronic heart failure and discuss how to determine its cause?

2000 -Case histories 11

1- A 64 years old man no.ced that he had gained 8kg in weight over the past six months he started to get

bruising easily he found difficulty in getting up from his arm chair or to climb stairs he felt depressed on

examination he is over weight particularly on the abdominal region with purpurish stria on his abdomen and

thighs his blood pressure in 168/100 examina.on of his heart respiratory system and abdoment is normal

there is some weakness in shoulder and hip regions his routine examinations are within normal aport from

elevated 2 hours post prandial blood sugar.

- What is the provisional diagnosis?

- How would you investigate and manage this patient?

2- A 44 years male was admiOed to hospital with a two weeks history of fever and right upper quadrant

abdominal pain there is no urinary symptoms, physical examination reveled a chronically ill appearing man

with temperature 38C there was no jaundice there was moderate hepatomegaly with tenderness all over the

right upper quadrant and inter costal tenderness there was no splenomegally on examination of the chest

there was dullness and diminished air entry over the base of the right lung.

- What is the mmost probable diagnosis?

- Men.on 2 possible DD?

- How to reach the proper diagnosis?

3- A 26 years old man complained of general weakness for 3-4 weeks he some.mes feels himself feverish he

may have lost few kilograms of weight over his period with depressed appetite on few occasions he had some

swea.ng at night on examina.on the pusle was 70 minute his blood pressure 110/60 there is an early

Page 45: ˘ˇˆ˙ - Ain Shams Universitymed.asu.edu.eg/uploads/med/Student Affairs/IEE_IO__2.pdf• Lt. Lower lobe bronchiectesis • Recurrent pul. Embolism • Chronic broncitis (May 96)

Faculty Of Medicine - ASU

Student Union Scientific Committee (SUSC)

Examination Note - 4th year P a g e | 45

diastolic murmur heard along the leV sternal border the spleen was palpable 2 CM below the leV costal

margin.

- What is the most probable diagnosis

- What do you expect to find in ECG X-ray chest and urine examination?

- How to prove the diagnosis what the immediate management?

4- A 30 years man presented with sever headache vomi.ng pyrexia and photophobia of 2 days dura.on by

examination there was neck stiffness but his heart chest and abdomen were free except for a scar of previous

splenectomy 5 years previously following a road traffic accident there was no neurological focal signs and his

fundi were normal.

- What is the most logical provisional diagnosis?

- Men.on 2 other possible diagnosis?

- Mention a ingle investigation that can confirm the diagnosis and mention the characteristic findings in each?

- What treatment should be given immediately for the most logic diagnosis?

5- A 67 years old man aOends his physician with persistent vomi.ng as the main complaint over the last four

months he sometimes vomit food eaten many hours previously his past medical history is unremarkable

except for some dyspepsia there no abnormalities to find on examination except a sunccession splash in the

upper abdomen his blood pressure was 100/60, his blood urea was found to be 98 mg/dl and his creatinine

was 8 mg/dl his sodium was 128 mmol/1 and his chloride was 82 mmol/1 and his bicarbonate 40 mmol/1

• What is your provisional diagnosis?

• How to diagnose the case and its aetiology?

• What is your comment on the bio-chemical findings?

6- A 23 years old nurse recently developed mul.ple spontaneous bruises over her trunk and limbs and as been

having heavy menses she has also noted sore throat with pain full ulcers on examination she is pale and there

are multiple bruises over her limbs and trunk and there is a deep ulcer over the soft palate with oral

condidacies her gums are bleeding spontaneously Examination otherwise normal blood picture was done

which was found to be diagnostic.

- men.on 2 possible causes and their characteris.c blood picture.

2001-5

1- An elderly man aged 55 years with established cirrhosis and ascites deteriorated recently with drowsiness-

hand tremors and temperature 37.8c.

• What is your diagnosis?

• Name five precipitating factors for deterioration?

• What is your immediate diagnosis?

2- A 30 years old male gives history of con.nuous wheezing and breathlessness for four days. He hade frequent

attacks of asthma since childhood and maintained on bronchodilators by his physician.

• What three physical signs will help in assessing the severity of this patient asthma?

• If the diagnosis was established mention three immediate lines of management?

3- A 19 years old girl was admiOed to hospital aVer she had fainted at work she complains of easy fa.gability in

the last six months on examination she was palie with no other remarkable physical signs her pulse was

100/mimn blood pressure was 10/50 and temperature microcy.c chest X-ray and ECG were normal.

• What other clinical and laboratory findings may help in the diagnosis of this girl' anaemia?

4- A 60 years old man was recovering from a myocardial infarc.on in hospital 5 days aVer transfer from the

coronary care unit to a general ward he woke up one morning with severe pain in his left leg there was no

obvious swelling or calf tenderness the pulse was 125/min and irregular his BP was 80mm Hg his ECG showed

AF and signs of resolving infrac.on /120.

Page 46: ˘ˇˆ˙ - Ain Shams Universitymed.asu.edu.eg/uploads/med/Student Affairs/IEE_IO__2.pdf• Lt. Lower lobe bronchiectesis • Recurrent pul. Embolism • Chronic broncitis (May 96)

Faculty Of Medicine - ASU

Student Union Scientific Committee (SUSC)

Examination Note - 4th year P a g e | 46

• Give 2 possible causes of the pain in the leg?

• What would be your further management?

• And suggestion of appropriate treatment?

5- A 16 old years boy with known insulin dependent diabetes mellitus was admitted to hospital in a lethargic

drowsy condi.on with history of vomi.ng polyuria and polydipsia 3 days before he was taking soluble insulin

10 units each morning his temperature was 36.6c pulse was 140/min BP100/60 mmHg heart lung and

abdomen clinically normal?

• What is the diagnosis?

• What are the possible precipitating factors?

• What is your immediate management?

6- An 18 years old female presented 4 weeks aVer a cold like illness with rapid onset of Weakness in the lower

and upper limbs associated with glove and stocking numbness of 4 limbs.

• What is the most possible diagnosis?

• What is the possible lines of treatment?

• What a serious complication this patient may have?

2001-5

1- Men.on 5 possible causes of prolonged fever how to suspect clinically

2- and how to plan for selected investigations for each of them?

3- Enumerate 10 cases of viral hepa..s?

4- Discuss briefly DD of unexplained somatic symptoms?

5- Men.on 4 causes of chronic diarrhea an discuss how to reach to a final diagnosis?

6- Men.on 5 causes of ascites and discuss the diagnos.c features of each of them?

7- What are the risk factors for coronary heart disease you know mention how to deal with the correctable

factors?

8- Give an account on hypothyroidism?

9- Men.on 4 groups of drugs used in the management of ac.ve rheumatoid arthri.s and what is principle

behind their therapeutic value?

10- Men.on 4 causes of generalized lymphadenopathy and the blood picture of each?

11- How do you explain the meaning and causes of following:

• Oliguria

• Low fixed urine specific gravity at 1010

• Proteinuria

12- Discuss how to determine the aetiology of hemiplegia?

11-2001

Read each case history carefully and give the diagnosis and answer the related

question.

Case I: (15 Marks)

A 35 years old woman presented with 6 months history of diarrhea, weight loss and palpita.on. She was seen

by a psychiatrist for feeling anxious and irritable and was given a tranquilizer without good response:

- What is the most important disease you have to exclude?

- Mention important clinical signs that can help you in its diagnosis?

- Mention important blood tests help you in its diagnosis?

- If the diagnosis proved to be correct how to manage?

Case II: (15 Marks)

Page 47: ˘ˇˆ˙ - Ain Shams Universitymed.asu.edu.eg/uploads/med/Student Affairs/IEE_IO__2.pdf• Lt. Lower lobe bronchiectesis • Recurrent pul. Embolism • Chronic broncitis (May 96)

Faculty Of Medicine - ASU

Student Union Scientific Committee (SUSC)

Examination Note - 4th year P a g e | 47

A 60 years old man complaining of increasing fa.gue excessive sweating and loss of weight. He was found

to be pale with multiple skin bruises and massive splenomegaly:

- What is the most likely diagnosis?

- What other clinical features to search for?

- How blood examination can help you in the diagnosis?

- How to confirm your diagnosis?

Case III: (15 Marks)

A 70 years old man presented with acute pulmonary oetema. BP was 170/100 and ECG showed elevated ST

segment in V1 to V6:

- What is the diagnosis of this case?

- What are the immediate lines of management?

- What is the diagnostic workup needed to investigate this condition?

Case IV (10 Marks)

Female pa.ent aged 20 years came complaining of fever with rigors of 5 days dura.on:

- Give 5 possibili.es for this condi.on.

- Men.on 4 inves.ga.ons that could help in the diagnosis.

- Propose the specific treatment for each possibility.

Case V: (10 Marks)

A 30 years old black female has a chest X-ray as a part of prenatal check up which showed the presence of

pilateral hilar masses:

- Give 3 possible causes.

- Mention how to reach the diagnosis.

- Propose the specific treatment for each possibility.

During evalua.on of hypertension a 54 years old woman was found to have a BUN of 142mg/dl and serum

crea.nine 3.2 mg/dl. She had no known history of renal disease or urinary infec.on. She has chronic headaches.

Her physical examina.on was normal. Her heamoglobin was 9.7g/dl and crea.nine clearance was 20ml/min:

- What is your diagnosis?

- What do you expect to find the kidney size by ultrasonography?

- What type of renal parenchymal disease (glomerular, interstitial or vascular) do you suspect?

- How to manage this case?

2002 -9

Paper I

Time allowed: 2 h

Answer all questions:

1- give an account of:

a) treatment of simple attack of asthma

b) treatment of status asthma.cus (10 marks)

2- give an account on:

a) symptoms & signs of myocardial infarction

b) investigations to diagnose myocardial infarction (10 marks)

3- give an account on:

a) clinical picture of typhoid fever

b) investigations to diagnose typhoid fever

c) treatment of non complicated cases of typhoid fever (10 marks)

4- give an account on:

a) clinical picture of myxoedema

Page 48: ˘ˇˆ˙ - Ain Shams Universitymed.asu.edu.eg/uploads/med/Student Affairs/IEE_IO__2.pdf• Lt. Lower lobe bronchiectesis • Recurrent pul. Embolism • Chronic broncitis (May 96)

Faculty Of Medicine - ASU

Student Union Scientific Committee (SUSC)

Examination Note - 4th year P a g e | 48

b) investigations of myxoedema

c) treatment of myxoedema (10 marks)

5- give an account on:

a) definition & criteria for diagnosis of nephrotic $

b) causes of nephrotic $ (10 marks)

6- give an account on:

a) causes & types of diarrhea. (6 marks)

b) treatment of intes.nal amoebiasis (4 marks)

7- - (10 marks)

a) enumerate the causes of megaloblastic anemia

b) diagnosis of megaloblastic anemia

c) treatment of megaloblastic anemia

8- - (10 marks)

a) how can you deal with a case of excitement in a medical ward?

b) Treatment of parkinsonism

9- give an account on the clinical picture of systemic lupus erythromatosis (10 marks)

Paper II

Time allowed: 2 h

Answer all questions:

1- a 54 years old female pa.ent came to the hospital with post-traumatic fructure femur. She was admitted

& received proper surgical treatment, analgesics & antibiotics. Few days later, the patient became febrile,

with cough & expectoration, & another antibiotic & antipyretics were added. The patient improver

dramatically, however 2 weeks aVer admission she developed oligurea & mild edema with skin rash &

arthralgia. Her urine analysis showed microscopic hematurea, mild proteinurea, increased WBCs &

increased eosinophils. CBC showed some eosinophilia. Her renal functions were impaired with serum

crea.nine 5.6 mg/dl & BUN 63 mg/dl.

a) what is the most likely diagnosis? (5 marks)

b) what is your differential diagnosis? (10 marks)

c) what investigations would you like to do to reach a diagnosis? (5 marks)

2- 50 years old diabetic male patient developed painful grouped vesicular lesions on the upper half of the

face confined strictly to the right side. He received proper treatment & improved. However, he had

severe bouts of stabbing pain at the site of the affected area even after recovery of the rash.

a) what is your diagnosis? (5 marks)

b) what is the treatment of this rash? (5 marks)

3- a 65 years old male pa.ent complained of ill health, fa.gue & lethargyover the last 4 months. He also had

spontaneous bruising. On examination he was pale & jaundiced. Few significant axillary & cervical lymph

nodes were palbable. Hepatosplenomegaly was evident. His CBC showed hemoglobin level of 8 mg/dl,

WBCs 50,000/cmm with 90% small lymphocytes, platelets were 40,000/cmm.

a) what is your diagnosis? (5 marks)

b) what is your differen.al diagnosis? (5 marks)

c) mention the possible causes of jaundice in this patient.

(5 marks)

4- a 50 years old female pa.ent known to have liver cirrhosis with +ve HBsAg since 10 years. During the last

2 months, she noticed increased fatigability & marked loss of weight. Abdominal examination showed

hard hepatomegaly & ascitis with audible bruit over the liver.

a) what is the most likely diagnosis? (5 marks)

b) what are the investigations you suggest to this patient?

(10 marks)

Page 49: ˘ˇˆ˙ - Ain Shams Universitymed.asu.edu.eg/uploads/med/Student Affairs/IEE_IO__2.pdf• Lt. Lower lobe bronchiectesis • Recurrent pul. Embolism • Chronic broncitis (May 96)

Faculty Of Medicine - ASU

Student Union Scientific Committee (SUSC)

Examination Note - 4th year P a g e | 49

5- a 25 years old female was presented to the emergency room with deteriora.on in the level of

consciousness. Her family gave a history that she had acute febrile illness 2 weeks before & quadriparesis

that started 3 days ago. On examina.on, she had central cyanosis with flaccid weakness of the four limbs.

a) what is your differen.al diagnosis? (5 marks)

b) what inves.ga.ons you need to seOle the diagnosis? (5 marks)

c) what emergency treatment you would like to be carried out to this pa.ent? (5 marks)

6- a 14 years old boy was brought to the hospital with rapid deteriora.on in the level of consciousness. His

family said that he had fever & sor throat few days ago. He had convulsions over the last day. On

examina.on his temperature was 39oc & his blood pressure was 80/40. there was neck rigidity & bilateral

+ve extensor plantar reflex.

a) what is your diagnosis? (3 marks)

b) what is your differen.al diagnosis? (4 marks)

c) what inves.ga.ons you would like to do to reach a final diagnosis? (4 marks)

d) what an.bio.cs would you like to give to this pa.ent? (4 marks)

7- a 40 years old female was admiOed to the hospital with mul.ple bone fractures following a car accident.

She was confined to bed for 1 week. She complained of sudden dyspnea & chest .ghtness after that

week. She gave no past history of medical importance apart from taking oral contraceptive pills.

Examina.on revealed central cyanosis. Her blood pressure was 80/60, & pulse was 120/min, regular.

There was epigastric pulsations as well as left parasternal pulsa.ons in 2nd , 3rd & 4th spaces. There was

audible systolic murmur in 2nd left space. Chest X-Ray showed only diminution of peripheral vascular

markings.

a) What is your diagnosis? (3 marks)

b) What is your differen.al diagnosis? (4 marks)

c) Men.on 3 inves.ga.ons to seOle the diagnosis. (4 marks)

d) What is the line of treatment you suggest? (4 marks)

8- a 12 years old underweight boy had fever for 2 days. Examina.on revealed a very tender swelling in his

right buttock that was diagnosed as a gluteal abscess. Next day the patient was very confused & his

mouth was dry. He had also acidotic breathing.

a) What is the likely diagnosis for the confusion? (5 marks)

b) What is your differen.al diagnosis? (5 marks)

c) What further tests do you like to perform to this patient to reach a diagnosis? (5 marks)

9-2003

Paper I

Time allowed: 2 h

Answer all questions: (90 marks)

1- Discuss diagnosis and treatment of a case of diarrhea.

2- Discuss causes, clinical picture, investigations and treatment of osteoporosis.

3- Discuss clinical picture and treatment of lobar pneumonia.

4- Discuss the clinical differentiation and treatment of different types of cerebrovascular strokes.

5- Mention in details the treatment of acute renal failure.

6- Discuss the possible causes of coma in a diabetic patient.

Paper II

Time allowed: 2 h

Answer all questions: (120 marks)

Page 50: ˘ˇˆ˙ - Ain Shams Universitymed.asu.edu.eg/uploads/med/Student Affairs/IEE_IO__2.pdf• Lt. Lower lobe bronchiectesis • Recurrent pul. Embolism • Chronic broncitis (May 96)

Faculty Of Medicine - ASU

Student Union Scientific Committee (SUSC)

Examination Note - 4th year P a g e | 50

1- A 60 ys old man was brought to the ER for impaired consciousness of 1 day dura.on. He had abdominal pain &

vomi.ng for 4 days. Exam. releaved a .nge of jaundice & ascites. He was delerious & his temp. was 38.4C .Chest

& heart was normal.

a) What is the most common cause of delerium?

b) What is the D.D?

c) What is the investigations U needto reach a final diagnosis?

d) What ttt would U prescribe?

2- A 35 ys old female complained of s.tching pain in the Rt side of the chest with hemoptysis for 2 days. She gave

a past history of SLE for the last 10 ys. She aborted twice last 2 ys. Exam. revealed swelling of the Rt leg &foot.

Temp. was 37.7C. Chest exam. revealed the presenceof Rt pleural rub.

a) In your opinion , what is the cause of her chest symptoms?

b) What is your D.D of her chest condition?

c) What is the relation of SLE to her recent events?

d) What future invest. would U suggest to this pt.?

3- A 50 ys old male visited his phisician for sense of fullness in his feet &.ghtness of his shoes that occured aVer

prolonged standing or walking at the end of the day. The pt. was diabe.c & hypertensive for the last 15 ys. He

also had ischemic Ht. Ds. for 2 ys &had laser Ot for his fundus for 2 ys. The pt. was under Ot using insulin,

Ca.Ch.blocker, alpha blocker, ACE inhibitor& nitrates. Exam. revealed that his BP was 170/110, he was

overweight, dyspnoeic& with slight L.L edema.

a) What are the possible causes of his L.L edema?

b) What are the investigations would U need to confirm the cause of this edema?

c) What is the management that U would suggest?

4- A 65 years old male complained of easy fa.gue and effort intolerance for the last 2 months. The pa.ent

denied any bleeding tendency or GIT symptoms apart from poor appetite and some weight loss. On examination

there was pallor and tenderness on the bones. Blood chemistry analysis revealed a serum creatinine of 3 mg/dl,

serum calcium was 12mg/dl, and phosphorus was 4 mg/dl, serum albumin was 3.4 gm/dl and serum globulin was

5.5 gm/dl. Serum potassium was 4 mEq/L and serum sodium was 134 mEq/L. CBC revealed hemoglobin of 8

gm/dl with normocytic normochromic RBCs. ESR was 140 in 1St hour.

a. What is your probable diagnosis?

b. What is your differential diagnosis?

c. What investigations would you suggest to reach a final diagnosis?

5- A 12 years old boy was admiOed with a 2-days history of fever, headache and vomiting Consciousness was

impaired. His temperature was 39.7°C pe.chae were present in conjunc.va and palatal mucosa. There was neck

stiffness with positive kernigs's sign. The pupil of right eye dilated and fixed and unresponsive to light. The eye

was deviated laterally and did not move vertically. Optic fundi were normal.

a. What is your most likely diagnosis ?

b. What is your differential diagnosis ?

c. What is the cranial nerve lesion ?

d. What investigations would you like to order ?

e. What treatment would you prescribe ?

6- A 50 years old male complained from compressing chest pain that increased on mild exer.on. He gave a past

history of diabetes mellitus, hypertension and mild renal impairment for which he was under conservative

Page 51: ˘ˇˆ˙ - Ain Shams Universitymed.asu.edu.eg/uploads/med/Student Affairs/IEE_IO__2.pdf• Lt. Lower lobe bronchiectesis • Recurrent pul. Embolism • Chronic broncitis (May 96)

Faculty Of Medicine - ASU

Student Union Scientific Committee (SUSC)

Examination Note - 4th year P a g e | 51

management. He was admitted by the cardiologist for coronary angioghraphy. Coronary stenting was done and

the pa.ent improved and leV the coronary care unit aVer 2 days . In the ward, the pa.ent started to complain of

dyspne. He was hemodynamically stable with a BP pf 150/100, temperature of 37°C, and pulse rate of 96/min,

regular . His urine output was 900 cc/day and urine examina.on showed (+++) proteinuria . His blood gases

showed metabolic acidosis .There were no new ECG changes.

a. What is your explanation for his dyspnoea and acidosis ?

b. What is your differential diagnosis ?

c. What investigations would you like to order to reach a definitive diagnosis ?

9- 2006

paper I B (Short essay Questions)

ALL QUESTIONS ARE TO BE ANSWERED:

1. Discuss diabe.c nephropathy. (15 Marks)

2. Describe the management of a case of haematemesis. (15 Marks)

3. Describe:

a- Clinical manifesta.on of thyrotoxic crisis. (7.5 Marks)

b- Treatment of Bronchial asthma during the aOach. (7.5 Marks)

4. Describe clinical manifesta.on and treatment of spontaneous pneumothorax. (15 Marks)

5. a- Describe the diagnosis and management of megaloblastic anemias. (7.5 Marks)

b- E.ology and management of Parkinson disease. (7.5 Marks)

6. Describe the most effec.ve drug treatment for each of the following including dose, dura.on of treatment.

a-Amoebic hepa..s. (3 Marks)

b-Tinea saginata infesta.on. (3 Marks)

c- Brucellosis (3 Marks)

d- Malaria:

i- During the attach. (2 Marks)

ii- Prophylaxis against further aOach. (2 Marks)

iii- Prophylaxis against infection in new comers to endemic

area. (2 Marks)

Paper II B (8 cases)

ALL CASES ARE TO BE ANSWERED:

1. A fourty years old obese lady had performed cholecystectomy for calcular gall bladder disease. She

complained of acute dyspnea and haemoptysis on the 8th day aVer opera.on.

a- What is the important possible diagnosis? (5 Marks)

b- Men.on the diagnos.c inves.ga.ons. (5 Marks)

c- What is the line of treatment? (5 Marks)

2. An 18 years male, presented with gradual progressive weakness of both lower limbs with diminished pain

sensation till the umblicus and localized tenderness over the mid-dorsal spine associated with night fever.

a- What is your provisional diagnosis? (4 Marks)

b- Choose ONE laboratory test to prove your diagnosis. (3 Marks)

Page 52: ˘ˇˆ˙ - Ain Shams Universitymed.asu.edu.eg/uploads/med/Student Affairs/IEE_IO__2.pdf• Lt. Lower lobe bronchiectesis • Recurrent pul. Embolism • Chronic broncitis (May 96)

Faculty Of Medicine - ASU

Student Union Scientific Committee (SUSC)

Examination Note - 4th year P a g e | 52

c- What is the best radiological imaging to confirm the diagnosis?

(4 Marks)

d- What is the line of treatment? (4 Marks)

3. A 42 years old male, smoker, hypertensive and diabe.c on regular medica.ons, presented with sudden

severe pain of his right ankle that started at night .clinical examina.on revealed B.P. 170/90 ,Temp.37.5c pulse

100/min his right ankle is swollen red,hotand markedly tender. WBC count 13000/mm and ESR 35mm/hr.

a- What is most likely diagnosis and differen.al diagnosis. (5 Marks)

b- Men.on inves.ga.ons to be done to confirm your diagnosis.(5 Marks)

c- What are the most important lines of therapy? (5 Marks)

4. A 28 years old leady with a past history of butterfly rashes and polyarthralgia is referred by the obstetric

department. When she was 21 years, she had a DVT. She has had three abor.ons at 16-18 weeks and a recent intra-

uterine fetal death at 30 weeks, followed by development of an extensive femoral vein thrombosis.

a- What is the diagnosis? 5 Marks)

b- What further tests should be done? (5 Marks)

c- What precau.ons should be advised in subsequent pregnancy. (5 Marks)

5. A 33 years old man presents to his doctor with painless lump on the right side of his neck for about two

months and seems to be enlarging. No recent throat infections, he lost about 8 kg in weight. The pa.ent has also

developed drench night sweats. His temp. is 37.8 c, palpable mass in the right supraclavicular fossa, firm in

consistency and about 3x4 cm. There are also lymph nodes 1-2 cm.

in diameter, palpable in both axillae and inguinal areas, cardiovascular and respiratory systems are normal. On

abdominal examina.on, there is a mass palpable 3 cm below the leV costal margin. The mass is dull to percussion

and it is impossible to palpate is upper edge. Neurological examination is free.

a- What is the likely diagnosis? (5 Marks)

b- What is the differen.al diagnosis? (5 Marks)

c- What investigation would you ask to support your diagnosis (5 Marks)

6. A 55 years old male started to feel epigastric pain both when hungry and aVer meals since 4 months. This was

accompanied by marked loss of appetite particularly to meat, which resulted in ten kg loss of weight at the end of

this period. Examination revealed marked pallor, tender epigastric region, and enlarged left supraclavicular lymph

node.

a- What is the diagnosis. (4 Marks)

b- What is the differen.al diagnosis. (4 Marks)

c- Inves.ga.ons would be done to prove your diagnosis. (4 Marks)

d- Treatment of the case based on your most probable diagnosis. (3 Marks)

7. A twelve years old boy complaining of anorexia, nausea, vomi.ng and mild rise of temperature. On the third

day he developed jaundice and dark urine discolouration.

a- What is your probable diagnosis? (3 Marks)

b- What is the cause of dark urine? (3 Marks)

c- What are the inves.ga.ons to confirm diagnosis? (3 Marks)

Three weeks later he improved, his appetite returned but unfortunately jaundice deepened, stools became

pale, urine darkened, itching and diarrhea occurred.

d- What is your new diagnosis? (3 Marks)

e- What inves.ga.ons would you ask again? (3 Marks)

8. A thirteen years old girl presented with severe headache, vomi.ng of four days duration. She looked pale,

with slightly puffy eyes and passage of dark urine. Her pulse was 55/min. and B.P 180/110.

Page 53: ˘ˇˆ˙ - Ain Shams Universitymed.asu.edu.eg/uploads/med/Student Affairs/IEE_IO__2.pdf• Lt. Lower lobe bronchiectesis • Recurrent pul. Embolism • Chronic broncitis (May 96)

Faculty Of Medicine - ASU

Student Union Scientific Committee (SUSC)

Examination Note - 4th year P a g e | 53

a- What is the possible diagnosis. (3 Marks)

b- Explain the etiology of her symptoms and sign. (4 Marks)

c- What inves.ga.ons would you ask for her. (4 Marks)

d- What are the sequel of her illness. (4 Marks)

Internal medicine 27 september 2007

Paper 1 – B

All ques.ons to be answered ( 15 marks each)

1- discuss causes of polyurea

2- give short account on gastrointestinal causes of chest pain

3- write short account on management of diabetic ketoacidosis

4- write short account on diagnosis and management of idiopathic thrmobocytopenic purpura

5- give short account on complications of acute hepatitis

6- give short account on secondary hypertension

Internal medicine 29 september 2007

Paper 2-B

Answer all questions

1- a 16- year- old Egyptian male patient was going to travel to Kenya . On the second day of his arrival to Kenya,

he developed jaundice and dark urine and returned back to Cairo. On examination : he had jaundice , pulse rate is

100/ min , B.P 110/70 mmHg and he had not felt par.culary ill and there is no fever .

investigations revealed :

haemoglobin : 7 g/l , mean corpuscular volume : 100 fl , toltal leucocy.c count : 5,000 / ml3 , platelet count :

300,000 / ml3

E.S.R : 34 mm/hr , haptoglobin : 12 mg % ( Normal 50 ; 200 mg % )

Blood film : extensive Heinz body forma.on , 11% re.culocytes

What the most likely diagnosis ? “ 10 marks”

Discuss differen.al diagnosis ? “ 10 marks”

2- A 54-year- old patient presents with a hoarse voice and an increasing inability to climb stairs and get out of his

chair . T he weakness worsens throughout the day .

investigations revealed :

Page 54: ˘ˇˆ˙ - Ain Shams Universitymed.asu.edu.eg/uploads/med/Student Affairs/IEE_IO__2.pdf• Lt. Lower lobe bronchiectesis • Recurrent pul. Embolism • Chronic broncitis (May 96)

Faculty Of Medicine - ASU

Student Union Scientific Committee (SUSC)

Examination Note - 4th year P a g e | 54

Na : 144 mmol/l , K: 1.9 mmol/l , urea 4.7 mmol/l ( normal up to 5.1 mmol/l) , Bicarnoate 34 mmol/l , Glucose 15

mmol/l ( Notmal up to 58 mmol/l ) , Serum cortisol is high

What is the most likely diagnosis ? “ 5 marks”

How can you confirm the diagnosis ? “ 5 marks”

What is the cause of his hoarse voice ? “ 5 marks”

Give 2 causes of his weakness ? “ 5 marks”

3- A15- year- old boy complaining of colicky abdominal pain , abdominal distension and vomiting with absent

flatus .

Two days before he had similar pains with mild bleeding per rectum . He had a rash on his buttocks and ankles.

Investigation results were as follows :

Urine : blood +++ & proteins +++ .

E.S.R78 mm/h , Haemoglobin : 9.5 g dl ( microcy.c hypochromic) , platelets : 400,000/ ml3 . Abdominal X-ray

showed fluid levels .

What the most likely diagnosis ? “ 5 marks”

What is the cause of his abdominal symptoms ? “ 10 marks “

What kind of rash will he have ? “ 5 marks”

4- A65-year-old woman with a history of Rheumatoid arthri.s since 20 year . He gave a 2- month history of

peripheral oedema and breathlessness.

On examination she was pale & had classic rheumatoid deformity.

There was oedema up to the sacrum , as well as ascites & Hepatosplenomegaly.

Investigations revealed :

Haemoglobin : 9.5 g/dl ( normocy.c normochromic), Urea: 80 mg % , crea.nine : 3 mg % , Albumin : 1.9 g/l ,

cholesterol : 320 mg % ( Normal up to 210 mg% ).

Urinary proteins : 6 gm/l , Chest X-ray : Cardiomegaly.

What is the most probable diagnosis ? “ 10 marks “

How can you confirm the diagnosis ? “ 10 marks “

5- A 60-year-old man was admitted with acute dysnea.

On Examina.on his pulse was 120 beat / min and irregular , BP 140/70 mmHg , JVP + 6 cm and there were

bilateral crackels on chest auscultation . He had grey coloration of his skin , gynecomastia and stigmata of chronic

liver disease . There was evidence of hepatomegaly , mild splenomegaly but no ascites . He was non smoker but

received Frusemide 40 mg/d , Amiodarone 200mg/d on the last 6 months.

Inves.ga.ons revealed : Sodium : 138 mmol/l , Potassium : 4mmol/l

Urea : 30mg % , Random sugar : 300mg% , Chest X-ray : evidence of pulmonary edema

ECG : Atrial fibrillation , Echo cardiograghy : Restrictive Cardiomyopathy.

Page 55: ˘ˇˆ˙ - Ain Shams Universitymed.asu.edu.eg/uploads/med/Student Affairs/IEE_IO__2.pdf• Lt. Lower lobe bronchiectesis • Recurrent pul. Embolism • Chronic broncitis (May 96)

Faculty Of Medicine - ASU

Student Union Scientific Committee (SUSC)

Examination Note - 4th year P a g e | 55

What is the underlying diagnosis ? “ 5 marks”

Why he has pulmonary edema ? “ 5 marks”

Give one important complica.on ? “ 5 marks”

What is your plane for management ? “ 5 marks”

6- A60-year-old man complained of weakness and stiffness in his legs . Past history was irrelevant apart from

abdominal opera.on since 20 years for perforated peptic ulcer . Neurological Examination Revealed : Bilateral

extesor plantar reflex , exaggerated knee reflex but lost ankles in both legs with impaired deep sensation in

both legs . Investiagtions revealed :

Haemoglobin: 6g/dl , White cell count : 2-3 X 10/l , Platelet : 90 X 10 /l . Bilirubin : 50 umol/l ( Normal up to 14

umol/l ) , Lac.c dehydrogenase : 1650 u/l ( Normal 88: 230 u/l )

What is your diagnosis ? “ 10 marks”

What are further investigations do you recommend ? “ 10 marks”

September 2008

Case 1:

20 years old man was found unconscious in his bathroom by his mother. He suffered from type I diabetes mellitus

for 10 years and his diabe.c control is poor.

1- Enumerate different causes which can cause this condi.on? (5marks)

Over the past 10 days he has developed produc.ve cough of greenish sputum. He stopped taking insulin for 2

days because he has lost his appetite and he was feeling nauseated and has not been eating.

2- What would your differential diagnosis include aBer this history?(3marks)

3- What is your comment on stopping insulin treatment in this pa.ent? (1 mark)

O/E he is clinically dehydrated, pulse 116/m, regular with low volume, Bp 90/60 mmHg

, respiratory rate 30/min with deep signing breath. There are no focal neurological signs. Auscultation of the chest

revealed upper right side consolidation and bronchial breathing.

4- How does examina.on narrow down your DD? (3marks)

Inves.ga.ons revealed CBC shows WBCs 25x109/l, Na 139mg/l, k 5.8 meq/l, chloride 95 meq/l, bicarbonate 10

meq/l, urea 50 mg/dl, crea.nine 1.4mg/dl, chest X ray shows right upper lobe homogenous opacity.

5- How do you interpret the inves.ga.ons? (4marks) interpret=explain

Arterial blood gases shows pH 6.28, pCo2 mmHg, pO2 90mmHg

6- How do you interpret these changes? (1mark)

7- Enumerate other precipita.ng factors for diabe.c ketoacidosis?(5marks)

8- Enumerate complica.ons of DKA?(3marks)

9- How would you treat this pa.ent? Enumerate only lines of treatment. (5marks)

Page 56: ˘ˇˆ˙ - Ain Shams Universitymed.asu.edu.eg/uploads/med/Student Affairs/IEE_IO__2.pdf• Lt. Lower lobe bronchiectesis • Recurrent pul. Embolism • Chronic broncitis (May 96)

Faculty Of Medicine - ASU

Student Union Scientific Committee (SUSC)

Examination Note - 4th year P a g e | 56

Case 2

A 35 years old man is referred to you because of hypertension. His hypertension is newly diagnosed having been

detected at rou.ne medical checkup. His blood pressure has been around 170/110 mmHg and has not been easily

controlled.

1- Enumerate different causes of treatment failure in this pa.ent (4marks)

He also complained of thirst and polyurea as well as general weakness and muscle crumps. There is no significant

past medical or family history. He is on nifedipin 20mg twice daily.

2- What does this complain add to your DD? (4marks)

O/E he looks well his blood presuure 170/110mmHg, pulse 84/m regular, JVP normal. Chest, heart, abdomen, CNS

is normal. Fundus shows arteriovenous nipping and silver wiring.

3- Has this examina.on narrowed down your DD? (2marks)

Investigations revealed normal CBC, Na 149 meq/l, K 2.2meq/l, bicarbonate 32 meq/l and urea 10.9mg/dl,

crea.nine 0.9 mg/dl, glucose 100 mg/dl, urine analysis normal, urinary k 50 meq in 24 hours (high), renal

ultrasound shows normal sized kidneys.

4- How can you interpret the results of the inves.ga.ons? (6marks)

5- Enumerate the endocrinal causes of 2nd

ry hypertension (5marks)

6- Enumerate the different causes of hypokalemia and increase K in urine (4marks)

7- How do you choose an.hypertensive drugs? (5 marks)

Case 3

A 40 years old man complaining of fatigue for the last few weeks. This was followed by sore throat for one week

duration and bleeding gums and rash on his legs.

1- What would be your differen.al diagnosis?(5marks)

On examina.on, temp 37°C, BP 120/80 mmHg, pulse 90/m regular. Pallor congested red throat, petechial rash on

his legs. There was no lymphadenopathy and his spleen and liver were not enlarged.

2- Has the examina.on narrowed down your differen.al diagnosis? (2marks)

The inves.ga.ons revealed Hb 9gm/dl, MCV 87fl, WBCs 34.5 x109 /l neutrophils 1x10/l lymphocytes 34 x109/l,

platelets 20x109/l, LDH 1000 IU/L, crea.nine is normal, uric acid 9mg /dl. Blood film showed many blast cell.

3- How can lab findings explain symptoms? (3marks)

4- Discuss definition of purpura, petichiae, echhymosis. (3marks)

5- What are other inves.ga.ons you order to reach a final diagnosis? (3 marks)

6- What is the explana.on of high LDH and serum uric acid? (1mark)

Page 57: ˘ˇˆ˙ - Ain Shams Universitymed.asu.edu.eg/uploads/med/Student Affairs/IEE_IO__2.pdf• Lt. Lower lobe bronchiectesis • Recurrent pul. Embolism • Chronic broncitis (May 96)

Faculty Of Medicine - ASU

Student Union Scientific Committee (SUSC)

Examination Note - 4th year P a g e | 57

7- How can you explain the presence of acute pharingitis in abcense of usual signs of infections (2marks)

8- Why is it important to diagnose the specific type of this disease? 2marks)

9- Men.on the general suppor.ve measure to be given to this pa.ent? (3marks)

10- What is the treatment strategy in acute leukemia? (2marks)

11- Men.on 2 drugs used n remission induc.on and 2 drugs used for consolida.on in acute myeloblas.c

leukemia? What are their side effects? (3maks)

Case 4

A 50 years old man complaining of fever, increasing abdominal girth, yellow kin and drowsiness. He has history of

chronic hepatitis C virus infection complicated by cirrhosis, portal hypertension and ascites. He used frusimide

and spironolactone. Ten days before admission he began to gain weight and to notice persistent increase in his

abdominal girth despite continuing to take his medication. This morning he has difficulty to arouse and appeared

disoriented. His wife denies that he has complained of chills, sweats, headache, chest pain, cough or haematemsis

or melena.

1- What would your differen.al diagnosis be? (3marks)

O/E temp 39°C, Bp 110/60 mmHg, pulse 100/m regular, jaundice, sleepy but arousable.

He had gynicomastia, palmer erythema and asterixis. Abdomen is diffusely tender to palpitation with ascites. The

liver and spleen is mildly enlarged.

2- Has the examination narrowed down your differen.al diagnosis?(3marks)

Inves.ga.ons revealed: a 129 meq/L, k 3 meq/l, crea.nine 1.8 mg/dl, albumin 1.5 gm/l, bilirubin 3mg/dl, Pt 20

seconds, WBCs 18x10 9/l, platelets 5 x10 9/dl, urine Na 10 meq/l

3- Does this narrow your differential diagnosis? (3marks)

4- What other inves.ga.ons you will order to confirm the diagnosis? 93marks)

5- What is the mechanism of high PT in this pa.ent? (2marks)

6- Enumerate other possible complica.ons that this pa.ent can develop.(4marks)

7- What is the cause of hyponatremia in this pa.ent? (2marks)

8- In analyzing acsitic fluid, what are the lab parameter you ordered. What is the serum ascites albumin

gradient and what is its value? (4marks)

9- Enumerate othere precipita.ng factors of hepa.c encephalopathy? (3 marks)

10- How to treat this pa.ent? Enumerate line of treatment only. (3marks)

Case 5

A 42 year old man is brought to the emergency room aVer sudden severe retrosternal chest pain unrelated to

effort and was not relieved by sublingual nitroglycerine administration. He has never had symptoms like this

before. His only medical history is hypertension for which he takes enlapril. He does not smoke and is usually

physically active.

1- What would your differen.al diagnosis be from this history? (6 marks)

Page 58: ˘ˇˆ˙ - Ain Shams Universitymed.asu.edu.eg/uploads/med/Student Affairs/IEE_IO__2.pdf• Lt. Lower lobe bronchiectesis • Recurrent pul. Embolism • Chronic broncitis (May 96)

Faculty Of Medicine - ASU

Student Union Scientific Committee (SUSC)

Examination Note - 4th year P a g e | 58

In the emergency room he started to feel pain radiating to the back with inability to move his legs. On

examina.on: pulse 96/m regular, there was difficulty feeling the pulse in the leV arm but it was easy to find on

the right. Bp in the leV arm was 90/60mmHg, and in the right arm 160/ 100 mmHg. The pulse in the right groin

and leg were impalpable.

2- How would these signs narrow your differen.al diagnosis? (4marks)

On neurological examination, there were absent reflexes on both lower limbs, including planter reflexes. Flaccid

paralysis was on all movement, absent pain and temperature sensation but normal touch and vibration and joint

posi.on sensa.on. The lost pain and temperature sensa.on extend on his trunk up to the level of T11

dermatome. The bladder was distended and the upper edge was palpable at the level of umbilicus.

3- Where is the site of lesion, why? (4marks)

4- What is the possible spinal cord segment involved, why? (4marks)

5- What is the underlying cause of this lesion? (4marks)

6- What is the neurological tract affected? (4marks)

7- Draw a sec.on in the spinal cord showing the incolved area and what is its blood supply? (4marks)

Case 6

A 15 years old boy had complained for two days of fever and increasingly severe headache. During the next 24

hours, hs condition deteriorated and he became drowsy and disoriented. He vomited several times small

quantities of clear fluid.

1- What is the differen.al diagnosis from the history? (5marks)

O/E temp 39.5°C, he was delirious. A macular rash was present on his trunk and purpura were noted on his arms

and legs. Pulse 130/m, Bp 90/60 mmHg. There was marked neck s.ffness and papilloedema. Chest, heart and

Abdomen were normal.

2- How did the examina.on narrow down your differen.al diagnosis? (4marks)

Inves.ga.ons revealed Hb 13.5g/dl, WBCs 30x10 9/l, polymorph 28 x10 9/l, platelets 10x10 9/l. chest X ray, skull X

ray are normal.

3- How do you interpret these results? ( 2marks)

4- What other inves.ga.ons would you order to reach your final diagnosis? (2marks)

5- What are the expected findings in these inves.ga.ons? (4marks)

6- What is the possible organism responsible for the present illness? (2marks)

7- What are the orther organisms that can lead to similar presenta.on? (3marks)

8- Discuss the following: asep.c meningi.s, xanthochromia (2marks)

9- What is the line of treatment of this pa.ent? (4marks)

10- What is the prophylaxis that you should give to the pa.ent’s family (2marks)

Case 7:

Page 59: ˘ˇˆ˙ - Ain Shams Universitymed.asu.edu.eg/uploads/med/Student Affairs/IEE_IO__2.pdf• Lt. Lower lobe bronchiectesis • Recurrent pul. Embolism • Chronic broncitis (May 96)

Faculty Of Medicine - ASU

Student Union Scientific Committee (SUSC)

Examination Note - 4th year P a g e | 59

A 58 years old smoker has noted worsening shortness of breath with minimal exercise and the onset of dyspnea

at rest. He reports a productive cough with yellowish sputum every morning throughout the year. He denies chest

pain, fever and chills.

1- What would your differen.al diagnosis be for this history? (5 marks)

O/E he was centrally cyanosed, obese. There were engorged neck veins 12 cm above sterna angle. He had

bounding pulse 112/m regular and flapping tremors. He has edema in both legs extending to the scrotum. The

liver was enlarged 20 cm liver span in MCL, rounded edge and tender. Heart examina.on showed accentuated

second pulmonary sound, s1 normal, s4 audible gallop. Chest examina.on revealed lower chest retrac.on and

bilateral wheezes and rhonci. The anteroposterior diameter of the chest appears increased. He had a flame

shaped subconjunctival hemorrhage in the right eye and papilloedema in both eyes.

Inves.ga.ons revealed Hb 17.7g/dl, WBCs 15x109/l, ESR 32 mm, electrolytes, kidney and liver func.ons are

normal.

2- What would your differen.al diagnosis be aBer you know the examina.on and inves.ga.on? (5marks0

3- What is the mechanism of the high level haemoglobin? (2marks0

4- Men.on 4 addi.onal inves.ga.ons necessary for evalua.on of this pa.ent? (2marks)

5- Write the results of these inves.ga.ons. (2marks)

6- What are the signs of respiratory failure on this pa.ent? (4marks)

7- What are the criteria of corpulmonale in this pa.ent? (4marks)

8- What illness is included in the family of COPD?(1mark)

What is the treatment op.on for acute COPD exacerba.on? (5marks)

2009

Answer all questions. GOOD LUCK

Case no.1

A 14 years old boy was admiOed to the hospital for evalua.on of a prolonged fever. His mother says that her son

had rheuma.c fever 5 year ago

A) Enumerate 5 major criteria to diagnose rheuma.c fever (5 marks)

On examina.on the pa.ent was oriented, pale, temperature 38°C. Pulse 110 beats per minute and

regular blood pressure 110/70 mmHg. A pan systolic murmur was heard at the apex radia.ng to the

axilla. Abdominal examination showed mild splenomegaly.

B) What is the most probable cause of his prolonged fever? ( 2marks)

C) Enumerate 5 cardiac diseases that can predispose to this condi.on? (5marks)

His laboratory results showed: HB 10gm/dl, normocy.c normochromic anemia, WBC 15x109 (80% polymorphs),

ESR 60 mm in the first hour, normal platelet count. Renal and liver func.ons were normal. Blood culture was

positive for bacteria.

D) What are further inves.ga.on required and why? (8marks)

E) Enumerate 5 causa.ve organisms for this condi.on. (5marks)

Page 60: ˘ˇˆ˙ - Ain Shams Universitymed.asu.edu.eg/uploads/med/Student Affairs/IEE_IO__2.pdf• Lt. Lower lobe bronchiectesis • Recurrent pul. Embolism • Chronic broncitis (May 96)

Faculty Of Medicine - ASU

Student Union Scientific Committee (SUSC)

Examination Note - 4th year P a g e | 60

F) Enumerate the main antibiotics used to treat five different causative organisms for this condition.

(5marks)

Case no.2

A 37 male pa.ent no.ced darkening of the skin of his face and hands. He was also complaining of easy fa.gue,

dizziness on standing for the last 4 months. He also complains of occasional bouts of nausea and vomi.ng. He lost

4 kg of his weight over the last 6 months.

O/E: Bp: 90/60, tongue is dry; skin was pigmented with some areas of vi.lig. Buccal mucous membrane showed

hyper pigmented patches.

A) What is your diagnosis? (2marks)

B) Enumerate five causes for this disease (5marks)

C) What are the blood tests necessary to reach the diagnosis? (5marks)

D) Enumerate the main lines of treatment of this pa.ent (10 marks)

E) Enumerate 5 endocrinal diseases that may be associated with skin hyper pigmenta.on or hypo

pigmenta.on. (5marks)

F) Enumerate 5 endocrinal diseases that may cause weight loss. (5marks)

Case no.3

A 25 year old male came to the emergency room with low back pain and s.ffness of insidious onset for the last 3

years. His pain is worse at night and relieved with activity. One year previously he was diagnose as having acute

iritis and treated with eye drops. Clinical examination revealed limitation of lumbar spine movements in all

direc.ons. CBC was normal, and ESR was 50 mm/ 1st hr.

A) What is the most likely diagnosis? (2marks)

B) What are the radiologic investigations needed to confirm your diagnosis.(2marks)

C) What are the laboratory tests needed to confirm your diagnosis? (2marks)

D) Enumerate the main lines of treatment for this case. (5marks)

E) Enumerate 3 diseases that may cause low back pain. (3marks)

The patient wanted Tramadol injection (synthetic opioid analgesic) to relieve his pain. He said that he takes it

frequently when pain is severe. The patient’s history reveals frequent visits to the emergency room for joint pain

seeking Tramadol injection in particular with increasing dose needed to relief pains. This time the physician

refused to give him the injection telling that he might be Tramadol dependent. Shortly, the patient experienced

severe pain, sweating, diarrhea and depressed mood.

F) What is the diagnosis of this patient insisting to have Tramadol? What criteria that verifies your answer?

(4 marks)

G) What are the precautions that should be taken in prescribing opiate analgesics? (4 marks)

H) How would you treat this case? (4 marks)

I) When would you refer this pa.ent to a psychiatry specialist? (4 marks)

Case no.4

Page 61: ˘ˇˆ˙ - Ain Shams Universitymed.asu.edu.eg/uploads/med/Student Affairs/IEE_IO__2.pdf• Lt. Lower lobe bronchiectesis • Recurrent pul. Embolism • Chronic broncitis (May 96)

Faculty Of Medicine - ASU

Student Union Scientific Committee (SUSC)

Examination Note - 4th year P a g e | 61

A 19 year old female complains of a purpuric rash on her ankles and shins of .bia and easy bruising for 10 days.

The rash is not itchy or painful. She denies recent contact with new soaps or detergents. Bruises occur also on her

arms and sides and are unrelated to trauma. She reports nosebleeds, gum bleeding and unusually heavy one

week ago. She had an upper respiratory infec.on 2 weeks ago, which has now resolved. On physical examina.on

she has no lymphadenopathy or hepatosplenomegalty. Her stool is positive for occult blood.

A) What is your diagnosis? (4marks)

Her complete blood count shows:

Patient Normal

WBC (x103/mm3) 6.0 4.3-10

Hemoglobin (gm/dl) 13.1 12-16

Hematocrit (%) 39 38-50

RBC (x 106/mm3) 4.7 4.2-5.4

MCV (fL) 86 78-96

Platelet Count (x103/mm3) 13 150-450

There are no abnormalities of other cell lines.

PT and PTT:

• PT (Prothrombin .me) =11.6 sec ( normal 10.4-12.8 sec)

• PTT (par.al thromboplas.n .me)= 32 sec ( normal 24-36 sec)

B) Enumerate 5 causes for decreased platelet produc.on. (5marks)

C) Enumerate 5 causes of immune mediated platelet destruc.on. (5 marks)

D) Enumerate 3 more tests that may help you to reach the diagnosis. (3 marks)

E) Would you start platelet transfusion for this pa.ent? Why? (3 marks)

F) Enumerate the main lines of treatment for this pa.ent. (10 marks)

Case no.5

A 65 year old woman presents to the emergency room with a diminished urine output for the last 24 hours. She

gave a history of nausea, vomi.ng and watery diarrhea for the last 3 days. She has no previous history of renal

disease. Her temperature is 37°C, blood pressure is 90/60 mm Hg and pulse is 110 beats per minute. The physical

examination showed the presence of decreased skin turgor, dry mucosal membranes, flat neck veins and no

edema of lower limbs. Her lungs are clear and the cardiac findings are normal.

The following laboratory data are obtained: haematocrit 50.2% , white blood cell count 7, 500/mm3 , platelets

312,000/ mm3 , hemoglobin 12gm/dl , soduim146 mEq/L, potassium 4.1mEq/L, bicarbonate 13mEq/L, PH of 7.12,

Page 62: ˘ˇˆ˙ - Ain Shams Universitymed.asu.edu.eg/uploads/med/Student Affairs/IEE_IO__2.pdf• Lt. Lower lobe bronchiectesis • Recurrent pul. Embolism • Chronic broncitis (May 96)

Faculty Of Medicine - ASU

Student Union Scientific Committee (SUSC)

Examination Note - 4th year P a g e | 62

blood urea 190mg/dl and crea.nine 4.6 mg/dl. Urine analysis reveals a pH of 5, a specific gravity of 1010 and ++

casts . Abdominal ultrasonography reveals normal kidneys with no hydronephrosis or increased echogenicity.

A) What is your diagnosis? (2 marks)

B) Enumerate 5 laboratory tests that differen.ate between pre renal and renal causes of this disease.

(5marks)

C) What is the significance of the hematocrit value in this pa.ent? ( 2marks)

D) What is the average normal ra.on between blood urea and serum crea.nine? (2marks)

E) Enumerate the main lines of treatment of this pa.ent (10 marks)

F) Would you give diuretics for this patient? (2marks)

G) Enumerate the main indica.ons of dialysis in this disease. (5marks)

H) Enumerate 3 causes of death from this disease. (2marks)

Case no.6

A 26 year old woman known to have bronchial asthma for many years presents to the emergency room

complaining of shortness of breath, chest, wheezes, cough with yellow green sputum, of 5 days dura.on. Her

symptoms began after an upper respiratory tract infection that has manifested as low grade fever, rhinorrhea

with postnasal discharge, and nasal congestion.

A) What is the cause of exacerba.on of asthma in this pa.ent? (2marks)

She reports poor sleep quality for the last 2 days because of severe coughing and shortness of breath. Her

physical examination reveals that she is sweating and unable to speak in sentences. Her vital signs reveal a

respiratory rate of 30 breaths per minute, a heart rate of 120 beats per minute, a temperature of 37.8°C. there

was a supraclavicular retraction with inspiration, diffusely diminished breath sounds with scattered, high pitched

inspiratory and expiratory wheezes.

B) What are the bad prognos.c signs in this pa.ent? (3marks)

Spirometry showed a forced expiratory volume (FEV1) of 30% of predicted value.

C) What is the value of spirometry in this pa.ent? (2marks)

On the second day of hospitalization, a palpable subcutaneous crepitation was palpated over her anterior thorax.

D) What is the cause of the subcutaneous crepitation in this patient? And how to manage it? (4marks)

E) What is the value of chest X-ray for this pa.ent? ( 3marks)

F) What is the value of arterial blood gas measurements in this pa.ent? (2marks)

G) Discuss the main lines of treatment for acute severe asthma. (10marks)

H) Enumerate 2 drugs you advice the pa.ent to avoid aVer discharge from hospital. (2marks)

I) Would you prescribe an an.bio.c drug for this pa.ent and why? (2marks)

Case no.7

A 50 year old man has recurrent severe epigastric pain for the last several years. Antacids gave him symptomatic

relief. The most recent aOack began 1 week ago and has not responded completely to antacids. The pain now

wakes him up at night. He smokes one pack of cigarettes per day and he takes aspirin several times a week. His

Page 63: ˘ˇˆ˙ - Ain Shams Universitymed.asu.edu.eg/uploads/med/Student Affairs/IEE_IO__2.pdf• Lt. Lower lobe bronchiectesis • Recurrent pul. Embolism • Chronic broncitis (May 96)

Faculty Of Medicine - ASU

Student Union Scientific Committee (SUSC)

Examination Note - 4th year P a g e | 63

family history is unremarkable. Physical examination reveals moderate epigastric tenderness without evidence of

a mass. The stool is soft and black.

A) Enumerate 3 major risk factors for pep.c ulcer disease. (3marks)

B) Enumerate 10 other condi.ons that may cause epigastric pain (5marks)

C) What are the diagnos.c tests for H.pylori associated pep.c ulcer disease? (5 marks)

D) Discuss the role of NSAIDS in the pathogenesis of pep.c ulcer (5marks)

E) How would you treat H. pylori? ( 5 marks)

F) Enumerate 6 causes that makes pep.c ulcer refractory to medical treatment. (6marks)

G) What caused the soV black stools in this pa.ent? (1mark)

December 2009

Answer all ques.ons. (Each case 30 marks) GOOD LUCK

Case no.1

A 20 year old man was found unconscious. He suffered from type I diabetes mellitus for 10 years and his diabe.c

control is poor.

Over the past 10 days he has developed cough produc.ve of greenish sputum. He stopped taking insulin for 2

days because he has lost his appetite and he was feeling nauseated and has not been eating.

On examination he is clinically dehydrated, pulse 112/m regular with low volume. Bp 90/60 mmHg, respiratory

rate 28/m with deep signing breath. There are no focal neurological signs, ausculta.on of the chest revealed

upper right side consolidation and bronchial breathing.

Investigations revealed CBC shows WBCs 25x109 /l RBC 350mg/dl, Na 133mg/L, K 5.7 mEq/L, chloride 95 mEq/l,

bicarbonate 11 mEq/l, urea 50 mg/dl, crea.nine 14 mg/dl, chest X-ray shows right upper lobe homogenous

opacity.

Arterial blood gases shows pH 6.28, pCo2 20 mmHg pO2 90mmHg.

A) Enumerate different causes of coma in a diabetic patient?

B) What are the criteria for diagnosis of diabetic ketoacidosis?

C) Enumerate the precipitating factor for diabetic ketoacidosis?

D) How to you interpret the results of blood gases and comment on the anion gap.

E) Enumerate the complication of diabetic ketoacidosis

F) How will you treat this patient in hospital and on discharge?

Case no.2

A 49 year old female was admiOed to the hospital with multiple bones fractures following a car accident. She was

confined to bed for 1 week. She complained of sudden dyspnea and chest .ghtness aVer that week. She gave no

past history of medical importance apart from taking oral contraceptive pills.

Page 64: ˘ˇˆ˙ - Ain Shams Universitymed.asu.edu.eg/uploads/med/Student Affairs/IEE_IO__2.pdf• Lt. Lower lobe bronchiectesis • Recurrent pul. Embolism • Chronic broncitis (May 96)

Faculty Of Medicine - ASU

Student Union Scientific Committee (SUSC)

Examination Note - 4th year P a g e | 64

Examination: revealed central cyanosis. Her blood pressure was 80/60 and pulse was 120/min, regular, There was

an epigastric pulsa.on as well as leV parasternal pulsa.ons in 2nd, 3rd and 4th spaces. There was audible systolic

murmur in 2nd left space, chest X-ray showed diminution of peripheral vascular markings.

A) What is your diagnosis?

B) What is your differential diagnosis?

C) Men.on 3 inves.ga.ons to verify the diagnosis?

D) What is the line of treatment and support?

E) How to follow up the patient on the treatment therapy?

F) Mention causes of thrombophilia

Case no.3

A 24 year old women presents to an endocrine clinic with a 4 years history of ammenorrhea progressive weight

gain (10 kg during the past year, fa.gue, bone aches and difficulty in climbing stairs and rising from a deep chair.

The lady mentions that her body shape and increases in wait circumference. Her face has become hairy with acne

and also noticed that she bruises easily, she finds it difficult to concentrate, experiences headache, and feels

depressed. She denies ever having taken any drugs. Her general practitioner has found that her blood pressure

was increased on several occasions.

On examination: the pa.ent looks plethoric as well as mild hirsu.sm, her blood pressure is 170/100 mmHg. The

lady is unable to rise from a squatting position

On investigation: serum potassium 2.1 mmol/L, fas.ng blood glucose 149 mg/dl

A) What is the most likely diagnosis?

B) How can you confirm the diagnosis?

C) What is the explanation of the clinical symptoms?

D) What are the main lines of treatment?

E) Mention causes of obesity?

F) Discuss complications of morbid obesity.

Case no.4

A 48 year old man presents to the emergency room with difficulty of breath that followed weakness of his legs

and later hands. Four weeks earlier he had symptoms of an upper respiratory tract infection. Four days before

admission he had a feeling that there was tingling and numbness of both hands and feet. His bowels and bladder

are functioning normally. He has no past history of diabetes mellitus or any significant past medical history. He

neither smokes nor drinks alcohol and is taking no medications.

On examination: his pulse 104 beat/min, blood pressure 140/90 mmHg. The pa.ent is cyanosed, cardiac and

abdominal systems are normal. Neurological examination showed weakness of both lower limbs and upper limbs,

more proximally impaired, pin prick sensation over hands and feet, ankle reflexes are lost, cranial nerves are

intact

On investigations: cerebro spinal fluid examina.on showed pressure 170mm(<200mm) CSF protein3.4g/L (<0.4

g/L) leukocyte 5/ml (<5/ml)

Page 65: ˘ˇˆ˙ - Ain Shams Universitymed.asu.edu.eg/uploads/med/Student Affairs/IEE_IO__2.pdf• Lt. Lower lobe bronchiectesis • Recurrent pul. Embolism • Chronic broncitis (May 96)

Faculty Of Medicine - ASU

Student Union Scientific Committee (SUSC)

Examination Note - 4th year P a g e | 65

A) What is the most likely diagnosis?

B) What is the differential diagnosis?

C) What is the emergency treatment of this case?

D) What are further lines of treatment?

E) Mention causes of peripheral neuropathy?

F) What are the investigations required in a patient with peripheral neuropathy?

Case no.5

A 41 year old house wife complains of generalized pruri.us for 11 months, with a steady weight. 1 week ago she

accidently discovered yellowish discoloration of her conjunctivae, she also mentions recurrent bleeding from her

gums on brushing her teeth. No history of skin rash or arthralgia, she does not smoke or drink alcohol and was on

no medication. No other members of her family had experienced itching.

On examination: she was jaundiced. There was no lymphadenopathy, there were scattered scratch marks,

cardiovascular and respiratory systems are normal apart pulse rate 58 beat/min. The liver was palpable three

fingers breadths below the costal margin. No other masses were palpable and the spleen could not be felt.

Inves.ga.ons: HB 12.5 gm/dl, AST and ALT 125 &127/L respec.vely (normal up to 45), alkaline phosphatase 600/l

(N 100), albumin 3.7gm/dl, serum bilirubin 3.9 mmg/dl, LHS 100mg/dl, PT is 19 seconds, chest x-ray is normal,

ANA, Anti DNA, antiLKM and SLA were negative, HHsAg and HCV antibodies are negative.

A) What is the most likely diagnosis?

B) What is the possible differential diagnosis?

C) What are the investigations in this case?

D) What is the line of treatment?

E) What is the cause of puriritis in this case and what is the treatment?

F) What are the causes of sinus bradycardia?

Case no.6

A 22 year old male develops the insidious onset of low back pain improved by exercise and worsens by rest. 3

days ago the patient developed right heel pain. There is no history of diarrhea, urethritis, or nail changes. On

exam there is tenderness over the sacrum, he was kyphotic posture. On cardiac examinatuin early diastolic

murmur was auscultated over left second intercostals space.

On investigations: hemoglobin 10.5 gm/dl. ESR 68 mm/h, AST 16µ and ALT 25µ (normal up to 40 µ). Alkaline

phosphatase 320 µ (normal 100 µ). Rheumatoid factor is nega.ve. Plain film of the spine shows sclerosis of the

sacroiliac joints.

A) What is the most likely diagnosis?

B) What is the possible differential diagnosis?

C) What is the cause and mechanism of cardiac murmur in the patient?

D) Is HLA typing a useful test in this patient and why?

E) What are the lines of treatment?

F) What are the complications of NSAIDrugs?

Page 66: ˘ˇˆ˙ - Ain Shams Universitymed.asu.edu.eg/uploads/med/Student Affairs/IEE_IO__2.pdf• Lt. Lower lobe bronchiectesis • Recurrent pul. Embolism • Chronic broncitis (May 96)

Faculty Of Medicine - ASU

Student Union Scientific Committee (SUSC)

Examination Note - 4th year P a g e | 66

Case no.7

A 14 year old boy is seen in the hospital because of a 3 wk history of fever between 38.3°C and 38.9°C, letharygy,

physical examination reveals marked cervical and inguinal lymphadenopathy, enlarged tonsils, with exudates, and

small haemorrhages on the soft palate, a WBC differen.al that has 50% lymphocytes and plapable spleen 2cm

below the left costal margin.

A) What is the most likely diagnosis?

B) What is the differential diagnosis?

C) How to confirm diagnosis?

D) What are the complications of ampicillin therapy in this patient?

E) What are the other complications of the agent causing this disease?

F) What are different patterns of fever and give example of each?

2010

Case 1

A 60 years old man was brought to the hospital by his wife because of being unwell . For the last 3 days he

has disturbed sleep rhythm . He has been taking diazepam 5 mg nightly . There was a history of blood

transfusion 25 years ago . On physical examina.on , he was sleepy but arrousable several large spider

angiomas were present . the parotid glands were enlarged . A sterixis was present His abdomen was slightly

distended with shiVing dullness . The liver was firm and percussed to a span of 8 cm . The spleen was palpable

. Laboratory data were as follows : peripheral blood white cell count 2.500 cell/mm3 ; Heamoglobin 10 g/dL ,

platelet count 90.000 /mm3 serum AST , 100 IU /L ( normal < 30 IU/L ) ALT 80 IU / L ( normal <45 IU /L ) , total

billirubin 1.2 mg / dL ; total protein 8.0 g/dL ; albumin 2.5 g/dL ; and prothrombin .me 16 seconds ( control ,

11 sec ) The asce.c fluid showed a white blood cell count of 389 cells/mm3 , 2% polymorphonuclear

leukocytes , an albumin concentra.on of 0.5 g/dL and a total protein level of 1 g/dL

1. What features help you to diagnose chronic versus acute liver disease in this patient ?

2. Does any particular factor from the history help you determine the cause of this man`s liver disease ?

3. What reversal factors could be contributing to this man`s presumed portosystemic encephalopathy ?

4. What is the serum ascitis albumin gradient , and of what value is it ? can you calculate it from this patient

?

5. Could you start diuretic therapy now ? Explain ?

6. What treatment would you offer this patient now and What is the long term plan ?

Case 2

A 71 year old female , 88 kg, transported to emergency department .the patient displayed altered mental

status , incon.nence and fever. The medical history included Type 2 Diabetes Mellitus of 17 years dura.on ,

treated with medical nutrition therapy and oral hypoglycemic agents ; hypertension ; hyperlipidemia and

recent hip fracture treated by iternal fixation.

Following admission to ICU .it was noted that she has weakness on her left side . On examination: she has

poor skin turgor , decreased alerteness , disoriented difficulity speaking and weakness on the left side BP

90/58 HR 128 bpm RR 32/ minute T 38.5 (oral ) .Lungs with rhonchi Blood test WBC 18.000 Hgb 10.8 gm /dl .

Hct 36%. Glucose 780 mg/dl .Alc 8.2 % Crea.nine 2.2mg/dl . BUN 39 mg/dl .Na 155 mEQ/L .Ca 8.1 mg/dl ,

Page 67: ˘ˇˆ˙ - Ain Shams Universitymed.asu.edu.eg/uploads/med/Student Affairs/IEE_IO__2.pdf• Lt. Lower lobe bronchiectesis • Recurrent pul. Embolism • Chronic broncitis (May 96)

Faculty Of Medicine - ASU

Student Union Scientific Committee (SUSC)

Examination Note - 4th year P a g e | 67

Serum osmolality 340mOsm/kg . ABGs : PH 7.5 ,PaO2 100 mmHg ,PaCO2 20 mmHg HCO3 7.5 mmol/L. SaO2

98% on 2 L . Urine analysis , pus cells > 100 , Specific gravity 1.034 Glucose 4+ Protein 2 + ,RBCs few.

1. Discuss the diagnosis, list the classic signs and symptoms ?

2. What is the differential diagnosis ?

3. Discuss the lab value of sodium , glucose ,osmolarity and other lab values ?

4. What are the potential precipitating factors ?

5. Discuss the treatment?

6. List the potential complications of this problem

Case 3

A 50 year old pa.ent know to be diabe.c for the last 15 years on oral hypoglycemic drugs , he presented to

the outpa.ent clinic with bilateral lower limbs edema , blood pressure was 180/110 ,pulse 80/ minute regular

On inves.ga.on serum crea.nine were 2.3 mg /dl blood urea 120 mg /dl, serum Na 120meq/L ,serum K 6.1

meq/L ,urinary protein 4 gm /24h urine .

1. What’s your diagnosis?

2. Enumerate 5 condi.ons that can lesd to proteinuria of the same range as such pa.ent ?

3. How can you control his blood pressure ?

4. What is the most probable explana.on of his hyponatremia ,enumerate 3 other condi.ons that can lead

to low Na level ?

5. How can you manage his hyperkalemia?

6. Name one diure.c that can be used in such condi.on and men.on 3 of main adverse reac.on of such

drug ?

Case 4

A 55 year old man , heavy smoker has been frequently admitted to the hospital for his chronic chest disease

The time he presented with fever , cough and blood tinged sputum with increase shortness of breath for the

last few days . After a brief management in the emergency room , the patient was admitted to the hospital ,

Few hours later the patient become drousy and confused , hence he was transferred from the ward to the

critical care unit . Chest x-ray showed right special lesion

1- What are the important points to be in the history taking , general and local chest examination to this

patient ?

2- What is the most likely diagnosis and differential diagnosis ?

3- What are the important investigations required for this patient ?

4- Discuss the progress and sequence of events in this case

5- What is the proper treatment of this patient , both in the emergency room and inside the hospital ?

6- What is the management and follow up of the patient after discharge from the hospital ?

Case 5

Page 68: ˘ˇˆ˙ - Ain Shams Universitymed.asu.edu.eg/uploads/med/Student Affairs/IEE_IO__2.pdf• Lt. Lower lobe bronchiectesis • Recurrent pul. Embolism • Chronic broncitis (May 96)

Faculty Of Medicine - ASU

Student Union Scientific Committee (SUSC)

Examination Note - 4th year P a g e | 68

A 65 years old man heavy smoker came to the emergency room complaining of chest pain . he described the

pain as a severe retrosternal pressure sensa.on that had awakened him from sleep 3 hours earlier . He

previously had been well but has medical history of hypercholesteremia . On examination he appears

uncomfortable and diaphore.c with a heart rate of 116 bpm , blood pressure 166/102 mmHg and respiratory

rate 22 breaths per minute and oxygen satura.on of 96% on room air . jugular venous pressure appears

normal . Auscultation of the chest reveals clear lung fluid , a regular rhythm with as S4 gallop and no murmers

or rubs . A chest reveals clear lung fluid a regular rhythm with an S4 gallop and no murmers or rubs . A chest

radiograph shows clear lungs and a normal cardiac silhouette . The ECG shows elevated ST segment

1- What is the most likely diagnosis and differential diagnosis of heart pain ?

2- What are the causes of elevated ST segment

3- Comment on further investigations

4- What is the next step in therapy ?

5- Mention risk factors for ischemic heart disease ?

6- What are causes and mechanism of jaundice in caediac patient ?

Case 6

A 48 year old man presented with right ankel pain for 8 hours . He states that the pain started abruptly at 2

AM and woke him from sleep ,was quite severe so painful that even the weight of the bed sheets on his knee

was unbearable .By the morning , the ankle had become warm , swollen and tender.He has never had pain ,

surgery or injury to his ankle and resolved in 2or 3 day aVer taking ibuprofen . His only medical history is

hypertension controlled with hydrochlorothiazide . On examina.on his temperature is 37.2 C , heart rate 104

bpm and blood .pressure 136/78 mmHg . His head and neck examina.ons are unremarkable , his chest is clear

with no gallops or murmurs on heart . His right ankle is swollen , with a moderate effusion and appears

erythematous warm and very tender to palpation . He has no other joint swelling , pain or deformity and no

skin rashes WBCs 13,000 ESR 35.

1. What is the most likely diagnosis ?

2. What is the differential diagnosis ?

3. What is your next step ?

4. What is the best initial treatment ?

5. What are the precipitating factors ?

6. What is the complicatins of the disease ?

Case 7

A 58 year old male presents with exer.onal dyspnea and palpita.on . The pa.ent complains of tingling and

numbness of feet and hands . He reports falling to the ground on washing his face in the morning without loss

of consciousness . Past history was irrelevant apart from history of gastric surgery for trea.ng pep.c ulcer 5

years before . On examina.on the pa.ent looks pale , the liver was palbable 2 fingers below cosatl margin .

Decreased pinprick sensation over the hands and feet with absent ankle and preserved knee reflex . Also

there is positive babiniski sign .

On investigation : hemoglobin 8 gm/dl . MCV 107 fl . WBCS 3000/mm platelets 90,000/ mm serum bilirubin

2mg/dl.

Page 69: ˘ˇˆ˙ - Ain Shams Universitymed.asu.edu.eg/uploads/med/Student Affairs/IEE_IO__2.pdf• Lt. Lower lobe bronchiectesis • Recurrent pul. Embolism • Chronic broncitis (May 96)

Faculty Of Medicine - ASU

Student Union Scientific Committee (SUSC)

Examination Note - 4th year P a g e | 69

1. What is the most likely diagnosis ?

2. What are the differential diagnosis of absent ankle and preserved knee reflex ?

3. What are the further investigations needed to confirm the diagnosis ?

4. What is the treatment of this patient?

5. What is the differential diagnosis of pancytopenia?

6. What is the treatment of peptic ulcer?

خواطر كرمي كامل " من كتاب

• When Bezos was thinking whether to stay with his job or quit to startup amazon.com, he asked which would I regret not doing when I am 80?

• The road to success is lined with countless tempting parking places. – Anonymous

• Insanely beautiful star clusters composed of 100,000 stars orbiting each other while whole cluster orbits the MilkyWay! Sub7an Allah!

• There is no telling how many miles you will have to run while chasing a dream. – Anonymous

• Success is going from failure to failure without losing your enthusiasm. -- Abraham Lincoln

• Depression was found to reduce the volume of the Hippocampus, a brain region critical for memory & navigation. Just cheer up & try again!

• Like fingerprints, brainwaves can serve as a personal signature consistent throughout life!

• The anger gene, the alcoholism gene, the gambling gene and others are nothing but attempts to blame misconduct on genetics.

• Just saw this on Facebook: One guy wrote: "I am board". Someone commented: "I am chalk, we should get together"!

• Explaining a joke is like dissecting a frog. Few people are interested and the frog dies. -- Elwyn White

• When working on a problem, don't let failed attempts discourage you. It is often the last key that opens the lock!

• To study spread of ideas over Social Networks, researchers use principles from epidemiology the science of disease transmission and control.

• Logic will get you from A to B. Imagination will take you everywhere. -- Albert Einstein

Page 70: ˘ˇˆ˙ - Ain Shams Universitymed.asu.edu.eg/uploads/med/Student Affairs/IEE_IO__2.pdf• Lt. Lower lobe bronchiectesis • Recurrent pul. Embolism • Chronic broncitis (May 96)

Faculty Of Medicine - ASU

Student Union Scientific Committee (SUSC)

Examination Note - 4th year P a g e | 70

PPPPPPPPeeeeeeeeddddddddiiiiiiiiaaaaaaaattttttttrrrrrrrriiiiiiiiccccccccssssssss

Page 71: ˘ˇˆ˙ - Ain Shams Universitymed.asu.edu.eg/uploads/med/Student Affairs/IEE_IO__2.pdf• Lt. Lower lobe bronchiectesis • Recurrent pul. Embolism • Chronic broncitis (May 96)

Faculty Of Medicine - ASU

Student Union Scientific Committee (SUSC)

Examination Note - 4th year P a g e | 71

Breast feeding

1. Describe the steps of feeding in breast fed infant from birth and up to one-year age. (Nov. 76) 2. Give an account on advantages of BREAST FEEDING and contraindications. (Dec. 77, Nov.

79, May 88, Sep. 92) 3. Give an account on the treatment of A BREAST FED INFANT aged 8 months and 8 kgm weight?

(Nov. 78) 4. Outline the weaning program of a BREAST-FED INFANT? (Nov. 79) 5. Enumerate the instructions given to the lactating mother to conduct a SUCCESSFUL BREAST-

FEEDING. (June 80, Nov. 83, May 84, Oct. 90) 6. Discuss difficulties of BREAST FEEDING (March 92, 99) 7. Mention 3 types and 3 indication of artificial milks? (Nov. 84) 8. How to judge an adequacy of breast-feeding? (Sep. 86)

• Discuss ten steps to successful breast-feeding. • Discuss fats in breast milk.

Growth

& development 1. Give an account on PROGRESS OF THE CHILD IN HEIGHT AND WEIGHT in the first 5 years of age?

(Dec. 77) 2. Discuss MILESTONES OF MOTOR DEVELOPMENT during first year. (79) 3. Give short notes on each of THE WEIGHT, HEIGHT, HEAD CIRCUMFERENCE OF THE BABY, at birth,

6 months and 3 years aged. (June 80) 4. Give an account on MILK TEETHING? (May 82, Sep. 87) 5. Give an account on REGULAR PHYSICAL GROW ASSESSMENT (84) 6. Give an account on ASSESSMENT TO DEVELOPMENT in one and half years old infant. (Sep. 86) 7. Discuss briefly the development of different stages of voluntary micturation? (Sep. 89) 8. An infant one year of age his Anterior fontanel is 5 cm wide, what are the causes and how to

diagnose one of them? 9. Discuss the development of the first year of life? (1994) 10. Discuss the ttt of short stature? (1993)

New born 1. Give an account on an etiology management and complications of NEONATAL JAUNDICE. (Nov.

87, Oct. 91, March 92) 2. Describe causes of RESPIRATORY DISTRESS in the new born (76) 3. Give an account on pathogenesis and management of RESPIRATORY DISTRESS SYNDROME. (Nov.

83) 4. Discuss clinical and radiological assessment of idiopathic respiratory distress syndrome. (May

94) 5. Discuss clinical and laboratory findings of different types of different types of neonatal jaundice.

(Nov. 80). 6. Enumerate the common causes of ICTERUS NEONATORUM, what are the important clinical ant

laboratory data that help in diagnosis and management. (May 82) 7. Mention Apger Score and how to treat NEONATAL ASPHYXIA? (Nov. 84) 8. Comment on

- The value of NEONATAL REFLEXES. (May 83) - Treatment of ERYTHOROBLASTOSIS FETALIS. (May 83)

Page 72: ˘ˇˆ˙ - Ain Shams Universitymed.asu.edu.eg/uploads/med/Student Affairs/IEE_IO__2.pdf• Lt. Lower lobe bronchiectesis • Recurrent pul. Embolism • Chronic broncitis (May 96)

Faculty Of Medicine - ASU

Student Union Scientific Committee (SUSC)

Examination Note - 4th year P a g e | 72

9. Finumerate - MATERNO-FETAL INFECTIONS. (Sep. 86) - Causes of NEONATAL, CONVULSIONS. (Sep. 86, 89)

10. Discuss the clinical assessment of CARDIORESPIRATORY FUNCTION OF THE NEWBORN? (Sep. 87).

11. Discuss HAEMORRHAGIC DISEASES of newborn. (March 92) 12. Discuss complications that may occur in PRETERM INFANTS. (93) 13. Discuss causes of ANEMIA in the neoborne. (94) 14. Enumerate POTENTIAL RISKS of infant of diabetic mother (96) 15. Give an account on CONGENITAL RUBELLA SYNDROME and its prevention (96) 16. Enumerate causes of CONVULSIONS in achild 2 years old & how to treat one of them (97) 17. Give an account on PHYSIOLOGICAL HANDICAPS of the preterm baby. What are the subsequent

complication (99)

Genetics 1. Discuss types of MONGOLISM you know as regards to chromosomal anomaly and prognosis. (Spt.

86) 2. Gave an account on KLINEFELTER SYNDROME. (Nov. 80) 3. What are the clinical manifestations that indicate the need for KRYOTYPING? (May 84) 4. Discuss 2 SEX CHROMOSOMAL ANOMALIES? (Sep. 87) 5. Give short account on CHROMOSOMAL ABERRATION leading to mental retardation. (Sep. 89) 6. Discuss briefly 2 AUTOSOMAL CHROMOSOMAL ABERRATION leading to mental retardation. (Oc. 90,

Sep. 92) 7. Discuss briefly clinical manifestations of TURNER'S SYNDROME

(March 91) 8. Discuss clinical picture down's syndrome and how to diagnose its various types? (Oct. 91) 9. Mention types of SEX CHROMOSOMAL ABERRATIONS and discuss one of them (March 91) 10. Discuss clinical picture & diagnosis of TRISOMY-18 (95) 11. Discuss clinical picture of TRISOMY-13 (96)

Nervous system

1. Enumerate the cause and grading of MENTAL RETARDATION (92) 2. Give differential diagnosis of A FLACCID PARALYSIS in a child 3 years old (June 80) 3. Give an account on:

a) Febrile Convulsions. (Nov. 80) b) Differential diagnosis of Flobby Baby. (Sep. 89, Oct. 90) c) Diagnosis And Management of Peripheral Neuropathy of Infant And Childhood. (Oct. 91)

4. Discuss causes and management of one of them (96) 5. 2 years old child unable to walk what are the possible causes and how to diagnose one of them?

G.I.T 1. what are the various types of DEHYDRATION? Describe clinical picture of each of

them (99)

2. Describe the treatment of MARKED ISOTONIC DEHYDRATION secondary to diarrhea in an infant weighting 8kg. (Nov. 76)

3. Discuss management of DEHYDRATION in toxic dyspepsia in an infant aged one year (79) 4. Discuss management of ACUTE DDEHYDRATION in infant 5 months age. (Nov. 79) 5. Discuss common complications of ACUTE GASTROENTERITIS. (82)

Page 73: ˘ˇˆ˙ - Ain Shams Universitymed.asu.edu.eg/uploads/med/Student Affairs/IEE_IO__2.pdf• Lt. Lower lobe bronchiectesis • Recurrent pul. Embolism • Chronic broncitis (May 96)

Faculty Of Medicine - ASU

Student Union Scientific Committee (SUSC)

Examination Note - 4th year P a g e | 73

6. Discuss manifestations and types of DEHYDRATION and role of O.R.T. in gastroenteritis? (Nov. 83)

7. Discuss REHYDRATION IN ACUTE G.E. (May 84) 8. What are THE SEVERE METABOLIC DISTURBANCE IN G.E. and how to manage? (Nov. 84) 9. Discuss I.V. FLUID in treatment of dehydration. (May 83) 10. Give an account on complications of ACUTE G.E. in infant one year old. (Sep. 86, 87) 11. What are the causes of DIARRHEA in infant aged one year Describe the clinical manifestation of

sever complicated (98) 12. Discuss clinical and laboratory diagnosis of HYPERTONIC DEHYDRATION? (Oct. 90) 13. Give an account on clinical manifestations and diagnosis of HYPERNATREMIA. (Oct. 91) 14. How to treat HYPERTONIC DEHYDRATION in an infant 5 months old. (Sep. 92)

Cardiology

1. Outline the prevention and treatment of RHEUMATIC FEVER. (91) 2. Discuss signs of ACTIVE RHEUMATIC CARDITIS. (Nov. 79) 3. Give an account on A) The diagnosis and treatment of ACUTE RHEUMATIC CARDITIS in a child 5 years old. (Sep. 89)

B) Differential diagnosis of heart failure. (Sep. 89)

4. Discuss the manifestations and management of ACUTE RHEUMATIC FEVER. (Sep. 89) 5. The most common causes of HEART FAHURE in a child 5 years. 6. Discuss aetiology prevention & treatment of RHEUMATIC HEART DISEASE. (94) 7. Enumerate causes of PARASTERNAL SYSTOLIC MURMURS, and how to reach the diagnosis in one of

them. 8. Discuss clinical picture, diagnosis in one of them. 9. Discuss clinical picture, diagnosis and complications of INFECTIVE ENDOCARDITIS in children. (97)

Respiratory system

1. Give an account on VIRAL PNEUMONIAS (Nov. 80)

2. Enumerate causes of STRIDOR in an infant one year old. (83) 3. Enumerate causes of G-VE PNEUMONTAS and discuss one of them?(Oct. 90) 4. Give an account on treatment of BRONCHEAL ASTHMA. 5. Give an account on LOBAR PNUMONIA in a child 5 years old. (95) 6. 2 years old child came to the emergency room because of Difficulty In Breathing And Cyanosis

Discuss (96) 7. Enumerate causes of ACUTE RESPIRATORY DISTRESS in a child aged 4 years How to dignose and

treat one of them (97) 8. Causes of CYNAOSIS in an infant one years old and how to diagnose one of them (99)

Haematology

1. Give an account on: A) ACUTE HAEMOLYTIC ANAEMIA common in 1st year of life.

B)Laboratory diagnosis of IRON DEFICIENCY ANAEMIA

(Nov. 83)

2. Outline the common types of ANAEMIA in Egyptian children (May 84). 3. What are the common causes of IRON DEFICIENCY ANAEMIA in school children in Egypt and what is

the treatment of such condition? (Nov. 84) 4. Mention causes of ANAEMIA in two years child (Sep. 87)

Page 74: ˘ˇˆ˙ - Ain Shams Universitymed.asu.edu.eg/uploads/med/Student Affairs/IEE_IO__2.pdf• Lt. Lower lobe bronchiectesis • Recurrent pul. Embolism • Chronic broncitis (May 96)

Faculty Of Medicine - ASU

Student Union Scientific Committee (SUSC)

Examination Note - 4th year P a g e | 74

5. How to manage patient 3 years old presented with PALLOR? (91) 6. Enumerate causes of CHRONIC HEMOLYTIC ANEMIAS in childhood. How to diagnosis & treat one of

them. (98)

Vaccination 1. Give a scheme of VACCINATION to be done for a child in the first year and describe

the methods of vaccination in table. (77, 79)

2. Discuss PROPER VACCINATION PROGRAM. (May 84) 3. How to assure the affinity of POLIOMYELITIS VACCINATION. (84) 4. Discuss THE VACCINATION SCHEDULE in infancy and childhood.

(May 83, Sep. 86) 5. How could VACCINATION against six killing diseases affect growth and development of children?

(Oct. 90, Sep. 92) 6. Enumerate the obligatory vaccinations during the first year of life & how to perform 2 of them.

Infectious diseases 1. Give the differential diagnosis of POLIOMYELITIS affecting both lower limbs and outline the

treatment in the acute stage. (Sep. 92) 2. Give the complications, treatment and prevention of WHOOPING COUGH.

(Oct. 91) 3. Discuss complication of Diphtheria (Nov. 78, May 83) 4. Describe the morphology of THE RASH OF THE MEASLES, CHICKEN POX and SCARLET FEVER.

Mention the incubation period of each. (80) 5. Erumerate diseases caused by or following STREPTOCOCCI INFECTIONS.

(Nov. 83) 6. Discuss value of TUBERCULIN TESTING in previously BCG vaccinated child. (Sep. 89) 7. Discuss management of case of DIPHTHERIA (May 84, 84, 86) 8. Discuss clinical types of ACUTE POLIOMYELITIS. (May 83) 9. Discuss clinical features of DIPHTHERIA. (Sep. 86) 10. Discuss complications of POLIMYELITIS. (Sep. 86) 11. Discuss the clinical picture and complications of mumps. (87) 12. Discuss the clinical features and treatment of TETANUS in a child 10 years old. (Sep. 89, 93) 13. Discuss prevention and treatment of MEASLES in infant less than 2 years of age. (Oct. 90, March

91) 14. Discuss clinical manifestations and complications of MEASLES. (92) 15. Clinical picture and complications of SCARLET FEVER. (97) 16. Discuss complications of GERMAN MEASLES. (98) 17. Describe the clinical picture of chicken pox. What are the possible complications? (99)

Endocrine 1. Discuss causes of DWARFISM (Nov. 83) 2. Discuss investigation of a child with STUNTED GROWTH (91) 3. Enumerate steps of management of INFANT OF BIABETIC MOTHERS. (90) 4. Give an account on the clinical picture of CRETINISM (95) 5. Diagnosis of insulin dependent DIABETES MELLITUS (97) 6. Give an account on DIABETIC KETOACIDOSIS (99)

Page 75: ˘ˇˆ˙ - Ain Shams Universitymed.asu.edu.eg/uploads/med/Student Affairs/IEE_IO__2.pdf• Lt. Lower lobe bronchiectesis • Recurrent pul. Embolism • Chronic broncitis (May 96)

Faculty Of Medicine - ASU

Student Union Scientific Committee (SUSC)

Examination Note - 4th year P a g e | 75

Renal 1. Discuss diagnosis and treatment of ACUTE GLOMERULO-NEPHRITIS. (Oct. 90) 2. Discuss clinical picture & management of NEPHROTIC SYNDROME in children (94) 3. How to manage a case of HEAVY PROTEINURIA in a child 5 years old. 4. Enumerate causes of EDEMA OF THE LOWER LIMBS in an infant 1.5 years old. How to treat one of

them. (98)

Hepatology 1. Discuss complications of LIVER CIRRHOSIS in children. (91) 2. Enumerate the causes of HEPATOMEGALY in child 4 years old

(Sep. 87, 89) 3. Give an account on causes of HEPATOMEGALY in infancy (91)

Rickets 1. Give an account on clinical & laboratory features of early RICKETS

(May 83) 2. What are the radiological and biochemical changes in VITAMIN DEFICIENCY RICKETS? (Dec. 77) 3. Discuss manifestations & signs of VITAMIN DEFICIENCY. (78) 4. Discuss RENAL, RICKETS? (May 84, Oct. 91) 5. What are the causes, clinical manifestations & lines of treatment of tetany? (98) 6. Clinical picture & investigations of a case of vit. D deficiency rickets.

Nutrition 1. Give briefly the etiology of marasmus & its management. (80) 2. Discuss prognosis of KWO. (Oct. 90) 3. Discuss prognosis of early preclinical KWO. (84, 92) 4. Discuss characterizing feature of chronic infantile protein energy malnutrition, what are the

preventive and therapeutic measures, which you recommended for its combat? (Nov. 80) 5. Discuss constant clinical features of protein malnutrition (84) 6. The W.H.O. recommended four measures for the developing countries to lower infant and child

mortality. Discuss these measures? (May 84) 7. What are the main pathological features of KWO (84) 8. Compare & contrast the etiology of marasmus and KWO (87) 9. Discuss types and causes of protein energy malnutrition? (92) 10. Give an account on diagnosis of generalized in a child 3 years.

(Nov. 78) 11. Discuss emergency complication of protein energy malnutrition?

�و��و���������� �?��������س�?� �

� �و ها هو ذا جالس باملقعد ، لقد جد و اجتهد طوال السنة ليصل لتلك اللحظة ، مل يصبح يف استطاعته االنتظار أكثر خطوة أخرى ، سارحا بتلك اللعبة اليت سيشتريها له والده كمكافأة على تفوقه ، اخللفي بالسيارة جبوار أخويه األكرب منه

السوق التجاري ليس ببعيد و لكنه ال يستطيع االنتظار أكثر، و امتالكه موعة األلعاب تلك كاملة حن .

Page 76: ˘ˇˆ˙ - Ain Shams Universitymed.asu.edu.eg/uploads/med/Student Affairs/IEE_IO__2.pdf• Lt. Lower lobe bronchiectesis • Recurrent pul. Embolism • Chronic broncitis (May 96)

Faculty Of Medicine - ASU

Student Union Scientific Committee (SUSC)

Examination Note - 4th year P a g e | 76

أخواه يقفزان من اجلانب األمين إىل اجلانب األيسر ، يذكر نفري السيارة ينطلق بصوت عال متصل ، مل يدر ما حدث بالظبط و بدال منها غطى ذلك اجلانب ، شمس غابت عن اجلانب األمين متاما و كأن ال، صوت شم زجاج ، باملقعد اخللفي حيث جيلس

و هو يف حالة ذهول و ال وعي مل يره عليها من ، والده يقفز خارج السيارة بعد توقفها ليطمئن عليهم ، جسد أسود ضخم بها األمين كامالحمطمة جان، خيرج من السيارة هو األخر لريى سيارة نقل ضخمة تقف مستندة إىل سيارم ، قبل .

و أن رغبته بتلك اللعبة هي ما جائت م إىل ذلك املكان ، و لكن أخوه األكرب المه على ذلك احلادث ، مل يصب أحدهم سوء و أن تلك األدعية هي ما ، و التسابيح بالطريق ، و أخوهم األوسط تلك األدعية ، و أنه مل يشاركه هو ، حيث حدثت احلادثة أن يفهم أن ما حدث ما هو إال تصاريف ، مل يستطع عقل الصغري أن يرد عن نفسه ذلك اهلجوم ، صابة محتهم من اإل

أن ال ذنب له باملوضوع كله، القدر .

.....و بدال من هدية النجاح حصل على عقدة االحساس بالذنب اليت ستظل معه ما بقي له من العمر

"! ابتعدوا"-

ليضعهم على ذلك الصدر العايل ، ممسكا بصاعقي جهاز اإلنعاش القليب ، كزة بطاقمه صرخ ا طبيب العناية املر

و كأا تريد رد اإلساءة ، و تنحين ألعلى فتترائى للناظر ، فتنقبض عضالت اجلسد كله اعتراضا على تلك اإلساءة ، أمامه و كأنه يعلن ، املرسوم عليها باللون األخضر ثابت و ذلك اخلط، ظل ذلك الطنني الصادر من شاشة املراقبة متصال عاليا .

رافضا ملا حيدث، و ثباته على موقفه ، عن موقف القلب .

"!أأنت جمنون ؟"-

فأجاا، و هي تؤنبه ، يتذكر صوت أمه :

-" و كان البد من أحد ليقف يف وجهه، مل أستطع أن أسكت عن ذلك اخلطأ ."

: صاحت به

-" و يف أغلب الوقت يكونوا خارج وعيهم، إم جمرمني ، و كان من املمكن أن يؤذيك ، عك مل تكن املشكلة م ."

: أجاا قائال

-" و وقتها سأحتاج ملن يقف جبانيب كما فعلت اليوم، يوما ما ستصبح مشكليت ."

و دموعها تسيل، قالت له :

-" يب عليكيوما ما ستأتيين غارقا بدمائك لتحرق قل، كما حيلو لك ."

حىت و إن كان ما فعله هو ، فصمت شاعرا بالذنب ملرأى دموعها ، و خوفها عليه ، كان يعلم أا تقول ذلك حلبها له

و كان ، لن يضر والدته أال تعرفه ، و خيصه ، فما ال تعلمه أمه ، فقد عاهد نفسه أال جيعل أمه تقلق عليه ثانية ، الصواب بينه و بني أسرته ذلك العهد حاجز أخر وضع .

Page 77: ˘ˇˆ˙ - Ain Shams Universitymed.asu.edu.eg/uploads/med/Student Affairs/IEE_IO__2.pdf• Lt. Lower lobe bronchiectesis • Recurrent pul. Embolism • Chronic broncitis (May 96)

Faculty Of Medicine - ASU

Student Union Scientific Committee (SUSC)

Examination Note - 4th year P a g e | 77

"! ابتعدوا"-

و كان اعتراض ، صرخ الطبيب مرة أخرى بعد أن زادت املمرضة شحنة الصاعق يف حماولة للتغلب على عناد ذلك القلب أو يتراجع ، دون أن يتوقف ذلك الطنني املتصل ، مث سقط مرة أخرى على السرير ، اجلسد يف املقابل أزيد فانثىن بشدة ألعلى

عن موقفهالقلب .

-" لكنك أضعت فرصتك، كان بإمكانك أن جتعل األحداث تسري يف املنحىن الذي تريده ."

و رده عليها قائال، تذكر تلك الكلمات :

".كانت مقامرة يستحيل أن أرحبها"-

".كان يكفيك عندها شرف احملاولة"-

-" و سأخسر ، مل أطبق ما أنادي به من مباديء وقتها مل يكن ليبقى يل من شرف ؛ سأفقد احترامي لنفسي كوين

إلنين بالفعل خسرت، و يف كل األحوال فكل هذا ال يهم األن ، و هذا ما ال أحتمله ، الشخص ."

"! ابتعدوا"-

حدث باملرة الثالثة دون أن يتغري شيء، و كما حدث باملرتني السابقتني ، صرخ ا الطبيب للمرة الثالثة .

ه مجل كثرية قيلت له على مدار حياته كل منها من شخص خمتلف جبرح خمتلفتوالت يف ذاكرت :

-" إنه قرارك وحدك، مل أطلب منك أال تسافر من أجلي ."

".مل تدافع عين إال لكرهك للشخص الذي أواجهه"-

-" حىت و إن كنت على صواب، و ينفرون منك ألسلوبك معهم ، و سيكرهك الناس ، فلتبق كما أنت ."

".ال إنه بال أصدقاء"-

انتظاره ألشياء مل حتدث، حبثه عن إجابات مل تأت أبدا ، و توالت مشاهد حياته سريعا يف رأسه ...

-" مساءا ٨:٤٥وقت الوفاة ."

مث استطرد متمتما، بعد أن رضخ لرغبة ذلك القلب يف عدم رغبته يف العودة لتلك احلياة ، قاهلا الطبيب :

-" جزنا عن إنقاذهاروح أخرى ع ." :أراد أن يرد قائال بسخريته اليت اشتهر ا حىت من مصائبه الشخصية

-" فالروح قد سبقته بوقت طويل، بل جمرد جسد عجزت عن إنقاذه ." او كان أخر ردوده اليت لن يقوهل

Page 78: ˘ˇˆ˙ - Ain Shams Universitymed.asu.edu.eg/uploads/med/Student Affairs/IEE_IO__2.pdf• Lt. Lower lobe bronchiectesis • Recurrent pul. Embolism • Chronic broncitis (May 96)

Faculty Of Medicine - ASU

Student Union Scientific Committee (SUSC)

Examination Note - 4th year P a g e | 78

EXAMS . . .EXAMS . . .EXAMS . . .EXAMS . . .

Page 79: ˘ˇˆ˙ - Ain Shams Universitymed.asu.edu.eg/uploads/med/Student Affairs/IEE_IO__2.pdf• Lt. Lower lobe bronchiectesis • Recurrent pul. Embolism • Chronic broncitis (May 96)

Faculty Of Medicine - ASU

Student Union Scientific Committee (SUSC)

Examination Note - 4th year P a g e | 79

1989-11

Answer all questions

1. Value of tuberculin testing in a previously B.C.G. vaccinated child. 2. Discuss clinical features of tetanus in a child 10 years old and its treatment. 3. Differential diagnosis of hepatomegaly and anemia in a child aged 6 years. 4. Give short account on chromosomal aberrations (structural and numerical) leading to mental

retardation. 5. Discuss briefly the development of different stages of voluntary micturition. 6. Differential dignosis of floppy baby. 7. Differential diagnosis of heart failure. 8. Discuss neonatal convulsions.

1989-12

Answer all questions

1. Discuss the prognosis of a case of kwashiorkor. 2. Give an account on the expanded program of immunization (Immunization during the first year of

lift). 3. Discuss the complications of scarlet fever. 4. Mention the differential diagnosis of neonatal jaundice. 5. What is she advise you give the mother to ensure successful breast feeding? 6. Enumerate causes of mental relation. 7. Discuss rheumatic carditis in children. 8. Discuss briefly the complications of severe gastroenteritis.

1990-2

Answer all questions

1. Compare and contrast the etiology and clinical features of marasmus and kwashiorkor. 2. Mention TWO common types of hemolytic anemia in Egyptian infants and children. What are the

main diagnostic measures and how to treat? 3. Mention the complications and treatment of measles in a child 2 years of age. 4. How can you investigate a case of liver cirrhosis in children. 5. Describe the clinical picture of trisomy 18. 6. Discuss the etiology of cardiomegaly in pediatric age group. 7. Discuss the complications of low birth weight. 8. Give an account on renal rickets.

5-1990

Answer all questions:

1. Complications of diphtheria in a child aged 5 years. 2. Primary and secondary prevention of Rheumatic fever. 3. Etiology of jaundice in a child aged 3 years old. 4. Give an account on:

a- Advantagees of lactose of breast milk. b- Hypo-allergenic dried milks.

5. Give an account on "Turner syndrome" 6. Diagnosis of kwashiorkor. 7. Ages and indications of booster vaccinations. 8. Discus the two most common types of chronic hemolytic anemias in Egypt.

(New system)

Page 80: ˘ˇˆ˙ - Ain Shams Universitymed.asu.edu.eg/uploads/med/Student Affairs/IEE_IO__2.pdf• Lt. Lower lobe bronchiectesis • Recurrent pul. Embolism • Chronic broncitis (May 96)

Faculty Of Medicine - ASU

Student Union Scientific Committee (SUSC)

Examination Note - 4th year P a g e | 80

1990-10

Answer eight (8) questions only:

1. Discuss the prognosis of a case of kwashiorkor. 2. Enumerate causes of gram positive pneumonias and discuss one of them. 3. Give an account on nutritional liver diseases in infants and children. 4. Discuss the prevention and treatment of measles in infants less than 2 years of age. 5. What are the instructions you give to mothers before and after delivery to insure successful breast

feeding. 6. a) Enumerate steps of management of infant of diabetic mother. b) Enumerate causes of floppy baby.

7. Discuss briefly TWO autosomal chromosomal aberrations. 8. Discuss clinical and laboratory diagnosis of hypertonic dehydration. 9. How would vaccination against the 6 killing disease affect growth and development in children? 10. Discuss the diagnosis and treatment of acute glomerulo-nephritis.

(Old System) 1990-12

1. How could you investigate a case of protein energy malnutrition? 2. Discuss the clinical manifestations and complications of mumps. 3. How could immunization against tetanus help eradication of the disease in its various forms? 4. Give an account on complications of acute viral hepatitis. 5. Enumerate causes of cyanosis in the newborn infant. 6. Discuss the difference in composition between breast and cow's milk. 7. Give an account on Down's syndrome. 8. Discuss respiratory poliomylitis?

(New system) 1991-3

1. Discuss nutritional rehabilitation of kwashiorkor. 2. Prevention and treatment of measles. 3. How to manage a patient 3 years-old presenting with pallor? 4. Discuss the complications of liver cirrhosis in children. 5. What are the advantages of the fat content of breast milk? 6. Discuss briefly clinical manifestations of Turner syndrome. 7. Prevention of rheumatic fever. 8. Treatment of bronchial asthma.

(Old system) 1991-5

1. Discuss clinical presentations of acute rheumatic fever. 2. Booster vaccination against the 6 killing diseases. 3. Complications of diphtheria in children. 4. Enumerate causes of infectious hepatitis and its complications. 5. Discuss advantages of breast feeding from the infant side. 6. Clinical manifestations of bullbar poliomyelitis. 7. Compare between mongolism and cretinism (Clinically and by laboratory). 8. Clinical picture, investigations, Complications of diabetes mellitus in children.

Page 81: ˘ˇˆ˙ - Ain Shams Universitymed.asu.edu.eg/uploads/med/Student Affairs/IEE_IO__2.pdf• Lt. Lower lobe bronchiectesis • Recurrent pul. Embolism • Chronic broncitis (May 96)

Faculty Of Medicine - ASU

Student Union Scientific Committee (SUSC)

Examination Note - 4th year P a g e | 81

1991-10

1. Clinical manifestations and diagnosis of hypernatremia. 2. Differential diagnosis of acute poliomyelitis of both lower limbs in an infant one year old. 3. Management of a newborn with intense jaundice on the first day of life. 4. Prevention and treatment of whooping cough. 5. Discuss causes of hepatomegaly in infancy. 6. Discuss clinical picture of down's syndrome. How to diagnose its various types. 7. Investigations of a child with stunted growth. 8. Give an account on renal rickets.

1991-12

1. Difficulties in breast feeding and how to overcome them. 2. Management of acute rheumatic fever. 3. Neonatal reflexes. 4. How to prevent measles and what are the complications of measles? 5. A 2.5 years old child complaining of delayed walking. What are the possible causes and lines of

treatment? 6. Enumerate the causes of generalized edema in an infant ONE year old, and discuss the

investigations. 7. A 10 years old girl after being continent started to develop nocturnal enuresis. What is your

differential diagnosis and Management. 8. Diagnosis of tuberculosis.

1993-3

1. Hemorrhagic disease of the new born. 2. Infant aged one year presenting with convulsions. What are the possible causes? 3. Discuss difficulties of breast feeding. 4. Peripheral neuropathy in infancy and childhood. Diagnosis and management. 5. Discuss the types and causes of protein energy malnutrition. 6. Mention the type of sex chromosome aberration and discuss one of them. 7. The most common causes of heart failure in a child aged 5 years. 8. Clinical manifestations and complications of measles.

1992-9

1. What are SIX killing diseases of infants and children? And how to immunize against TWO of them? 2. How to treat: a- Hypertonic dehydration in an infant 5 months old? b- Acute poliomyelitis of both lower limbs with respiratory affection?

3. Discuss manifestations and management of rheumatic fever. 4. Clinical manifestations and treatment of tetanus in children. 5. Clinical picture of TWO sex chromosome aberrations and their chromosomal pattern. 6. Enumerate cause of jaundice in the neonatal period and discuss the most important complication. 7. Advantage of breast milk. 8. Management of:

a) bronchial asthma

b) Acute post-streptococcal glomerulo glomerulo-nephritis.

Page 82: ˘ˇˆ˙ - Ain Shams Universitymed.asu.edu.eg/uploads/med/Student Affairs/IEE_IO__2.pdf• Lt. Lower lobe bronchiectesis • Recurrent pul. Embolism • Chronic broncitis (May 96)

Faculty Of Medicine - ASU

Student Union Scientific Committee (SUSC)

Examination Note - 4th year P a g e | 82

(Old system ) 1992-12

1. Clinical manifestations of measles. How to immunize against it? 2. Give an account on hepatomegaly in the neonatal period. 3. Advantages of breast milk. 4. Management of Nephrotic syndrome. 5. What are the causes of secondary inability to walk? How to diagnose one of these causes? 6. Differential diagnosis of cyanosis in a newborn. 7. Discuss complications that might occur in an infant of diabetic mother. 8. How to diagnose and treat an infant with hypothyroidism.

1993-3

1. Discuss management of rheumatic fever. 2. Assessment of growth and development in the first year of life. 3. Give an account on the flaccid paraplegia in infancy and childhood. 4. Differential diagnosis of fever with maculo-papular eruptions and how to vaccinate against ONE of

them. 5. How to treat an infant suffering from severe dehydration with manifestations of keto-acidosis? 6. Give a short account on protein-calorie-malnutrition (PCM). 7. Differential diagnosis of wheezy chest. 8. Discuss different types of Down syndrome.

1993-7

1. Advantages of breast feeding. 2. Discuss growth and development during the first year of life. 3. Clinical manifestations of measles and prevention of the disease. 4. Common causes of ascites in Egyptian children and management of

a case. 5. Manifestations of rheumatic fever in a child 5 years old. 6. Primary inability to walk in a child 3 years old.

1993-9

1. Enumerate the obligatory vaccinations during the first year of life, and how to perform 2 of them. 2. Complications of poliomyelitis. 3. Diagnosis and treatment of short stature. 4. Symptoms and signs of dehydration. How to differentiate between its various types and degrees. 5. Prevention and treatment of rheumatic fever. 6. Causes and clinical manifestations of Down's syndrome. 7. Complications that may occur in preterm infants. 8. Advantages and difficulties that may accompany breast feeding.

1993-12

1. Causes of dehydration and how to manage a case in an infant 7 months old? 2. Discuss the staphylococcal pneumonia with special emphasis on clinical picture and radiological

findings.

Page 83: ˘ˇˆ˙ - Ain Shams Universitymed.asu.edu.eg/uploads/med/Student Affairs/IEE_IO__2.pdf• Lt. Lower lobe bronchiectesis • Recurrent pul. Embolism • Chronic broncitis (May 96)

Faculty Of Medicine - ASU

Student Union Scientific Committee (SUSC)

Examination Note - 4th year P a g e | 83

3. What are the advantages of breast feeding? 4. Give an account on ascites in infancy and childhood. 5. Discuss carditis in a child aged 5 years. 6. Indications and complications of exchange blood transfusion.

1994-3 1. Give an account on measles, differential diagnosis, complications, management and prevention. 2. Discuss carditis in a child 6-years-old. 3. Discuss neonatal convulsions. 4. What are the advantage of breast feeding? 5. What are the types and clinical features of dehydration following severe attack of gastroenteritis in

children? 6. Complications of exchange transfusion. 7. How to differentiate between acute glomerulo-nephritis and nephrotic syndrome? 8. Enumerate common Trisomy syndromes and discuss one of them.

1994-9

1. Discuss emergency complications of protein energy malnutrition. 2. Types of dehydration and its clinical picture. 3. Etiology, prevention and treatment of rheumatic heart disease. 4. Complications of diphtheria and differential diagnosis of these complications. 5. Causes of anemia in the newborn. 6. Mention two sex chromosomal aberrations. Describe the clinical picture and diagnosis of one of

them. 7. Ten steps to successful breast feeding. 8. Discuss development in the first year of the life. 9. Clinical picture and management of nephrotic syndrome in children. 10. Discuss vaccination schedule in the first year of life.

1994-12

1. Discuss vaccinations in the first year of life. 2. Differential diagnosis of fever with rash in children. 3. Causes of anemia in the 1st year of life. 4. Discuss the handicaps of prematurity. 5. Discuss advantages of breast milk. 6. Management of marasmic infant 5-months old. 7. Give an account on clinical picture, complications and treatment of minimal lesion nephrotic

syndrome. 8. Floppy baby.

1995-3

1. Antenatal, natal, and postnatal instructions to the mothers to ensure successful breast feeding.. 2. CNS complications of measles, chicken pox and whooping cough. 3. Child aged 2 years with delayed walking. Enumerate the causes and how to diagnose ONE of them. 4. Causes of neonatal jaundice and how to diagnose ONE of them. 5. Clinical manifestations of hypertonic dehydration. 6. Etiology and clinical picture of bronchial asthma. 7. Clinical picture and complications of kwashiorkor. 8. Types of renal edema in a child 3 years old. And how to diagnose?

Page 84: ˘ˇˆ˙ - Ain Shams Universitymed.asu.edu.eg/uploads/med/Student Affairs/IEE_IO__2.pdf• Lt. Lower lobe bronchiectesis • Recurrent pul. Embolism • Chronic broncitis (May 96)

Faculty Of Medicine - ASU

Student Union Scientific Committee (SUSC)

Examination Note - 4th year P a g e | 84

9. Prophylaxis against rheumatic fever. 10. Complications of obligatory vaccinations in the first year of life.

6-1995

1. Clinical picture, investigations and treatment of infantile rickets. 2. Advantages of breast milk. 3. Causes of neonatal hyperbilirubinemia. 4. Obligatory vaccinations during the 1st year of life. 5. Management of severe hypertonic dehydration complicated with acidosis in a child 2 years old. 6. Complications of acute post-streptococcal glomerulonephritis. 7. Complications of chicken pox. 8. Diagnosis of rheumatic fever.

Page 85: ˘ˇˆ˙ - Ain Shams Universitymed.asu.edu.eg/uploads/med/Student Affairs/IEE_IO__2.pdf• Lt. Lower lobe bronchiectesis • Recurrent pul. Embolism • Chronic broncitis (May 96)

Faculty Of Medicine - ASU

Student Union Scientific Committee (SUSC)

Examination Note - 4th year P a g e | 85

1995-9

1. Discuss causes of neonatal hyperbilirubinemia. 2. Discuss proteins of breast milk and its functions. 3. Discuss clinical picture and treatment of acute glomerulonephritis. 4. Give an account on complications and treatment of whooping cough. 5. Give an account on clinical picture of cretinism. 6. Discuss clinical picture and diagnosis of trisomy 18. 7. Discuss prevention of a case of acute polimyelitis. 8. Give an account on orla rehydration. 9. Discuss growth in the first year of life. 10. Give an account on llobar pneumonia in a child 5 years old.

1996-3

1. discuss karyotyping of down's syndrome and their recurrence risk. 2. Enumerate causes of neonatal hyper-bilirubinemia and how to treat one of them. 3. Discuss differential diagnosis of flaccid paralysis of both lower limbs in a child 3 years old. 4. Discuss the clinical picture, complications and treatment of chicken pox. 5. How to diagnose and treat a case of rheumatic fever in a child aged 8 years. 6. Discuss the clinical picture and investigations of a case of nephrotic syndrome. 7. Discuss the complications of protein energy malnutrition. 8. Discuss the clinical picture and how to diagnose streptococcal pneumonia.

1996-9

1. Causes and differential diagnosis of flaccid paraplegia and management of one of them. 2. Discuss the advantages of the composition of breast milk. 3. Enumerate different causes of Neonatal jaundice and how to treat one of them. 4. Discuss clinical picture of Trisomy-13. 5. A child 2 years old came to the emergency room because of difficulty in breathing and cyanosis.

Discuss. 6. How to assess physical growth during the 1st year of life. 7. An infant one year of age. His anterior fontanel is 5cm. Wid. What are the causes and how to

diagnose one of them? 8. Discuss types and management of severe dehydration following gastro-enteritis. 9. Enumerate potential risks of infant of diabetic mother. 10. Give an account on congenital rubella syndrome and its prevention. [

1997-3

1. Discuss mumps, mention its clinical picture, differential diagnosis and management. 2. Discuss fats in "breast Milk". 3. Enumerate causes of para-sternal systolic murmurs, and how to reach the diagnosis in short in one

of them. 4. Discuss the causes of Gram-negative pneumonias. 5. Common causes of anemia in Egyptian children, and discuss one of them. 6. A child 2-years old unable to walk. What are the possible causes and how to dignose on of them. 7. How to manage a case of heavy proteinuria in a child 5 years old.

Page 86: ˘ˇˆ˙ - Ain Shams Universitymed.asu.edu.eg/uploads/med/Student Affairs/IEE_IO__2.pdf• Lt. Lower lobe bronchiectesis • Recurrent pul. Embolism • Chronic broncitis (May 96)

Faculty Of Medicine - ASU

Student Union Scientific Committee (SUSC)

Examination Note - 4th year P a g e | 86

8. Causes of neonatal convulsions, and how to treat one of them.

1997 -9 ALL QUESTIONS ARE TO BE ANSWERED

1. Discuss the differential diagnosis of FLACCID PARALYSIS of both lower limbs in a child 3 years old. 2. Enumerate causes of CONVULSIONS in a child 2 years old. And how to treat one of them. 3. Give an account on differential dignosis of NEONATAL JAUNDICE. 4. Give an account on HYPERTONIC DEHYDRATION and how to manage the case. 5. Discuss clinical picture, diagnosis and complications of INFECTIVE ENDOCARDITIS IN CHILDREN. 6. Diagnosis of INSULIN-DEPENDENT DIABETES MELLITUS. 7. Enumerate causes of acute RESPIRATORY DISTRESS in a child aged 4 years. How to diagnose and

treat one of them.. 8. Discuss clinical picture and diagnosis of TRIOSMY 13. 9. Clinical picture and investigations of a case of VIT. D DEFICIENCY RICKETS. 10. Clinical picture and complications of SCARLET FEVER.

1998

ALL QUESTION ARE TO BE ANSWERED:

1. Causes of SHORT STATURE in children. Discuss the pathogenesis and treatment of one of them. 2. What are the indications and complications of EXCHANGE TRANSFUSION in the newborn? 3. What are the causes of DIARRHEA in an infant aged one year, Describe the clinical manifestations of

a severe complicated case. 4. What are the causes, clinical manifestations and lines of treatment of TETANY. 5. Discuss the complications of GERMAN MEASLES. 6. Enumerate causes of EDEMA OF THE LOWER LIMBS in an infant one and half year old. How to treat

one of them. 7. Enumerate causes of CHRONIC HEMOLYTIC ANEMIAS in childhood. How to diagnose and treat one

of them. 8. Advantages of BREAST-FEEDING. 9. Enumerate the common AUTOSOMAL TRISOMIE. Describe the clinical picture of them. 10. Diagnosis, prevention and treatment of INFECTIVE ENDOCARDITIS.

1999

ALL QUESTIONS ARE TO BE ANSWERED:

1. Composition of breast milk, what are the difficulties that may occur during BREAST FEEDING 2. Give THE SCHEDULE OF OBLIGATORY VACCINATION in the first 2 years of life. 3. Give an account on PHYSIOLOGICAL HANDICAPS of the preterm baby what are the subsequent

complications. 4. What are the various types of DEHYDRATION? Describe the clinical picture of each of them? 5. Child 3 years old suffering from secondary inability to walk due to LOWER MOTOR NEURON LESION.

What are the possible causes and how to diagnose one of them? 6. Describe the clinical picture of CHICKEN POX. What are the possible complications? 7. Give an account on DIABETIC KETOACIDOSIS. 8. A child 5 years old presenting with PALLOR AND HEPATOSPLENOMEGALY. What are the possible

causes and how to diagnose one of them? 9. Describe the clinical picture and diagnosis of TRISOMY 13 . 10. Causes of CYANOSIS in an infant one-year-old and how to diagnose one of them.

Page 87: ˘ˇˆ˙ - Ain Shams Universitymed.asu.edu.eg/uploads/med/Student Affairs/IEE_IO__2.pdf• Lt. Lower lobe bronchiectesis • Recurrent pul. Embolism • Chronic broncitis (May 96)

Faculty Of Medicine - ASU

Student Union Scientific Committee (SUSC)

Examination Note - 4th year P a g e | 87

2000-9

Time allowed 2 h

All questions are to be answered:

1. Enumerate the various causes of diarrhia and describe the clinical picture of dehydration. 2. What are the complications of congenital heart disease and how to treat one of them. 3. What are the advantages and contraindications of breast feeding. 4. Give an account on persistant jaundice after the neonatal period. 5. Discuss the clinical features and recurrence risk of down's syndrome. 6. Enumerate morbidities that occur in infants of diabetic mothers and how to treat one of them. 7. Discuss prevention and treatment of pulmonary tuberculosis. 8. Enumerate the cause of artheritis in a child aged 5 years and how to treat one of them. 9. Enumerate three virus infections with rash that may lead to nervous complications. How to prevent

one of these diseases? 10. How to assess growth and development in the first year of life.

2001-3

All questions are to be answered:

Discuss the following:

1. clinical picture and management of acute Poststreptococcal glomerulonephritis in children. 2. Aetiology, clinical picture, diagnosis and treatment of phenyl ketonuria. 3. Clinical picture and investigations of diabetes mellitus in children. 4. Clinical picture, investigations and treatment of iron deficiency anaemia. 5. Diagnosis of congenital heart disease. 6. Enumerate neonatal reflexes and discuss two of them. 7. Nocturnal enuresis in children. 8. Difficulties of breast-feeding. 9. Clinical picture and treatment of acute severe asthma. 10. Clinical picture and complications of chickenpox.

2001-9

All questions are to be answered:

Discuss:

1. aetiology, clinical picture and management of phenyl ketonuria. 2. Aetiology and clinical picture of childhood nephrotic syndrome. 3. Diagnosis of type I diabetes mellitus. 4. Anti-infective properties of breast milk. 5. Enuresis in children. 6. Enumerate causes of anaemia with hepatosplenomegaly in pediatrics and how to diagnose one of

them. 7. Idiopathic respiratory distress syndrome in the newbron. 8. Diagnosis of acute rheumatic fever. 9. Obligatory vaccination in the first year of life 10. Clinical manifestations of different types of dehydration.

2002-10

All questions are to be answered:

Page 88: ˘ˇˆ˙ - Ain Shams Universitymed.asu.edu.eg/uploads/med/Student Affairs/IEE_IO__2.pdf• Lt. Lower lobe bronchiectesis • Recurrent pul. Embolism • Chronic broncitis (May 96)

Faculty Of Medicine - ASU

Student Union Scientific Committee (SUSC)

Examination Note - 4th year P a g e | 88

Time allowed 2 h

Discuss:

1. Aetiology & clinical features of down $. 2. Causes & management of growth hormone defeciency. 3. Aetiology & clinical picture of acute renal failure in children. 4. Complications of prematurity. 5. Clinical picture & diagnosis of meningitis. 6. Risks of blood transfusion in pediatrics. 7. Differential diagnosis of anemia with hepatosplenomegaly. 8. Enumerate difficulties of breast feeding. 9. Treatment of acute bronchial asthma in children. 10. Differential diagnosis of different types of dehydration.

10-2006

ALL Questions are to be Answered:

1- Enumerate extra-pulmonary causes of peripheral respiratory

distress in the newborn and discuss ONE of them.

2- Give an account on nocturnal enuresis.

3- Enumerate causes of anemia in a 5-yr. old child and give a short

account on ONE of them.

4- Give a short account on the management of acute renal failure in childhood.

5-Discuss guidelinese of weaning and give account on complications of faulty weaning.

6-Enumerate causes of neonatal jaundice and discuss ONE of them.

7- How to diagnose and prevent bacterial meningitis.

8- Discuss clinical picture and management of a common congenital heart disease.

9-Causes and management of growth hormone deficiency.

10-Give an account galactosemia.

2007-10Pediatrics

ALL QUESTIONS ARE TO BE ANSWERED :

1- Enumerate causes of bleeding aVer 5 days of birth and discuss management of ONE of them .

2- Discuss clinical presentations of acute poliomyelitis .

3- Enumerate causes of chronic vomiting in infancy and childhood.

Discuss ONE of them . 4- Growth assessment in the first year of life .

5- Discuss types of milk given to sick babies.

6- Enumerate major respiratory problems in preterm babies . Discuss management of ONE of them Discuss

the clinical picture and management of VSD.

7- Discuss causes and diagnosis of acute renal failure.

8- Enumerate vaccinations of the six killing diseases in infancy and discuss TWO of them .

9- How to diagnose a case of jaundive in a 2 year old child .

October 2008

( 20 marks each)

Page 89: ˘ˇˆ˙ - Ain Shams Universitymed.asu.edu.eg/uploads/med/Student Affairs/IEE_IO__2.pdf• Lt. Lower lobe bronchiectesis • Recurrent pul. Embolism • Chronic broncitis (May 96)

Faculty Of Medicine - ASU

Student Union Scientific Committee (SUSC)

Examination Note - 4th year P a g e | 89

1- A 9 years old boy, recently developed nocturnal enuresis aVer a stressful event, discuss diagnosis and

management.

2- Discuss C/P and inves.ga.ons of a 5 years old child with random blood sugar 300mg/dl

3-discuss how breast feeding decreases infection in infants.

4- A 3 years old child presents with fever and maculopapular rash. What is your differential diagnosis and discuss

one of them.

5- Enumerate causes of non hemoly.c anemia in a 6 years old child and discuss management of one of them

6- A 6 years old presented with puffy eye lids and piWng edema of lower limbs, urine examination revealed

albumin 3+.

• Enumerate differential diagnosis

• Discuss in short clinical features

• Investigation and treatment of the most likely diagnosis.

7- Discuss clinical picture and investigations of bronchial asthma.

8-A neonate delivered at 30 weeks gesta.on and weighing 1400g. Discuss the risks he is facing.

2009

questions are to be answered

1-A 2 year old child presented with delayed speech and milestones of development. Discuss the differen.al

diagnosis.

2- A 7 year old child presented in the emergency room with fever, rash and neurological signs.Discusss differential

diagnosis.

3-A 6 month old infant with delayed milestones is suspected to have congenital hypothyroidism. How to confirm

diagnosis?. Discuss treatment and prevention.

4-A 6 year old child complaining of recurrent bouts of abdominal pain of 6 months dura.on. Discuss how to reach

the diagnosis.

5- 3 year old male child presented with cutaneous .ny bleeding spots. Enumerate suspected causes. How to

diagnose one of them?

6-A 5 year old boy presented with painful swelling of his leV knee of two days dura.on. Enumerate causes.

Mention how to diagnose and treat one of them.

7- A 24 hour old male neonate weighing 1900 gm is brought to emergency room for repeated cessa.on of

respiration associated with change in color and floppiness. What is your diagnosis? , discuss management.

8- An 8 month infant presented with feeding difficul.es, inability to gain weight, tachypnea and tachycardia. On

examination a loud systolic murmur was heard along the left sternal border. What is the possible diagnosis? How

to manage this case?

2010

All questions are to be answered

Page 90: ˘ˇˆ˙ - Ain Shams Universitymed.asu.edu.eg/uploads/med/Student Affairs/IEE_IO__2.pdf• Lt. Lower lobe bronchiectesis • Recurrent pul. Embolism • Chronic broncitis (May 96)

Faculty Of Medicine - ASU

Student Union Scientific Committee (SUSC)

Examination Note - 4th year P a g e | 90

1-One year old infant presented with anemia not responding to hematenics, with history of 4 blood transfusions

.O/E: pallor, mild jaundice, enlarged spleen. Discuss lab investigation to confirm diagnosis.

2-A 4 year old boy presented with short stature. His parents are of normal heights, he has neither chronic

illnesses nor history of drug intake. What are possible causes and management?

3-A 5 year old boy presented to the ED with loss of consciousness and generalized tonic-clonic convulsions of 1 hr

duration. Discuss how to proceed in the management of this case

4-Enumerate bioactive components of breast milk.

5-A child 6 year old has a fever of 38°C for 2 days, on the third day of fever, he develops a rash. What are common

possible conditions?

6-A full term baby presented with a respiratory rate of 70/minute at his 5th

day of life. Enumerate causes and

discuss management of one of them.

7-A 3 year old boy presented with red, hot and swollen leV knee; what is your differen.al diagnosis.

8-A 2 year old infant presents with respiratory distress, fever. O/E: decrease air entry on the whole right side,

dullness on percussion, shifting of the mediastinum. List the etiology of this case.

9-A 6 year old boy presents with frequent bed weWng at night: 3.mes/night. Discuss differen.al diagnosis.

10- A 5 year old girl developed sudden hypotension, chest .ghtness and loss of consciousness after long

acting penicillin injection. Discuss shortly the treatment of this condition

مشاركة من أحمد نصر خطاب

رسول هللا صلى هللا عليه وسلم: قال "يا علي � تنم إ� بعد أن تأتي بخمسة أشياء"

قراءة القرآن كله .1 التصدق بأربعة آ�ف درھم .2 زيارة الكعبه .3 حفظ مكانك في الجنه .4 إرضاء الخصوم .5

علي كيف ذلك يارسول هللا؟؟: قال

: فقال له . إذا قرأت سورة اFخBص ثBث مرات فقد قرأت القرآن كله .1 . إذا قرأت الفاتحه ثBث مرات فقد تصدقت بأربعة آ�ف درھم .2 ه له الملك وله الحمد يحي ويميت وھو على كل شئ قدير عشر مرات فقد و إذا قلت �إله إ�هللا وحده �شريك ل .3

. زرت الكعبه . وإذا قلت �حول و�قوة إ� باN العلي العظيم عشر مرات فقد حفظت مكانك في الجنة .4 . موإذا قلت أستغفر هللا العظيم الذي �إله إ�ھو الحي القيوم وأتوب إليه عشر مرات فقد أرضيت الخصو .5

Page 91: ˘ˇˆ˙ - Ain Shams Universitymed.asu.edu.eg/uploads/med/Student Affairs/IEE_IO__2.pdf• Lt. Lower lobe bronchiectesis • Recurrent pul. Embolism • Chronic broncitis (May 96)

Faculty Of Medicine - ASU

Student Union Scientific Committee (SUSC)

Examination Note - 4th year P a g e | 91

؟ دراستك يف تتفوق كيف : اصال دراستك يف التفوق تريد ملاذا

العلم وطالب العلم على حتث اليت والنصوص) فليتقنه عمال احدكم عمل إذا( يقول الكرمي الرسول وجل عز هللا ارضاء .1 عديدة

الوالدين بر من وهذا املرات عشرات مضاعفة تكون ابنها بنجاح االم فرحة فإن لوالديك ارضاء .2 بني الدهر ابد يعش اجلبال صعود يتهيب ومن( القوم دون مع يكون ان يرضى ال الطموح الشخص فإن لنفسك ارضاء .3

( احلفر

: ؟ بدراسيت أتفوق كيف

اهلدف حدد (1 األجل طويلة أخر وأهدافا األجل قصرية أهدافا ألنفسهم حيددوا أن من للطالب بد وال

أسابيع عشرة إىل أسبوعني خالل بالكامل وحتقيقها مهماا إجناز ميكن اليت هي ألجلا القصرية فاألهداف الفصل هذا يف ممتاز تقدير على احلصول أريد: مثل متابعة أو، دراسية ملنحة بعثة على واحصل، ممتاز مبعدل اجلامعة من التخرج أريد: مثل فهو األجل الطويل اهلدف وأما

التخصص الجم يف العليا الدارسة

: اهدافك يف طموحا كن (2 مل النجاح طموحه كان ومن ، الثمانني على حصل التسيعني اىل طمح ومن ، التسعني على حصل للمئة طمح من

احلظ حيالفه مقبولة فالنتيبجة جيدة ادؤه كان ان االنسان ألن جيدة النتيجة ممتاز ادؤه كان وان

متميزاا ادؤك يكون أن جيب ممتازة النتيجة تكون ولكي انت من لك اقل صديقك من يل قل: فائقة بعناية أصدقائك اختر (4

الذاتية بقدراتك ثق (5 الدراسية النجاح متطلبات أول مها الذاتية بالقدرات والثقة بالنفس الثقة أن

: النجاح ادوات كل ما متلك انت (6

: احلقائق هذه امسع ا اهللا اكرمنا بصري_ حس تقبالاس عصب مليون مئة االنسان ميلك واألداء العمل باهر التصميم رائع الدقة بالغ مسعي نظام االنسان ميلك يريد أينما حتملع جسمه يف عضله) ٥٠٠( االنسان ميلك عظمه) ٢٠٠( من وأكثر واألوردة الشرايني شبكات من مترا كيلو) ١٠٠( من وأكثر الدم من جالون آالف أربعة من اكثر ويضخ ، توقف دون ايومي نبضة ألف مائة من أكثر ينبض وقلبا

سحريا حال اريد يل تقول مث نفسك كافأ (

Page 92: ˘ˇˆ˙ - Ain Shams Universitymed.asu.edu.eg/uploads/med/Student Affairs/IEE_IO__2.pdf• Lt. Lower lobe bronchiectesis • Recurrent pul. Embolism • Chronic broncitis (May 96)

Faculty Of Medicine - ASU

Student Union Scientific Committee (SUSC)

Examination Note - 4th year P a g e | 92

اإلجناز هذا على نفسك تكافئ أن فيستحسن ممتاز درجه على احلصول لنفسك حدد إذا فمثال : الشخصية املشكالت حل حاول (9 ( افكارك ترتب مكتبك رتب ) : عندك الصرب تنميه حاول(

: بذكاء وجباتك تناول (5 ( ذلك يف االختصاص اصحاب اىل ارجع )

( جناحك يف السبب تكون قد معلومة غياب ، حماضرة غياب( الدراسية مقرراتك حضور على واظب (6 لنفسك دراسيا جدوال ضع (8

االيام خالل دراستك يف ونوع ، الدوام وايام العطل ايام تراعي ( قلبني من ألحد اهللا جعل وما( الدراسة ءأثنا التركيز تطوير حاول (9

خطأ ، احملاضرات ، االغاين مساع مع الدراسة للدراسة مناسبا مكانا اختر (10 هذه يف تنجز كم املهم ، تدرس ساعة كم املهم ليس أجيب كنت للدراسة الالزم الوقت عن اسأل كنت حينما ( 11

أجنزت قد وأنت إال اليوم ميضي أال املهم ، ساعات ست يف تنجزه وقد ساعات ٣ يف منك املطلوب تنجز فقد ، الساعة واجباتك

آخر من ابدأ ، العكس من ابدأ ، ألخره يصل حىت الكتاب أول من يبدأ دائما الطالب فإن: الدراسية عاداتك تغيري (12 من وابدأ ، الوحدة دروس مستوى لىع العكس من وابدأ ، تدرجييا يقل وانتباه االنسان تركيز معدل ألن الكتاب ألول الكتاب معلوماتك ثبتت قد فإنك الطريقتني بكال املادة استيعاب استطعت اذا النقاط مستوى على العكس

مبعزل منفصلة وادرسها وسجلها صغري دفتر يف ضعها( نسياا سرعة ، الفهم صعبة ، الفهم عسرية النقاط (13 ثانية وادجمها الكتاب ارجع مث جيدا واحفظها وافهما ، الدرس عن

ركز ، أوال الفكرة افهم) مبفتاحك اغلقته أن ظهرك فوق البيت حتمل ألن داعي ال( املفتاح استرتيجية واستخدم افهم (14 اجعل( كاملة الفكرة تكمل ان تستطيع تتذكرها حينما االمتحان يف ألنك معينة كلمات على الفكرة نقاط حفظ يف

مفيدة مجلة لتؤدي الكلمات تلك مجع تستطيع فإنك ميدالية عمل تستطيع وكما) مفتاحا فكرة لكل

صوتك ارفع بصمت تدرس كنت ان ، وامشي قف جالسا كنت ان وضعيتك من غري دراستك اثناء بامللل تشعر ال كي (15 احلركات ببعض قمو الشمس حتت قليال خترج ال ملا غرفة يف كنت ،أن ذهنك يف املعلومة يركز ألنه نفعا االكثر وهذا

الرياضية

ارباكك على يعمل هذا فكل واالمتحانات والكورسات وامللخصات االوراق بآالف حتتفظ وال واحدة نسخة من ادرس (16 تتذكر االحيان من كثري يف ألننك) ، صورة صورة الكتاب صور ختزن كامريا عينك وكأن ختيل كتابك نسخة يف تركز حينما )

النقاط تيبتر شكل من االجابة

ذاكرتك يف ستختزن ألا ذلك يف وابدع والتصاميم واالشكال التوضيحية بالرسومات استعن ( 17

: املالحظات تدوين مهارة تعلم (18 ، مهم: جنمة: اخلاصة رموزك استخدم ، خطوطا ضع ، التحديد اقالم استعمل ، جديدا كتابا تؤلف ال ، مالحظاتك دون

وهكذا ، مهم غري : x ، حفظ: صح

Page 93: ˘ˇˆ˙ - Ain Shams Universitymed.asu.edu.eg/uploads/med/Student Affairs/IEE_IO__2.pdf• Lt. Lower lobe bronchiectesis • Recurrent pul. Embolism • Chronic broncitis (May 96)

Faculty Of Medicine - ASU

Student Union Scientific Committee (SUSC)

Examination Note - 4th year P a g e | 93

، لك نفسع يعرف ملن تنتبه بأن االمساء كحفظ الدراسي التفوق على تساعدك مهارة: وفعلها املالحظة دقة تعلم (19 ستتكون املمارسة مع( ، السوبرماركت يف فقدت اليت السلعة ما ، صفك يف الغائب هو من ، املعلم يلبس كان ماذا

( عندك ( .... عنقي ربطة تتكون لون كم من حتدد أن تستطيع هل ناآل عينك اغمضت لو: لك سؤال : قصة

توصل معينة طريق يف املرور أشهر ستة وملدة يوم كل اعتادوا مساعديه من وعشرين سبعة أن إديسون توماس اكتشف) جانب على تقوم كرز شجرة هناك وكانت. نيوجرس بوالية) منلوبارك( يف الرئيسية املصانع إىل املصابيح مصنع من

.والعشرين السبعة هؤالء من شخص أي الشجرة هذه بوجود يشعر مل ذلك ومع الطريق، : ويقول

ـ مالحظة أية ـ املالحظة على قدرتنا وضعف املشاهدين، عيون تراه مما ألف من جزءا اليالحظ العادي اإلنسان عقل إن) .يصدق اليكاد

مدخل ؟ العامل يف ذاكرة أسوأ متتلك بأنك يوما شعرت هل · ؟ وآخر درس بني التنقل بسبب التركيز بعدم يوما شعرت هل · ؟ الدراسية املواد بعض بكراهية يوما شعرت هل · ؟ االمتحانات من باخلوف يوما شعرت هل · ..... تتصور مما أكرب فذاكرتك تقلق ال ·

يف يتابع وقد اجلامعة يف سنوات/ ٥ -٢/ من اعليه يضيف وقد ، املدرسة يف سنة/ ١٢/ عادة منا اإلنسان يقضي ليس جهدا منا الفرد يبذل الطويلة املدة هذه وخالل ، األخرى التعليمية الدورات إىل باإلضافة ، والدكتوراه املاجستري

إن ولكن ، وآخر شخص بني متفاوتة التوفيق درجة إن حتى اآلخرون يوفق ال وقد البعض يوفق وقد ، للنجاح بقليل ..... والتعب اخلسائر بأقل املراحل هذه اجتياز على هذا سيساعده للدراسة مناسبة وسيلة جيد أن اإلنسان استطاع

فعالة طريقة جمتمعة تكون ونصائح خططا حتتوي واليت القليلة السويعات هذه خالل من عليه سنتعرف ما هذا . للدراسة

: العصيب اجلهاز برجمة: أوال : السليب والتفكري االجيايب تفكريال .1

: اإلجيايب ، كلمة/ ٥٠٠٠/ من أكثر يوميا نفسه مع يتكلم اإلنسان أن الدراسات أثبتت وقد ، مفيدة بطريقة النفس مع التفكري هو

. والتركيز احلفظ على يؤثر مما وخوف وتردد تشاؤم فيها سلبية منها%٨٠ على قادر غري أنا( مثل أحكاما تطلق وال السلبية اإلحياءات واحذر ، تؤجل وال) سوف( كلمة قاموسك من احذف: احلل

( فاشل أنا – وقت يوجد ال – ضعيفة ذاكريت – احلفظ ؟ وكيف ؟ ملاذا نفسك اسأل وإنما ؟ وقيت أنظم كيف ؟ ذاكريت أقوي أن ميكنين كيف ؟ املعيقات هذه أزيل كيف ؟ قادر غري أنا ملاذا ؟ وأتذكرها أحفظها اليت األشياء عدد ما: اسأل ؟ أنساها اليت األشياء عدد ما تسأل أن من وبدال

( املشكلة يف يفكر والفاشل احلل يف يفكر الناجح )

Page 94: ˘ˇˆ˙ - Ain Shams Universitymed.asu.edu.eg/uploads/med/Student Affairs/IEE_IO__2.pdf• Lt. Lower lobe bronchiectesis • Recurrent pul. Embolism • Chronic broncitis (May 96)

Faculty Of Medicine - ASU

Student Union Scientific Committee (SUSC)

Examination Note - 4th year P a g e | 94

االستدعاء وكذلك عاما ثالثني من توفي قد رجال ترى فأنت ، األحالم هي حجمها وكرب الذاكرة سعة على دليل واكرب . املفاجئ

: اإلجيابية للروح الست بالقواعد فعليك السلبية النظرات هذه تغير أن أردت ذاإ

وجرب حيفظون للذين انظر) الذهنية اخلريطة( احلفظ على قادرين غري اننا نعتقد: الكاملة احلقيقة ليست اخلريطة .1 العمل على تؤثر السلبية الرسائل ألن يبإجيا معتقد إىل السليب معتقدك غير) الواقع( احلفظ على قادرا نفسك ستجد . ( الضفادع قصة( داخلية أم خارجية واحلفظ

....و أطباء جتد فلن لغريه العلم منا كل ترك إذا: مسؤول أنت إذا جدير أنت .2 . يل ممكن لغريي ممكن هو ما .3 ( جيرب مل من هو يفشل ال الذي: وخربات نتائج ولكن فشل اليوجد .4 ( الباب ذلك سيفتح الذي هو األخري املفتاح يكون فلربما تيأس ال( سونإيدي

: تقول اليابانية اجلنون وحكمة ، متغيرة الطريقة ولكن ثابت اهلدف: املرونة .5 ( خمتلفة نتائج ويتوقع مرة كل يف نفسه الشيء يفعل من انون )

اهلدف إميان عمل ؟ اهلدف حتقق هل نعم نتيجة ال

اهلدف حتديد نةمرو كل تتجاوز حياة منهم كل يعيش لكي البشر لدى داخلي توق من ينبع اإلنسانية احلضارة عاشته الذي التقدم كل )

. ( عليها حتصل اليت النتائج تغير فلن دائما تفعله ما تغير مل إذا) ( احلالية إمكانام . ختدمين نتيجة؟ ختدمين بطريقة تفكري+ حدث: حيدث ما جتاه تفعله ما وإمنا حيدث ما املهم ليس .6

. واحلفظ السابقة املواد مراجعة اجل من فرصة فرآها رجله كسرت طالب: مثال ( البناءة األفكار هو اهلدامة األفكار ضد والفعال املؤثر الوحيد السالح ) دائما األوىل ألن الضرورية غري األفكار من والأ نتخلص أن جيب ولكن ، الضرورية غري األعمال من فقط نتخلص أن اليكفي )

( للثانية نتيجة : السلبية األفكار مع تتعامل كيف

. اإلحباط ودرجة السليب املعتقد اكتب .1 صحته عدم على الدليل اكتب .2 فيه الشك اغرس .3 ( اليوم سأبدأ فعال أنا – وقيت سأنظم – قادر أنا – احفظ فيين أنا( إجيابيا معتقدا منه بدال اكتب .4 بني مييز ال الباطن فالعقل: اجلذب قانون يعمل وهنا) احلسية الكثافة( تسمى ما تنشأ كي االجيايب املعتقد هذا كرر

القناعات تشكلت كيف نفهم فحني ، قناعة االجيايب املعتقد هذا من جتعل أن وعليك) الليمونة( واخليال احلقيقة : إجيابية بقناعات نستبدهلا أن عنستطي السلبية ( يعتقد ما نتاج اإلنسان( مصري؟ شخصية؟ عادة؟ فعل؟ القناعة

جائزة ستأخذ إنك لك وقيل ، صباحا التاسعة الساعة حىت عادة تنام: مثال سيتحقق نريد ما على دماغنا برجمنا فإذا ؟ ستفعل فماذا ، كامل أسبوع ملدة صباحا السادسة الساعة من استيقظت إذا لتجين شوكا تزرع فال بذرة عن عبارة فكرة كل( سلبيا موقفا ال ذاتيا جهدا تتطلب تفكري عملية هو اجليد احلفظ أن اعلم . عنبا : النفسية الصحة: ثانيا

: اخلوف فاملعلومات. الدراسية أهدافك حتقيق يف صعوبة تواجه ال حىت الذهين تركيزك يف بالتدخل واخلوف للقلق تسمح ال

أيضا وسيطر للخطر اإلنسان يتعرض حني اجليم على السيطرة يتولى حيث الدماغ جذر إىل احلواس طريق عن تدخل

Page 95: ˘ˇˆ˙ - Ain Shams Universitymed.asu.edu.eg/uploads/med/Student Affairs/IEE_IO__2.pdf• Lt. Lower lobe bronchiectesis • Recurrent pul. Embolism • Chronic broncitis (May 96)

Faculty Of Medicine - ASU

Student Union Scientific Committee (SUSC)

Examination Note - 4th year P a g e | 95

الدماغ ويسخر ، الفرعية املهام مجيع الدماغ جذر يوقف اخلوف موقف ففي ، التنفس عملية وعلى القلب دقات على . به احملدق طراخل عن اجلسم إلبعاد املتاحة طاقاته

اليت) الكورتيسول( و) األدرينالني( مادة اجلسم يفرز حيث ، ثا/ م١٢٠ بسرعة الدماغ يف تنتقل العاطفية املعلومات إن العصيب اجلهاز شرايني من الدم بإرسال اجلسم فيبدأ) . الشارع وقطع الشاحنة: ( مثال. تقريبا التفكري عملية توقف

. اجلسم حياة إنقاذ اجل من األطراف إىل الدم يوصل كي النبضات القلب ويسرع ، الفعل لردة بللتأه األطراف إىل ، الرعب أفالم( الدماغ خباليا األذى إحلاق إىل تؤدي) الكورتيسول – األدرينالني( اجلسم يف) الكيماويات( نسبة زيادة إن -

هذه إفراز إىل تؤدي واألحاسيس والعواطف فاملشاعر دماغال على سلبية آثار من هلا وما) االمتحانات من اخلوف . الكيماويات

ال حىت بكثري ليس ولكن منك أكثر املتميزين من فقط صديقني أو صديق عن احبث: الوقت وقتلة السوء رفقاء احذر - واجعل ، باألوىل ظاهرتت ال الثانية تريد كنت فإذا اجتماعية جمموعة أم دراسة جمموعة هي هل وحدد ، باإلحباط تصاب . اجلاد عملك يستغل أن ويأمل ممكن قدر أقل يعمل أن يريد الذي بالشخص لك حاجة وال ، وجدية صارمة االجتماع أوقات

( اجلميع يتثاءب احدهم تثاءب إذا أنه االجتماعات يف كامال اتفاقا يلقى الذي الوحيد الشيء ) ( الصعاب ويتجاوزون املشاق يتحملون أقوياء أناس مبساعدة إال املنال بعيدة أهدافك إىل تصل لن ) ( املشكلة من أهم االجتماعات تصبح الزمن مرور ومع ، االجتماعات من كثري عقد يف املشكلة تتسبب ) طريق عن إال التفكري يتعلم أن يستطيع ال واإلنسان ، املمارسة طريق عن إال السباحة تستطيع ال: الذايت النشاط -

. املاء يف جسمك تبلل حىت السباحة تتعلم لن ألنك واجلهد بالعمل فابدأ ، ممارسته . ( تستطيع ما كل وافعل ، تسمع ما كل تصدق وال ، ماتعرف كل تقل ال )

املعرفة وسائل أهم وهي الكائنات من غريه عن اإلنسان ا امتاز القراءة أن واعلم يف إال اجلهد من جمردة فرص توجد فال ، العمل مبالبس دائما تأتيهم ألا تواجههم اليت الفرص معظم الناس خيسر )

.( النيام إال األحالم يف الفرص يقابل وال ، األحالم على قدرتك بازدياد سيشعرك ذلك ألن حتفظ ما تسميع على مصمم أنك احلفظ يف البدء قبل لنفسك تؤكد أن جيب -

. التعب من اجلسم نال مهما محاس يف العمل إىل العقل يدفعان انواألم فالطموح ، احلفظ وسرعة التركيز ( للحاضر معىن هناك يكون لن حتما املستقبل يف أمل لديك يكن مل إذا ) ( أهدافك تناسب اليت بالقدرات فكر بل ، قدراتك تناسب اليت باألهداف تفكر ال ) ( القدوة( األعلى املثل خذو ، متفوق وأنت فيه تتخرج الذي اليوم تتخيل أن جرب - ( عظيم هو ما إىل التطلع عن يصرفك جيد هو مبا االكتفاء ) ( الكثري تنجز الكثري نتوقع فعندما ، العالية التوقعات شكل تأخذ والسامية العالية اإلجنازات )؟ ) املتوقعة غري النتائج( أي االجيابية النواحي على وركز ، القصرية األهداف حتقق أن تستطع مل إذا مهتك تفتر ال -

. بالنفس الثقة مضاعفة . اآلخرين على واعرضه بعملك فخورا كن - . حتفظه ما مجيع تتذكر أن تستطيع أنك التدريب مع وستجد احفظ ، بذاكرتك ثقتك فتفقد جبانا تكن ال - ال( انه وتأكد) إلقناعهم الوحيد لطريقا هو الناس نفوس إىل والسعادة البهجة إدخال( ألن عائلتك مع عالقتك حسن -

. ( ضغطه يتم مل ما شيء أي يدفع أن الغاز يستطيع ال فمن) نفسه على اإلنسان انتصار عدا ما ، هزمية تقابلها االنتصارات كل( ألن عليك تسيطر الشريرة األفكار جتعل ال -

. يتعلم ال خيطئ باإلحباط يصيبك مما احلقيقي الزمن عن وخيتلف قصري األفالم زمن ألن لسهلا النجاح لك تقدم اليت األفالم عن ابتعد -

. والتعب اجلهد من الرغم على . والعمل التحفيز لزيادة حيلة وهذه ، الدراسة مهارات صقل يف اجليد بقرارك وأصدقائك عائلتك أخرب - ماء بركة يف أو باملرآة إال نرامها أن نستطيع ال ولكن ، ننابأعي أعيننا نرى أن أي ، بالعقل العقل لرؤية حماولة هو التفكري -

. ( والسلوك والفعل العمل( هي املرآة وهذه ، صافية

Page 96: ˘ˇˆ˙ - Ain Shams Universitymed.asu.edu.eg/uploads/med/Student Affairs/IEE_IO__2.pdf• Lt. Lower lobe bronchiectesis • Recurrent pul. Embolism • Chronic broncitis (May 96)

Faculty Of Medicine - ASU

Student Union Scientific Committee (SUSC)

Examination Note - 4th year P a g e | 96

. الراحة من قسط بأخذ العمل يف جيدة درجات على احلصول عند نفسك كافئ - . اآلن تفعله شيء كل تغري ألن مستعدا كن - . اآلن تبدأ أن هي الوقت الستثمار طريقة أفضل أن تذكر -

: االمتحاين القلق ( عالجها – ظهورها على املساعدة العوامل – احلالة أعراض )

سلبية فكرية وانشغاالت باخلوف تتصف نفسية حالة وهي واألهل للطالب القلق أعراض بعض االمتحانات فترة يصاحب . العقلية املهام على سلبا يؤثر مما التركيز مع تتداخل

: الطبيعي غري القلق حالة أعراض - . الوسواسية األفكار بعض وتسلط الشهية وفقدان بالضيق الشعور - . والغثيان البطن وأمل األطراف وبرودة ، اليدين وارتعاش العرق وتصبب والشفتني احللق وجفاف القلب خفقان تسارع - : أسباا - دارسون بركس( قانون جند وهنا طبيعي قلق هو القلق وهذا يةللشخص ديد موقف االمتحان موقف يف يرى الطالب أن -

التركيز ضعف الطبيعي غري القلق زاد وكلما. واألداء التركيز مستوى زاد الطبيعي القلق زاد كلما( أنه على ينص الذي)

) : احلالة ظهور على املساعدة العوامل - . القلقة الشخصية - . لالمتحان الطالب استعداد عدم - . نتائج من عليه يترتب وما االمتحان عن اخلاطئة التصورات - خالل من واملعلمني األسرة قبل من االمتحانات من اخلوف تعزيز -

. العقاب . القلقني لنموذج تقليد خالل من اآلخرين من االجتماعي التعلم -

: العالجية األساليب . بأنفسهم الطالب ثقة تعزيز - ( راجع – مسع – اقرأ – تساؤل – تصفح( بالدراسة) تتسر( طريقة اتباع - .( احلسنة القدوة( النموذج توظيف - مجيلة صورة ختيل: اإلحيائي التخيل خالل من العميق االسترخاء حالة على والتدريب السليمة غري بالطريقة التنفس -

. للقلق املثرية راألفكا سيطرة دون حتول باالمتحان ارتباط هلا ومتحركة مرحية : والتمرينات - عضالت من صغرية جمموعة وإرخاء العميق بالتنفس البدء مث مريح مكان يف العينني وإغماض ، الظهر على االستلقاء -

. الراحة تبعث داللة ذات معينة فكرة على ومركزا) اليدين بع\أصال من تبدأ( اجلسم . االمتحاين املوقف على عممت سابقة سلبية خربات عن الناجتة ةواخلاطئ السلبية األفكار عن االبتعاد - ( شاي – قهوة( املنبهة املشروبات من التقليل - . فقط تقييم وسيلة هو االمتحان بأن والعلم ، العالية الدرجات حتصيل يتركه الذي الطيب األثر على التركيز - . االمتحان ليايل طيلة النوم من الكايف القسط أخذ - . االمتحان قاعة دخول قبل املادة من جزء أي مراجعة عدم - . متخمة أو خالية معدة على االمتحان إىل الذهاب عدم -

: التغذية: ثالثا مستوى رفع اجل من يأكل من ولكن ، األكل مبتعة اإلحساس أجل من األطفال ويأكل ، اجلوع أجل من يأكل اإلنسان إن ·

( لتأكل كل( وليس) لتتعلم لك( ؟ والتفكري التعلم

Page 97: ˘ˇˆ˙ - Ain Shams Universitymed.asu.edu.eg/uploads/med/Student Affairs/IEE_IO__2.pdf• Lt. Lower lobe bronchiectesis • Recurrent pul. Embolism • Chronic broncitis (May 96)

Faculty Of Medicine - ASU

Student Union Scientific Committee (SUSC)

Examination Note - 4th year P a g e | 97

حوايل يستهلك أنه إال البالغ اإلنسان وزن من% ٢ سوى يشكل ال الدماغ أن من الرغم وعلى ،% ٨٧ الدماغ يف املاء نسبة إن - يستهلك فهو ، اليوم يف/ جالون ١٩٢ أي ، سا/ جالونات ٨ إليه تصل اليت الدم كمية تبلغ حيث ، اجلسم طاقة من% ٢٠ إىل يوميا حيتاج إنتاجه ذروة إىل الدماغ يصل ولكي ، الدماغ يف التوازن من حالة إجياد على املاء ويعمل ، اجلسم طاقة ربع ( لتتعلم اشرب) ( والنعاس الكسل( اجلسم يف املاء نقص ويسبب ، املاء من كأس) ١٢ -٨(

. السريعة اجلاهزة املأكوالت عن امتنع - فالقهوة ، جديد من بط ما سرعان ولكن ، القصري املدى على وتركيزك انتباهك تصعد ألا ، املنبهات من خفف -

التنفس لدور نأيت وهنا) والسكر األكسجني( هو الذاكرة وغذاء وهيدروجني أكسجني هو واملاء ، اجلسم من املاء تسحب ( تنشيطي – تفريغي: ( الذاكرة يف

: التعلم على الدماغ درةق من تزيد اليت املأكوالت من -

زيت – اللوز – احلليب الفستق – اجلوز – الدهون من اخلايل األمحر اللحم – الفواكه – األمساك – الورقية اخلضراوات ) - . األمد الطويلة الذاكرة يف املعلومة تثبيت إىل تؤدي اليت) األسيلكولني( مادة على حيتوي الذي: الزيتون

الناقلة احملاور بتنظيف تقوم اليت) الكالبني( مادة لىع حيتوي الذي: اللنب : السحرية الوصفة

. الريق على ملعقة تؤكل) زبيب+ صنوبر+ عسل كغ مع ختلط – الربكة حبة غ٥٠ – زجنبيل غ50 ) : والتعلم الفضالت طرح إىل البول من خاليا الصايف الدم يصل وهكذا إلخراجه متهيدا املثانة يف وخزنه الدم من البول باستخالص الكليتان تقوم

نسبة وترتفع ، جزئيا ولو الكليتني عمل يتوقف بتفريغها اإلنسان يقوم وال بالبول املثانة متتلئ عندما ولكن ، الدماغ من وختفف الدماغ تؤذي اليت الضارة األمالح من نسبة على حيتوي الدماغ إىل يصل الذي الدم أن يعين وهذا الدم يف البول . والتفكري التعلم على قدرته : املكان: رابعا

. الطبيعي النهار ضوء يف القراءة واألفضل ، للعني محاية أكثر األبيض الضوء ألن) اللمبات( األصفر الضوء جتنب -1 ويستمر اطهبنش القارئ حيتفظ حىت د) ٢١ - ١٥( بني حرارا تكون حبيث التهوية حسنة الدراسة حجرة تكون أن جيب -2

. طويلة لفترة . بالتسلية الدراسة اختلطت وإال املرتل يف واحلركة الضوضاء أماكن عن البعد جيب -3 – الذباب – الدقاق - املنبه ذات الساعة( أمامك تتحرك اليت األشياء أبعد: والتركيز الذهن مشتتات عن االبتعاد جيب -4

( الشموع أن وجيب ، باملغادرة الرغبة وتولد الكآبة تبعث املبعثرة األشياء ألن ومرتب نظيف) كتبم – طاولة( مريح عمل سطح -5

تتراوح حيث ، مستقيم ظهر مسند وله باللني مفرط وال القسوة يف مفرط هو ال كرسي على منتصبا اجلسم يكون . املثالية احلالة وهي سم) ٣٥ -٢٥( والنص عينيك بني املسافة

بالقيام الوقت تضيع ال حىت الطاولة على) النوط( مثل الالزمة األدوات وضع ، دراستك يف ترتاح تككراس نظم -6 . والقعود

: التذكر وسرعة احلفظ لزيادة العملية الطرق:خامسا

( واحد كتاب( واحدة نسخة من احلفظ .1 أنك ومبا للدراسة املخصص الوقت من قليال نكا ولو شيء إهدار وعدم ، للجلوس األوىل اللحظة يف بالدراسة البدء .2

. احملدد الوقت من االنتهاء فعليك الدراسة بدء جددت ( ساعة خالل الرابع الدرس سأي: ( مثل هلا الالزم والوقت ، موضوعات من ستدرس ما حتدد أن جيب .3 . الدرس يف األساسية النقاط على تعرف .4 . احفظ مث وحدات إىل النص قسم .5

Page 98: ˘ˇˆ˙ - Ain Shams Universitymed.asu.edu.eg/uploads/med/Student Affairs/IEE_IO__2.pdf• Lt. Lower lobe bronchiectesis • Recurrent pul. Embolism • Chronic broncitis (May 96)

Faculty Of Medicine - ASU

Student Union Scientific Committee (SUSC)

Examination Note - 4th year P a g e | 98

احلواس وتوجيه البصرية احلركات تتابع البشرية فالعني ، اهلامة النقاط حتت إشارات لتضع القلم بيدك أمسك .6 تبدأ عقلي عمل فالقراءة. والعقل البصر بني أو والعني املخ بني احتاد إىل يؤدي القلم رأس وهو واحد مكان يف وحصرها ( اإليقاعية( والسمعية البصرية لصورا هي قوة الذكريات وأكثر بالنظر

. جهرية قراءة وبوضوح باحلركات اقرأ أي الغمغمة عن ابتعد .7 الدرس اية يف املوضوعة األسئلة على جتيب أن تستطيع حبيث تاما فهما الدرس فهم من التأكد .8 املعلومات ترتيب من الالواعي العقل يتمكن كي/ دقيقة مخسني أو ساعة كل بعد/ د) ١٠ – ٥( بني ما راحة فترة خذ .9

. الترتيب فترة تطول؟ الدراسة فترة تطول فعندما ، ختزينها مت اليت . األول املوضوع من كبري قدر ونسيان جديد مبوضوع الذهن انشغال ?

التذكر يسهل ممنظ وبشكل واحدة ورقة على تريده ما رسم على تعتمد رائعة طريقة وهي: الذهنية اخلارطة .10 . مركز بشكل ورقة يف املعلومات من ممكن قدر أكرب تضع وجتعلك ، الدراسة موضوع عن شاملة صورة وتعطيك

استقبال إىل الذاكرة فتخيل أخرى خلية بألف ترتبط إا يقال خلية فكل ، شبكي شكل هو الدماغية اخلاليا شكل إن واألشكال األلوان اخلارطة هذه يف تستخدم أن وجيب أسرع بشكل جاعهاواستر ختزينها ليتم التشعبية الشبكية األمور

شطرين إىل ينقسم الدماغ ألن واألعداد اخليال – األلوان – باإلبداع ويهتم: األمين الشطر . التخطيطي الرسم – التسلسل – األعداد – والكالم باملنطق ويهتم: األيسر والشطر

( نياتهإمكا نصف يستخدم إنسانا ختيل) تليها باليت فقرة كل أواخر وبصريا صوتيا اربط ، احملسوسات أو باملكان الربط وهو) الرومانية احلجرة نظام: ( الربط .11 . الصويت التكرار خالل من الذاكرة يف سوخهار؟ الطويلة إىل نقلها مث ومن ، املدى القصرية الذاكرة يف احملفوظة املعلومات إحياء: فائدته: التكرار .12

. . التذكر يف خطأ إىل يؤدي ألنه النطق أو اللفظ يف اخلطأ وعدم) الفهم مع( واعيا التكرار يكون أن جيب : اخلاليا من نوعني من يتكون الدماغ إن ١٠٠( لبالغا اإلنسان عند عددها ويبلغ) عصبون( الواحدة وتسمى ، التفكري بعملية وتقوم% ١٠ وتشكل: العصبية - أ

( تسمى مادة عليه تترسب الذي) األكسون( أو) الناقل احملور( يسمى ما طريق عن بعضها مع وتتصل ، عصبون) مليار حفظ؟ ضياعها أو تشتتها دون الناقل احملور داخل املعلومات انتقال يسهل مما الناقل احملور بعزل تقوم واليت) املايلني

. املعلومة وهذا العزل يزيد مما ، الناقل احملور حول املايلني مادة ترسب تزايد؟ للمعلومات العصبونية الشبكة ستقبالا تكرار إن

( الكهربائية األسالك يشبه وهذا( واالستدعاء التذكر يسرع . حلركتها مناسب وسط وتوفري العصبونات بتغذية وتقوم ، الدماغ خاليا من% ٩٠ وتشكل: الصمغية - ب

، قبل ذي من أكثر متقاربة بفترات تراجع أن جيب تعلمته ما معظم نسيت انك وشعرت باملراجعة بدأت إذا: ملراجعةا .13 : إىل تؤدي املراجعة ألن أقوى التذكر كان قصريا واملراجعة التخزين بني الزمن كان فكلما ، بيوم يوما راجع

احلفظ مقدار بيان - أ النجاح متعة؟ احلفظ يف باملتعة أكرب داجه بذل على حافزا تعطي - ب

. والتثبيت التخزين على يساعدان واالستراحة النوم ألن حفظت ما استرجع مباشرة النوم قبل ؟ شيئا أجنزت هل ؟ أخرى مرة اليوم هذا يعود أن تريد هل: واسأل النوم قبل يومك قيم .14 ؟ التايل اليوم يف خمتلفة بطريقة تقوم أن تريد هل يف تفكر مل إذا( اهللا شاء إن دروس) ١٠( مثال التايل اليوم يف واحفظ ، نشيطا استيقظ أن أريد: النوم قبل دماغك شوق .15

( املستقبل يف مستقبال لديك يكون فلن املستقبل . املتقطع من أفضل املنظم اليومي احلفظ - : أن تأكد .16

. التعب يرافقها اليت من نفعا أكثر لصباحا يف مباشرة االستيقاظ بعد الدراسة -

Page 99: ˘ˇˆ˙ - Ain Shams Universitymed.asu.edu.eg/uploads/med/Student Affairs/IEE_IO__2.pdf• Lt. Lower lobe bronchiectesis • Recurrent pul. Embolism • Chronic broncitis (May 96)

Faculty Of Medicine - ASU

Student Union Scientific Committee (SUSC)

Examination Note - 4th year P a g e | 99

بعادات تستبدهلا أن جيب سيئة عادة لديك أن يعين فهذا كبريا وقتا تقضي أنك حني يف دراستك يف ببطء تتقدم كنت إذا - . جيدة

. اإلرهاق أو امللل وقت حتفظ ال - .تقرأ فيما اجلادين الدراسة زمالء مناقشة حاول - . احلاضر بالدرس البدء قبل ملاضيا الدرس راجع - . والضيق للملل دفعا اليوم يف مادة من أكثر دراسة على احرص - الرياضية التمارين بعض ومارس آلخر حني من الدراسة عن توقف -

اجلسد رياضة- الوراء واىل األعلى إىل دائما ذراعيك مد . . دقائق بضع الدراسة مكان غادر - .( املراجعة – التلخيص – التكرار – الربط – التخيل – الراحة – احلماسة( ب التذكر ادعمو النسيان قاوم - . األولويات عليه حتدد شهريا ، أسبوعيا ، يوميا جدوال وضع وقتك نظم -

العني رياضة ؟ عينيك عن تعرف ماذا: الرائعتان عيناك

؟ تقرأ أن تستطيع سرعة بأي املختلفة؟ األلوان من ماليني عشرة حوايل بني متيزا أن تطيعانتس عينيك أن تعلم هل

: التركيز على العني لتدريب التمارين بعض : رأسك حترك أن ودون إجهاد دون وخف بسرعة انظر - اليسرى العني زاوية إىل اليمىن العني زاوية من مرات ١٠ - أ ليمىنا العني زاوية إىل اليسرى العني زاوية من مرات١٠ - ب . اليمىن العني زاوية إىل اجلبهة باجتاه اليسرى العني زاوية من مرات١٠ - ت . مرات) ١٠( احلالة كرر مرة معكوس وبشكل مرة الساعة عقارب باجتاه ترمسهما وكأنك عينيك شكل - ث كي ، شديد ببطء اليدين نيب ما افتح وبعدها ، كامل باسترخاء الظالم إىل دقائق بضع اآلن انظر مث بيديك عينيك غط -

. فشيئا شيئا الضوء على عيناك تعتاد . واسترخائها طراوا على لتحافظ بعينيك اغمز -

: الدراسي اجلدول: سادسا . والصحية النفسية والراحة املمتاز التحصيل يوفر فهو الدراسي والتفوق النجاح على تساعد اليت القضايا أهم من

( الوقت إدارة إساءة عن ناجتة والندم السيئ ظاحل حاالت معظم ) تنظيم بني الفرق يدركوا مل ألنهم ، اتباعها يستطيعوا مل ولكن ، اجلدول إلعداد ناجحة مبحاوالت مروا الطالب من كثري

. خاص بشكل والدراسة املذاكرة جدول وبني ، عامة الوقت : فوائده

. رسوبك يف سببا فتكون املواد إحدى مل ال يثحب مادة كل لدراسة الالزم الوقت حتديد - . واملراجعة والتسميع للحفظ الكايف الوقت لك يتوفر وبذلك ، الدراسي العام من ساعة كل أمهية إدراك - . االضطراب عن أيضا وتبتعد احلفظ مادة اختيار يف الوقت ضياع عن تبتعد حيث القراءة مواد حتديد - . احلياة شؤون مجيع تنظيم على بالقدرة الشعور -

: للجدول الرئيسة األبواب العمل على قدرا استعادة املخ خاليا تستطيع حىت متواصل بشكل األقل على ساعات ست: للنوم الالزم الزمن .1

. ينسى ما سريعا لكن الكبري اهود من الرغم على بأنه الفرد يوهم فاإلرهاق ، والتثبيت والفهم

Page 100: ˘ˇˆ˙ - Ain Shams Universitymed.asu.edu.eg/uploads/med/Student Affairs/IEE_IO__2.pdf• Lt. Lower lobe bronchiectesis • Recurrent pul. Embolism • Chronic broncitis (May 96)

Faculty Of Medicine - ASU

Student Union Scientific Committee (SUSC)

Examination Note - 4th year P a g e | 100

/ ٦/ الساعة يف ملرة تستيقظ أن منك وطلب ، صباحا/ ٩/ الساعة يف تستيقظ عادة أنك فلو ، نومك اتعاد يف انظر . ربطها يتم شاء كيفما ساعة للدماغ إن ؟ ستفعل ماذا معينة مادة يف تنجح أن مقابل

على اعدكتس الصحيحة اإلجابة ؟ للدراسة حتتاج الوقت من كم: نفسك اسأل: للدراسة الالزم الزمن .2 وقتك تنظيم ؟ الدراسة يف تقضيه الوقت من كم

( لنشاطك خمططا ضع ) الذهن تشتيت إىل يؤدي مما مبكرا استيقاظه لعدم) اإلفطار( الطعام يهمل من الطالب من كثري: للطعام الالزم الزمن .3

. اهلدوء أو االستقرار وعدم . تعقبها اليت الراحة مهيةأ إدراك مع دوء تناوهلا جيب: الغداء وجبة تناول من مانع وال ، النوم يف والرغبة بالكسل تشعر ال -االستقرار - حىت وخفيف مبكر بشكل تكون: العشاء وجبة

. بالفيتامينات الغنية السوائل مع الراحة فترات أثناء السندويشات إىل وترسلها باحلركة املتعلقة املعلومات جتمع اخليةالد األذن قنوات ألن واحلركة بالرياضة قم: الفراغ أوقات تنظيم .4

الداخلية اإلذن يف السائل حتريك على تساعد الرياضية والتمرينات ، الدماغ أجزاء بقية على بتوزيعها يقوم الذي املخيخ . أفعال إىل التفكري وحتويل اجلسم حركات توازن؟ والتفكري احلركة بني التفاعل تنشيط إىل يؤدي مما

: اجلدول تنفيذ يف وأعداؤك الوقت لصوص

. سلبيون) أقارب – أصدقاء( احمليطون .1 . حبزم معها التعامل جيب وهنا) تلفاز – راديو( التسلية وسائل .2 أثار مبوضوع تتحدث األسرة مسع؟ إخوته سأل؟ املرباة عن حبث؟ مكسور قلمه؟ الدراسة وقت أتى: التافهة اإلضاعة .3

له أعادت؟ الصور بعض ملح أو ، مساعه من ضرر ال أنه نفسه أقنع؟ إذاعيا برناجما تذكر؟ معهم اخنرط؟ مهاهتما . الذكريات

( ... الكوخ فيغدو أحبب( وللمدرسني الدراسية املواد لبعض كراهيتك .4 جناحهم وكان بتفوق جنحوا ونكثري يوجد أنه تأكد ولكن ، فيها السابق الفشل بسبب املواد بعض بصعوبة شعورك .5

( ..... فشل ال( السابق فشلهم على قائما . منها التخلص مث حتديدها أو معرفتها جيب: اخلاصة املشاكل .6

: اجلدول يف االستمرار اجل من . طاقاتك حدود يف يكون أن جيب - . مادة كل لدراسة الالزمة الساعات عدد مالحظة - . فيها بالضعف تشعر اليت للمادة األكرب الوقت توفري - . دراسية ساعة كل بعد الراحة أوقات إىل االنتباه - . تثبيتها؟ اهلامة األجزاء ملراجعة النوم قبل ساعة نصف عن يقل ال وقت وضع -

. ومتدرج ممكن أمر األهداف لنيل العمل: أن واعلم ( املرونة( اجلديدة األوضاع مع والتكيف العقبات من الرغم على بالعمل االستمرار - . مشروعك يف تستمر جتعلك اليت اإلرادة امتالك - . األخطاء من للتعلم االستعداد -

Page 101: ˘ˇˆ˙ - Ain Shams Universitymed.asu.edu.eg/uploads/med/Student Affairs/IEE_IO__2.pdf• Lt. Lower lobe bronchiectesis • Recurrent pul. Embolism • Chronic broncitis (May 96)

Faculty Of Medicine - ASU

Student Union Scientific Committee (SUSC)

Examination Note - 4th year P a g e | 101

. يوم كل جديد شيء تعلم على العمل - . واالنفعاالت بالعواطف التحكم - . الوقت لصوص من وهم النجاح لذة املرء حيرمون الذين األشخاص من التخلص - . الراحة بفترات والتمتع العمل يف التفاين بني عاجلم - . بك احمليطني مع التفاهم - . القرار اختاذ حال يف العمل بدء -

Page 102: ˘ˇˆ˙ - Ain Shams Universitymed.asu.edu.eg/uploads/med/Student Affairs/IEE_IO__2.pdf• Lt. Lower lobe bronchiectesis • Recurrent pul. Embolism • Chronic broncitis (May 96)

Faculty Of Medicine - ASU

Student Union Scientific Committee (SUSC)

Examination Note - 4th year P a g e | 102

:شكر خاص

�'�� ����� ( �)�* �� +,� -�. �,/� ��0�� ����� ��1&2�� �34&�� ���$5

!�"�#$�6�:

- ���*8 9�1�:

- ;<�0� =�

- �8 �1��

- ��)���� �1��>

- ?�@ !��A

- B��� ���.

- ��)���� C 0D�

- ��&E��F� G�0&�� �F� �8

- �)�� �8

- �H1I J�1�

- K�F� C 0D�

- �8L����� �F� � �F�

- M�H&1��� ��*�

- =�N�L G��:

- M142�� �1I

- �N. OL�� �1I

Page 103: ˘ˇˆ˙ - Ain Shams Universitymed.asu.edu.eg/uploads/med/Student Affairs/IEE_IO__2.pdf• Lt. Lower lobe bronchiectesis • Recurrent pul. Embolism • Chronic broncitis (May 96)

Faculty Of Medicine - ASU

Student Union Scientific Committee (SUSC)

Examination Note - 4th year P a g e | 103

��0�� ����� ��1&2�� �34&�� 9�N�8 ��0�� ����� ��1&2�� �34&�� 9�N�8 ��0�� ����� ��1&2�� �34&�� 9�N�8 ��0�� ����� ��1&2�� �34&�� 9�N�8::::

- �*����� ��� ��) ?�F0�� �1I– C*H� �1I(

- ������ ��� �� ) S3� ���.– TJ�U =�V�L��(

- �2����� ��� �� ) W�5 �1�– X�� YJ��(

- �Z��Z�� ��� �� ) 9�� �8– [4\ KH��](

- ��"�Z�� ��� �� ) C 0D� 96A– �8 G*H�(

- ^J_� ��� �� ) ���� �1��– �8 ��/)(

< <

<솉ù<”^}<†Ó�ÓÊÓÊÓÊÓÊì†ì†ì†ì†<<솉_<æ†Óeçe_†Óeçe_†Óeçe_†Óeçe_<Ö]<Üâ�Êçjý<l^Þ^vjÚ

<Ý^ÃÖ]<îjuNLLU<<�^�^e<íéÛ×ÃÖ]<íßr×Ö<^㵂Ïi<æ<�…ææ<íÇé‘

l^Þ^vjÚ÷]<݇øÚ<�]‚Âc<íÖçã‰<^ßÖ<îߊßjè<êÓÖ<høŞÖ]< <